diff --git "a/data/fr/fr.test_casimedicos.jsonl" "b/data/fr/fr.test_casimedicos.jsonl" new file mode 100644--- /dev/null +++ "b/data/fr/fr.test_casimedicos.jsonl" @@ -0,0 +1,125 @@ +{"id": 190, "year": 2013, "question_id_specific": 94, "full_question": "Un garçon de 18 mois, dont le calendrier de vaccination est complet à ce jour, consulte le service des urgences pour un gonflement du genou droit après avoir joué dans un parc, sans traumatisme évident. Dans l'anamnèse, la mère mentionne qu'un de ses oncles a eu des problèmes similaires. L'examen échographique est compatible avec une hémarthrose et à l'examen analytique seul un allongement du TCA de 52'' (normale 25-35'') ressort. Quelle est l'hypothèse diagnostique la plus probable ?", "full_answer": "Bien que d'autres troubles de la coagulation puissent entraîner un allongement du temps de thromboplastine, sur la base de l'intensité de la lésion, du sexe de l'enfant et de ses antécédents familiaux, le diagnostic le plus probable est celui de l'hémophilie.", "type": "PÉDIATRIE", "options": {"1": "Le syndrome de Marfan.", "2": "Maladie de Von-Willebrand.", "3": "La maladie d'Ehlers-Danlos.", "4": "Hémophilie A.", "5": "Maladie de Bernard-Soulier."}, "correct_option": 4, "explanations": {"1": {"exist": false, "char_ranges": [], "word_ranges": [], "text": ""}, "2": {"exist": false, "char_ranges": [], "word_ranges": [], "text": ""}, "3": {"exist": false, "char_ranges": [], "word_ranges": [], "text": ""}, "4": {"exist": true, "char_ranges": [[0, 259]], "word_ranges": [[0, 41]], "text": "Bien que d'autres troubles de la coagulation puissent entraîner un allongement du temps de thromboplastine, sur la base de l'intensité de la lésion, du sexe de l'enfant et de ses antécédents familiaux, le diagnostic le plus probable est celui de l'hémophilie."}, "5": {"exist": false, "char_ranges": [], "word_ranges": [], "text": ""}}} +{"id": 150, "year": 2012, "question_id_specific": 201, "full_question": "Une femme de 84 ans nous consulte pour une insomnie de conciliation. Après l'échec des mesures d'hygiène du sommeil, il a été décidé de commencer un traitement pharmacologique. Parmi les médicaments suivants, lequel choisiriez-vous pour cette patiente ?", "full_answer": "Sachant que nous avons pris des mesures d'hygiène du sommeil, que nous avons exploré les causes de son insomnie, qu'elle ne dort pas neuf heures et qu'elle souhaite toujours dormir davantage,... la réponse la moins nocive est 2. Parce que c'est celle qui a la demi-vie la plus courte et qu'il ne veut pas que la dame tombe le lendemain.", "type": "PSYCHIATRIE", "options": {"1": "Diacepam.", "2": "Lormetacepam.", "3": "Phénobarbital.", "4": "Chlordiazepoxide.", "5": "Chloracepate."}, "correct_option": 2, "explanations": {"1": {"exist": false, "char_ranges": [], "word_ranges": [], "text": ""}, "2": {"exist": true, "char_ranges": [[195, 336]], "word_ranges": [[31, 60]], "text": "la réponse la moins nocive est 2. Parce que c'est celle qui a la demi-vie la plus courte et qu'il ne veut pas que la dame tombe le lendemain."}, "3": {"exist": false, "char_ranges": [], "word_ranges": [], "text": ""}, "4": {"exist": false, "char_ranges": [], "word_ranges": [], "text": ""}, "5": {"exist": false, "char_ranges": [], "word_ranges": [], "text": ""}}} +{"id": 454, "year": 2018, "question_id_specific": 153, "full_question": "Une femme de 65 ans consulte pour une faiblesse de la main droite qui s'est étendue en quelques mois à d'autres territoires musculaires des deux bras et des deux jambes, principalement distaux. L'examen révèle une atrophie et des fasciculations dans différents territoires métamériques, avec une sensibilité préservée, ainsi qu'un signe de Babinski bilatéral. Le signe de Babinski est bilatéral. Quel est le test diagnostique qui confirmerait le diagnostic suspecté ?", "full_answer": "La réponse est 2, un EMG, qui serait le test diagnostique pour confirmer le diagnostic suspect de SLA. Les tests d'imagerie tels que le scanner, mais surtout l'IRM, nous aideraient à écarter la possibilité d'une SLA, mais ne permettraient pas de confirmer le diagnostic.", "type": "NEUROLOGIE", "options": {"1": "Tomodensitométrie cérébrale.", "2": "Étude électromyographique.", "3": "IRM cérébrale.", "4": "Potentiels évoqués multimodaux.", "5": null}, "correct_option": 2, "explanations": {"1": {"exist": true, "char_ranges": [[103, 270]], "word_ranges": [[18, 44]], "text": "Les tests d'imagerie tels que le scanner, mais surtout l'IRM, nous aideraient à écarter la possibilité d'une SLA, mais ne permettraient pas de confirmer le diagnostic."}, "2": {"exist": true, "char_ranges": [[0, 101]], "word_ranges": [[0, 18]], "text": "La réponse est 2, un EMG, qui serait le test diagnostique pour confirmer le diagnostic suspect de SLA."}, "3": {"exist": true, "char_ranges": [[103, 270]], "word_ranges": [[18, 44]], "text": "Les tests d'imagerie tels que le scanner, mais surtout l'IRM, nous aideraient à écarter la possibilité d'une SLA, mais ne permettraient pas de confirmer le diagnostic."}, "4": {"exist": true, "char_ranges": [[103, 270]], "word_ranges": [[18, 44]], "text": "Les tests d'imagerie tels que le scanner, mais surtout l'IRM, nous aideraient à écarter la possibilité d'une SLA, mais ne permettraient pas de confirmer le diagnostic."}, "5": {"exist": false, "char_ranges": [], "word_ranges": [], "text": ""}}} +{"id": 541, "year": 2021, "question_id_specific": 5, "full_question": "Patient de 25 ans, victime d'un accident de moto dans la nuit de vendredi à samedi. Il a été transporté au service des urgences où l'on a diagnostiqué un traumatisme abdominal (écho-fast négatif), un traumatisme crânien léger (Glasgow = 14) et une fracture intracapsulaire déplacée de la hanche droite. Son état hémodynamique est stable, quel serait le traitement de choix ?", "full_answer": "La prise en charge d'une fracture intracapsulaire déplacée de la hanche chez un jeune patient est chirurgicale et doit être précoce pour réduire le risque de nécrose avasculaire de la hanche. Il ne s'agit pas d'une menace vitale suffisante pour différer une intervention chirurgicale nécessaire ou pour envisager un fixateur externe dans le cadre d'une politique de limitation des dommages ; nous écartons donc l'option 3. Il s'agit d'un patient de 25 ans ; nous devons donc procéder à une réduction et à une fixation en évitant les options de remplacement (nous écartons les options 2 et 4).", "type": "TRAUMATOLOGIE", "options": {"1": "Réduction, ouverte si nécessaire, et ostéosynthèse de la fracture dans les 24 à 36 heures.", "2": "Arthroplastie totale de resurfaçage de la hanche le lundi comme prévu.", "3": "Attendre l'amélioration du traumatisme crânien et programmer la semaine suivante une intervention chirurgicale régulière consistant en une réduction et une ostéosynthèse de la fracture.", "4": "Compte tenu du risque de non-union de ce type de fracture, je procéderais en urgence à une hémiarthroplastie bipolaire de la hanche.", "5": null}, "correct_option": 1, "explanations": {"1": {"exist": true, "char_ranges": [[0, 191]], "word_ranges": [[0, 31]], "text": "La prise en charge d'une fracture intracapsulaire déplacée de la hanche chez un jeune patient est chirurgicale et doit être précoce pour réduire le risque de nécrose avasculaire de la hanche."}, "2": {"exist": true, "char_ranges": [[423, 592]], "word_ranges": [[66, 98]], "text": "Il s'agit d'un patient de 25 ans ; nous devons donc procéder à une réduction et à une fixation en évitant les options de remplacement (nous écartons les options 2 et 4)."}, "3": {"exist": true, "char_ranges": [[192, 422]], "word_ranges": [[31, 66]], "text": "Il ne s'agit pas d'une menace vitale suffisante pour différer une intervention chirurgicale nécessaire ou pour envisager un fixateur externe dans le cadre d'une politique de limitation des dommages ; nous écartons donc l'option 3."}, "4": {"exist": true, "char_ranges": [[423, 592]], "word_ranges": [[66, 98]], "text": "Il s'agit d'un patient de 25 ans ; nous devons donc procéder à une réduction et à une fixation en évitant les options de remplacement (nous écartons les options 2 et 4)."}, "5": {"exist": false, "char_ranges": [], "word_ranges": [], "text": ""}}} +{"id": 602, "year": 2022, "question_id_specific": 112, "full_question": "Une femme de 61 ans, administrative, aux antécédents de surpoids, d'hypertension, de dyslipidémie et de syndrome métabolique, consulte pour des douleurs dans les deux fesses, la région trochantérienne gauche, la face latérale de la cuisse gauche jusqu'au genou et la jambe gauche jusqu'au tiers moyen. La douleur apparaît lorsque le membre inférieur est soulevé avec le genou en extension, mais est soulagée lorsque le genou est fléchi. Quelle est la première suspicion clinique ?", "full_answer": "Dans ce cas, lorsqu'on parle de \"réapparition des symptômes lors de l'élévation du membre inférieur avec le genou en extension\", il s'agit de la manœuvre de Lasègue. Cette manœuvre est positive en cas de lombosciatique, car elle provoque un étirement du nerf sciatique. La bonne réponse est donc le numéro 3. Les autres pathologies n'ont pas de manœuvre de Lasègue positive, donc ces options ne sont pas correctes.", "type": "TRAUMATOLOGIE", "options": {"1": "Arthrite goutteuse de la hanche gauche.", "2": "Arthrose coxo-fémorale gauche.", "3": "Lombalgie irradiée / lombosciatique.", "4": "Claudication due à une sténose du canal.", "5": null}, "correct_option": 3, "explanations": {"1": {"exist": true, "char_ranges": [[309, 414]], "word_ranges": [[51, 68]], "text": "Les autres pathologies n'ont pas de manœuvre de Lasègue positive, donc ces options ne sont pas correctes."}, "2": {"exist": true, "char_ranges": [[309, 414]], "word_ranges": [[51, 68]], "text": "Les autres pathologies n'ont pas de manœuvre de Lasègue positive, donc ces options ne sont pas correctes."}, "3": {"exist": true, "char_ranges": [[0, 308]], "word_ranges": [[0, 51]], "text": "Dans ce cas, lorsqu'on parle de \"réapparition des symptômes lors de l'élévation du membre inférieur avec le genou en extension\", il s'agit de la manœuvre de Lasègue. Cette manœuvre est positive en cas de lombosciatique, car elle provoque un étirement du nerf sciatique. La bonne réponse est donc le numéro 3."}, "4": {"exist": true, "char_ranges": [[309, 414]], "word_ranges": [[51, 68]], "text": "Les autres pathologies n'ont pas de manœuvre de Lasègue positive, donc ces options ne sont pas correctes."}, "5": {"exist": false, "char_ranges": [], "word_ranges": [], "text": ""}}} +{"id": 127, "year": 2012, "question_id_specific": 77, "full_question": "Une patiente diabétique de 32 ans, traitée à l'insuline et dont la glycémie est bien contrôlée, est venue consulter pour des fourmillements dans les deux mains, une sensation de bouchon et une insensibilité thermique d'apparition progressive au cours des deux dernières semaines. Elle n'a pas signalé de troubles visuels, de déficit de force, de maladresse motrice ou d'autres symptômes. L'examen a révélé une anesthésie à la douleur et à la température des deux mains et de la partie distale des avant-bras ; la sensibilité positionnelle et vibratoire a été préservée. Il n'y a pas d'atrophie musculaire ni de déficit de force. Les réflexes musculaires sont normaux et symétriques. Il n'y a pas de dysmétrie, de dysdiadochokinésie ou de tremblement intentionnel. Le reste de l'examen neurologique est strictement normal. Indiquez le diagnostic le plus probable dans ce cas :", "full_answer": "La question brûlante de la section Neurologie de cette année. Le patient est diabétique mais sous insuline avec un bon contrôle (bien que les neuropathies diabétiques ne se développent pas seulement chez les patients avec un mauvais contrôle) avec des symptômes sensoriels limités aux membres supérieurs et de courte durée, sans symptômes moteurs ou à tout autre niveau. À l'examen, il y a une dissociation claire avec une anesthésie thermoalgique et une préservation de l'arthrokinétique et du vibratoire. Les réflexes sont normaux, ni abolis ni exaltés. De plus, le reste de l'examen est strictement normal. Je pense donc que la bonne réponse est 5, c'est-à-dire une lésion syringomyélique, dont la caractéristique initiale est la dissociation sensorielle avec anesthésie pour la thermoalgésie et préservation de la moelle postérieure. La sclérose en plaques et les autres lésions rachidiennes s'accompagneraient d'autres signes d'exploration comme l'exaltation des réflexes, pour le canal carpien l'exploration dépasse le territoire distal du carpe, et pour la réponse 1, s'il existe des formes de polyneuropathies périphériques \"pseudosyringomyéliques\", elles sont le plus souvent d'apparition lombaire. La courte durée d'évolution (2 semaines), le respect des membres inférieurs et la préservation des réflexes musculaires me font écarter cette réponse malgré la distribution \"en gant\".", "type": "NEUROLOGIE ET NEUROCHIRURGIE", "options": {"1": "Neuropathie périphérique sensitivo-symétrique distale d'origine diabétique.", "2": "Lésion par compression de la moelle épinière cervicale.", "3": "Syndrome du canal carpien bilatéral.", "4": "Maladie démyélinisante de type sclérose en plaques.", "5": "Lésion de la moelle épinière cervicale centrale."}, "correct_option": 5, "explanations": {"1": {"exist": true, "char_ranges": [[1075, 1207]], "word_ranges": [[160, 175]], "text": "s'il existe des formes de polyneuropathies périphériques \"pseudosyringomyéliques\", elles sont le plus souvent d'apparition lombaire."}, "2": {"exist": true, "char_ranges": [[838, 977]], "word_ranges": [[127, 144]], "text": "La sclérose en plaques et les autres lésions rachidiennes s'accompagneraient d'autres signes d'exploration comme l'exaltation des réflexes,"}, "3": {"exist": true, "char_ranges": [[978, 1052]], "word_ranges": [[144, 155]], "text": "pour le canal carpien l'exploration dépasse le territoire distal du carpe,"}, "4": {"exist": true, "char_ranges": [[838, 977]], "word_ranges": [[127, 144]], "text": "La sclérose en plaques et les autres lésions rachidiennes s'accompagneraient d'autres signes d'exploration comme l'exaltation des réflexes,"}, "5": {"exist": true, "char_ranges": [[610, 837]], "word_ranges": [[95, 127]], "text": "Je pense donc que la bonne réponse est 5, c'est-à-dire une lésion syringomyélique, dont la caractéristique initiale est la dissociation sensorielle avec anesthésie pour la thermoalgésie et préservation de la moelle postérieure."}}} +{"id": 316, "year": 2016, "question_id_specific": 138, "full_question": "Un homme de 37 ans s'est présenté avec une arthrite des articulations métacarpophalangiennes des deux mains et une pleurite droite. L'examen révèle un érythème malaire bilatéral. Des anticorps antinucléaires positifs (titre 1/640) sont détectés, avec des anticorps anti-ADN natif positifs ; des anticorps anti-Sm négatifs. Quel serait le traitement initial de choix pour ce patient ?", "full_answer": "Il s'agit d'un LED avec des atteintes articulaires et cutanées et une sérosité. Le traitement consiste à administrer des CE à des doses modérées et élevées. Le mycophénolate n'est jamais un traitement en phase aiguë mais en entretien, et son utilité n'a pas été démontrée dans les atteintes articulaires.", "type": "RHEUMATOLOGIE", "options": {"1": "Glucocorticoïdes à haute dose.", "2": "Glucocorticoïdes et mycophénolate.", "3": "Anti-inflammatoires non stéroïdiens et antipaludiques.", "4": "L'affection est susceptible de se résorber d'elle-même et ne nécessite pas de traitement.", "5": null}, "correct_option": 1, "explanations": {"1": {"exist": true, "char_ranges": [[0, 156]], "word_ranges": [[0, 26]], "text": "Il s'agit d'un LED avec des atteintes articulaires et cutanées et une sérosité. Le traitement consiste à administrer des CE à des doses modérées et élevées."}, "2": {"exist": true, "char_ranges": [[157, 304]], "word_ranges": [[26, 49]], "text": "Le mycophénolate n'est jamais un traitement en phase aiguë mais en entretien, et son utilité n'a pas été démontrée dans les atteintes articulaires."}, "3": {"exist": false, "char_ranges": [], "word_ranges": [], "text": ""}, "4": {"exist": false, "char_ranges": [], "word_ranges": [], "text": ""}, "5": {"exist": false, "char_ranges": [], "word_ranges": [], "text": ""}}} +{"id": 153, "year": 2012, "question_id_specific": 74, "full_question": "Une femme de 75 ans consulte pour des lésions violacées sur les mains et le cou, accompagnées d'une faiblesse musculaire progressive depuis 3 mois. Quels tests, parmi ceux indiqués, peuvent être utiles au diagnostic ?", "full_answer": "Compte tenu de ce tableau clinique, une possibilité est la dermatomyosite. Parmi les tests proposés, le dosage de l'aldolase sérique peut être utile. Son élévation est caractéristique de cette maladie. La biopsie doit être musculaire et non sous-cutanée. Les anticorps anti-muscle lisse ne sont pas caractéristiques de cette maladie.", "type": "RHEUMATOLOGIE", "options": {"1": "Détermination de l'aldolase sérique.", "2": "Électroencéphalogramme.", "3": "Biopsie du tissu cellulaire sous-cutané.", "4": "Détermination des anticorps anti-muscle lisse.", "5": "Étude génétique de leurs descendants."}, "correct_option": 1, "explanations": {"1": {"exist": true, "char_ranges": [[0, 201]], "word_ranges": [[0, 30]], "text": "Compte tenu de ce tableau clinique, une possibilité est la dermatomyosite. Parmi les tests proposés, le dosage de l'aldolase sérique peut être utile. Son élévation est caractéristique de cette maladie."}, "2": {"exist": false, "char_ranges": [], "word_ranges": [], "text": ""}, "3": {"exist": true, "char_ranges": [[202, 254]], "word_ranges": [[30, 38]], "text": "La biopsie doit être musculaire et non sous-cutanée."}, "4": {"exist": true, "char_ranges": [[255, 333]], "word_ranges": [[38, 49]], "text": "Les anticorps anti-muscle lisse ne sont pas caractéristiques de cette maladie."}, "5": {"exist": false, "char_ranges": [], "word_ranges": [], "text": ""}}} +{"id": 369, "year": 2016, "question_id_specific": 122, "full_question": "Un homme de 67 ans, ex-fumeur, avec un diagnostic de BPCO sévère (indice multidimensionnel BODE 5, VEMS 38%, indice de masse corporelle 23, indice de dyspnée selon l'échelle mMRC 3, distance parcourue lors du test de marche de 6 minutes 260 m) qui a été admis à l'hôpital 3 fois pour exacerbation de sa BPCO au cours des 7 derniers mois. Il a également des antécédents d'hypertension, de cardiopathie ischémique avec IAM il y a 5 ans et de claudication intermittente. L'examen clinique a révélé une diminution du murmure vésiculaire avec une respiration sifflante expiratoire dans les deux champs pulmonaires et une saturation oxymétrique de 88 %. Laquelle des stratégies thérapeutiques suivantes NE serait PAS recommandée pour ce patient ?", "full_answer": "Chez ces patients, les corticostéroïdes systémiques doivent être utilisés principalement pour les exacerbations aiguës et non pour le contrôle des exacerbations.", "type": "PNEUMOLOGIE ET CHIRURGIE THORACIQUE", "options": {"1": "Ajustement du traitement inhalé avec des bronchodilatateurs de longue durée d'action combinant des anticholinergiques et des bêta-2 adrénergiques avec des glucocorticoïdes inhalés.", "2": "Commencer à administrer des glucocorticoïdes par voie orale pendant 6 mois pour contrôler les exacerbations.", "3": "Vérifier que le patient effectue correctement la technique d'inhalation.", "4": "Mettre en place une oxygénothérapie chronique à domicile.", "5": null}, "correct_option": 2, "explanations": {"1": {"exist": false, "char_ranges": [], "word_ranges": [], "text": ""}, "2": {"exist": true, "char_ranges": [[0, 161]], "word_ranges": [[0, 21]], "text": "Chez ces patients, les corticostéroïdes systémiques doivent être utilisés principalement pour les exacerbations aiguës et non pour le contrôle des exacerbations."}, "3": {"exist": false, "char_ranges": [], "word_ranges": [], "text": ""}, "4": {"exist": false, "char_ranges": [], "word_ranges": [], "text": ""}, "5": {"exist": false, "char_ranges": [], "word_ranges": [], "text": ""}}} +{"id": 413, "year": 2018, "question_id_specific": 68, "full_question": "Une femme de 87 ans, aux antécédents d'hypertension, admise il y a 48 heures dans l'unité coronaire pour un infarctus aigu du myocarde avec un sus-décalage du segment ST en position antérieure. Elle a appelé pour une dyspnée. A l'examen, elle note un souffle systolique avec fremitus, irradiant vers le bord sternal droit, qui n'était pas présent à l'admission. Quelle complication suspectez-vous ?", "full_answer": "Rupture du septum interventriculaire. Les complications mécaniques sont typiques des infarctus chez les femmes âgées. En outre, un souffle qui n'était pas présent indique une circulation sanguine anormale : les complications les plus courantes sont l'insuffisance mitrale, qui se manifeste par un souffle musical ou sonore, et la CIV, qui se manifeste par un souffle irradiant vers le bord droit (pensez à la direction de la circulation sanguine).", "type": "CARDIOLOGIE ET CHIRURGIE CARDIOVASCULAIRE", "options": {"1": "Insuffisance cardiaque due à une nécrose importante.", "2": "Anévrisme antérieur.", "3": "Rupture de la paroi libre du ventricule gauche.", "4": "Rupture du septum interventriculaire.", "5": null}, "correct_option": 4, "explanations": {"1": {"exist": false, "char_ranges": [], "word_ranges": [], "text": ""}, "2": {"exist": false, "char_ranges": [], "word_ranges": [], "text": ""}, "3": {"exist": false, "char_ranges": [], "word_ranges": [], "text": ""}, "4": {"exist": true, "char_ranges": [[207, 447]], "word_ranges": [[29, 68]], "text": "les complications les plus courantes sont l'insuffisance mitrale, qui se manifeste par un souffle musical ou sonore, et la CIV, qui se manifeste par un souffle irradiant vers le bord droit (pensez à la direction de la circulation sanguine)."}, "5": {"exist": false, "char_ranges": [], "word_ranges": [], "text": ""}}} +{"id": 314, "year": 2016, "question_id_specific": 178, "full_question": "Un adénocarcinome pancréatique de stade IV a été diagnostiqué il y a 8 mois chez un patient de 66 ans. Il est traité avec de la morphine à libération retardée 200 mg/12 heures par voie orale, de la paraffine et du lactulose. Depuis 15 jours, il signale des paresthésies et des douleurs lancinantes occasionnelles dans la région lombaire et périombilicale droite qui ne disparaissent pas avec la morphine de secours. L'examen neurologique est normal. Indiquez la prise en charge la plus appropriée :", "full_answer": "Nous avons devant nous un patient présentant des douleurs neuropathiques qui, compte tenu de l'origine de la tumeur, sont très probablement dues à une atteinte du plexus cœliaque. Il est important d'établir un diagnostic radiologique en vue d'un éventuel traitement local, tel qu'un traitement neurolytique ou un blocage du plexus. Cependant, nous ne devons pas laisser le patient sans traitement qui pourrait atténuer, mais plutôt résoudre la situation actuelle. Le patient prend une analgésie OMS step 3, qui est clairement insuffisante, et nécessitera donc des adjuvants ; bien que les opioïdes pour les douleurs neuropathiques ne soient pas très efficaces, nous ne réduirons pas la dose mais les associerons à des adjuvants. On prendra en charge le patient avec de la gabapentine et des corticoïdes (vraie réponse 4) ; on ne diminuera pas la dose de morphine (fausse réponse 3), et en attendant on demandera un scanner (ou une IRM abdominale) pour évaluer ultérieurement le bloc plexique (réponses 1 et 2 pour la prise en charge future).", "type": "ONCOLOGIE", "options": {"1": "Effectuer une tomodensitométrie et rechercher une compression nerveuse, car il s'agit d'une douleur neuropathique.", "2": "Envisager un traitement neurolytique car la douleur neuropathique est difficile à contrôler.", "3": "Diminuer la dose de morphine car elle est inefficace dans ce type de douleur.", "4": "Administrer de l'amitriptyline ou de la gabapentine, de la dexaméthasone et augmenter la dose de morphine.", "5": null}, "correct_option": 4, "explanations": {"1": {"exist": true, "char_ranges": [[886, 1041]], "word_ranges": [[141, 167]], "text": "en attendant on demandera un scanner (ou une IRM abdominale) pour évaluer ultérieurement le bloc plexique (réponses 1 et 2 pour la prise en charge future)."}, "2": {"exist": true, "char_ranges": [[886, 1041]], "word_ranges": [[141, 167]], "text": "en attendant on demandera un scanner (ou une IRM abdominale) pour évaluer ultérieurement le bloc plexique (réponses 1 et 2 pour la prise en charge future)."}, "3": {"exist": true, "char_ranges": [[823, 882]], "word_ranges": [[129, 140]], "text": "on ne diminuera pas la dose de morphine (fausse réponse 3),"}, "4": {"exist": true, "char_ranges": [[576, 822]], "word_ranges": [[87, 129]], "text": "bien que les opioïdes pour les douleurs neuropathiques ne soient pas très efficaces, nous ne réduirons pas la dose mais les associerons à des adjuvants. On prendra en charge le patient avec de la gabapentine et des corticoïdes (vraie réponse 4) ;"}, "5": {"exist": false, "char_ranges": [], "word_ranges": [], "text": ""}}} +{"id": 263, "year": 2014, "question_id_specific": 37, "full_question": "Juan a 60 ans, il fume deux paquets de cigarettes par jour depuis des années et signale une toux persistante depuis six mois. Il remarque que sa paupière gauche est plus tombante et que la pupille de son œil gauche est plus petite. Juan signale que la partie médiane de sa main gauche est engourdie et qu'elle a moins de force. Le médecin vérifie la ptose des paupières et le myosis gauche ; il s'assure qu'il peut fermer les deux paupières de manière symétrique et que les deux pupilles réagissent correctement à la lumière. Il vérifie également qu'il n'y a pas de transpiration sur l'hémiface gauche, qu'il ressent moins de pincement sur la face interne de la main gauche et qu'il a moins de force dans la préhension de la main gauche. En ce qui concerne les symptômes oculaires, où se situe la lésion ?", "full_answer": "La question serait plutôt d'ordre neurologique. Il s'agit d'un syndrome de Claude Bernard Horner et la réponse correcte est probablement 1. Mais pour des raisons de sécurité, il est préférable de faire vérifier la situation par un neurologue.", "type": "PNEUMOLOGIE", "options": {"1": "Fibres sympathiques, s'étendant à un certain niveau de l'hypothalamus à la colonne de Clark interindio-latérale de la moelle dorsale.", "2": "Le nerf moteur oculaire commun gauche dans le mésencéphale.", "3": "Noyau d'Edinger-Westphal, au-dessus du noyau du nerf moteur oculaire commun gauche.", "4": "Fibres parasympathiques, s'étendant à un certain niveau du noyau d'Edinger-Westphal au muscle constricteur pupillaire gauche.", "5": "Le muscle tarsien exclusivement."}, "correct_option": 1, "explanations": {"1": {"exist": true, "char_ranges": [[48, 139]], "word_ranges": [[6, 21]], "text": "Il s'agit d'un syndrome de Claude Bernard Horner et la réponse correcte est probablement 1."}, "2": {"exist": false, "char_ranges": [], "word_ranges": [], "text": ""}, "3": {"exist": false, "char_ranges": [], "word_ranges": [], "text": ""}, "4": {"exist": false, "char_ranges": [], "word_ranges": [], "text": ""}, "5": {"exist": false, "char_ranges": [], "word_ranges": [], "text": ""}}} +{"id": 344, "year": 2016, "question_id_specific": 158, "full_question": "Une femme de 24 ans, primigeste, fait une fausse couche spontanée à 7 semaines de grossesse. L'étude anatomopathologique des restes avortés indique une maladie molaire. Nous devons l'informer que :", "full_answer": "Elle ne doit pas tomber enceinte tant qu'elle n'a pas été suivie régulièrement et que son taux de bêta-HCG n'est pas négatif depuis un an. Les patientes seront suivies chaque semaine par dosage de l'hCG jusqu'à ce que le taux devienne indétectable, et ce pendant trois périodes consécutives. Ensuite, le suivi sera mensuel pendant six mois, puis tous les deux mois pendant encore six mois avant une nouvelle grossesse.", "type": "GYNÉCOLOGIE ET OBSTÉTRIQUE", "options": {"1": "Le risque d'une nouvelle gestation molaire lors d'une prochaine grossesse est de 50 %.", "2": "Vous ne devez pas tomber enceinte avant d'avoir subi des examens réguliers et d'avoir passé un an avec des taux de BHCG négatifs.", "3": "Aucune surveillance supplémentaire n'est nécessaire si l'évacuation du tissu trophoblastique a été complète.", "4": "Des contrôles réguliers sont nécessaires car dans 40 % des cas, une néoplasie trophoblastique gestationnelle se développe.", "5": null}, "correct_option": 2, "explanations": {"1": {"exist": false, "char_ranges": [], "word_ranges": [], "text": ""}, "2": {"exist": true, "char_ranges": [[0, 418]], "word_ranges": [[0, 69]], "text": "Elle ne doit pas tomber enceinte tant qu'elle n'a pas été suivie régulièrement et que son taux de bêta-HCG n'est pas négatif depuis un an. Les patientes seront suivies chaque semaine par dosage de l'hCG jusqu'à ce que le taux devienne indétectable, et ce pendant trois périodes consécutives. Ensuite, le suivi sera mensuel pendant six mois, puis tous les deux mois pendant encore six mois avant une nouvelle grossesse."}, "3": {"exist": false, "char_ranges": [], "word_ranges": [], "text": ""}, "4": {"exist": false, "char_ranges": [], "word_ranges": [], "text": ""}, "5": {"exist": false, "char_ranges": [], "word_ranges": [], "text": ""}}} +{"id": 43, "year": 2011, "question_id_specific": 151, "full_question": "Une fillette de 10 ans vient pour un bilan de santé. L'examen physique est normal, avec un poids et une taille dans le 50e percentile et une tension artérielle de 109/65. Une analyse d'urine de routine montre une gravité spécifique de 1035 pH6 sang 2+ sans protéine. Le sédiment urinaire présente 5 à 10 globules rouges par champ. Quel serait le traitement le plus approprié ?", "full_answer": "La bonne réponse est 4. Ce qui n'est pas très clair pour moi, ce sont les \"analyses d'urine de routine\" parce qu'elles ne sont généralement pas effectuées chez un enfant en bonne santé asymptomatique. La microhématurie sans symptômes associés peut être une constatation sans signification pathologique et doit être confirmée par un nouveau sédiment à une date ultérieure (bien que je ne sache pas si c'est après 15 jours ou plus tard).", "type": "PÉDIATRIE", "options": {"1": "Déterminer la créatinine et l'azote sanguin.", "2": "Orienter la jeune fille vers une cystoscopie.", "3": "Déterminer les anticorps antinucléaires et le complément.", "4": "Répéter le sédiment urinaire dans 15 jours.", "5": "Réaliser un scanner abdominal."}, "correct_option": 4, "explanations": {"1": {"exist": false, "char_ranges": [], "word_ranges": [], "text": ""}, "2": {"exist": false, "char_ranges": [], "word_ranges": [], "text": ""}, "3": {"exist": false, "char_ranges": [], "word_ranges": [], "text": ""}, "4": {"exist": true, "char_ranges": [[201, 435]], "word_ranges": [[34, 72]], "text": "La microhématurie sans symptômes associés peut être une constatation sans signification pathologique et doit être confirmée par un nouveau sédiment à une date ultérieure (bien que je ne sache pas si c'est après 15 jours ou plus tard)."}, "5": {"exist": false, "char_ranges": [], "word_ranges": [], "text": ""}}} +{"id": 616, "year": 2022, "question_id_specific": 119, "full_question": "Femme de 13 ans, sans antécédents pertinents, avec ménarche depuis 3 mois, suivie depuis l'âge de 10 ans pour une scoliose idiopathique qui s'est aggravée. À l'examen physique, elle présentait une bosse de 7 degrés au test d'Adams et, à la scoliographie, une courbe thoraco-lombaire T4-L1 de 35 degrés de Cobb et une courbe de Risser de 0 :", "full_answer": "Orthèse. Angle de Cobb entre 25º et 45º. Squelette immature (Risser 0).", "type": "TRAUMATOLOGIE", "options": {"1": "Recommander la natation et réexaminer la situation dans trois mois.", "2": "Prescrire une orthèse.", "3": "Consulter un physiothérapeute pour l'élasticité de la colonne vertébrale.", "4": "Révision dans 6 mois avec une nouvelle radiographie.", "5": null}, "correct_option": 2, "explanations": {"1": {"exist": false, "char_ranges": [], "word_ranges": [], "text": ""}, "2": {"exist": true, "char_ranges": [[0, 71]], "word_ranges": [[0, 12]], "text": "Orthèse. Angle de Cobb entre 25º et 45º. Squelette immature (Risser 0)."}, "3": {"exist": false, "char_ranges": [], "word_ranges": [], "text": ""}, "4": {"exist": false, "char_ranges": [], "word_ranges": [], "text": ""}, "5": {"exist": false, "char_ranges": [], "word_ranges": [], "text": ""}}} +{"id": 489, "year": 2020, "question_id_specific": 105, "full_question": "Patient de 67 ans qui, au cours des 6 derniers mois, a présenté une lymphocytose progressive lors de deux analyses de routine. Dans la dernière, hémoglobine 15,4 g/dl ; leucocytes 18,5 x103/μL avec 82% de lymphocytes matures qui, par cytométrie de flux, expriment les antigènes CD5/CD19/CD23 et plaquettes 240 x103/μL. Quelle est, à votre avis, l'approche correcte ?", "full_answer": "Le patient a déjà été diagnostiqué avec une leucémie lymphoïde chronique B (B-CLLL), présente une lymphocytose depuis plus de 3 mois et a un phénotype compatible avec celle-ci par cytométrie de flux. Il faut donc d'abord connaître les facteurs de risque qui indiqueront la priorité avec laquelle il faudra commencer le traitement, leur intensité et le pronostic, dont dépend beaucoup la présence de mutations TP53 ou de délétions 17p. L'aspiration ou la biopsie de la moelle osseuse n'est indiquée que s'il existe d'autres cytopénies d'accompagnement qui ne sont pas justifiées par la B-CLL elle-même, la PET-CT n'est indiquée que si l'on suspecte un syndrome de Ritcher et qu'il n'y a pas suffisamment de données pour le suspecter, qu'il n'y a pas de symptômes B et qu'il n'est pas fait mention d'une élévation des LDH.", "type": "HÉMATOLOGIE", "options": {"1": "Étude des mutations TP53 pour établir un pronostic.", "2": "Aspiration/biopsie osseuse pour confirmer le diagnostic.", "3": "PET/CT pour établir l'approche thérapeutique.", "4": "Nouveau contrôle clinique et analytique en 6 mois.", "5": null}, "correct_option": 4, "explanations": {"1": {"exist": false, "char_ranges": [], "word_ranges": [], "text": ""}, "2": {"exist": true, "char_ranges": [[435, 601]], "word_ranges": [[69, 94]], "text": "L'aspiration ou la biopsie de la moelle osseuse n'est indiquée que s'il existe d'autres cytopénies d'accompagnement qui ne sont pas justifiées par la B-CLL elle-même,"}, "3": {"exist": true, "char_ranges": [[602, 820]], "word_ranges": [[94, 134]], "text": "la PET-CT n'est indiquée que si l'on suspecte un syndrome de Ritcher et qu'il n'y a pas suffisamment de données pour le suspecter, qu'il n'y a pas de symptômes B et qu'il n'est pas fait mention d'une élévation des LDH."}, "4": {"exist": true, "char_ranges": [[0, 434]], "word_ranges": [[0, 69]], "text": "Le patient a déjà été diagnostiqué avec une leucémie lymphoïde chronique B (B-CLLL), présente une lymphocytose depuis plus de 3 mois et a un phénotype compatible avec celle-ci par cytométrie de flux. Il faut donc d'abord connaître les facteurs de risque qui indiqueront la priorité avec laquelle il faudra commencer le traitement, leur intensité et le pronostic, dont dépend beaucoup la présence de mutations TP53 ou de délétions 17p."}, "5": {"exist": false, "char_ranges": [], "word_ranges": [], "text": ""}}} +{"id": 534, "year": 2021, "question_id_specific": 119, "full_question": "Une femme de 45 ans, ayant des antécédents de reflux gastro-œsophagien, présente depuis un an des épisodes de pâleur de certains doigts et orteils lorsqu'elle est exposée au froid. On lui a récemment prescrit de la prednisone à raison de 20 mg/jour pour des douleurs articulaires et une induration de la peau des mains et des bras. Au cours des dernières 48 heures, elle a présenté un malaise général et des céphalées sévères qui l'ont amenée à se rendre au service des urgences. L'examen n'a révélé qu'une tachycardie rythmique à 100 bpm, sans focalité neurologique. La tension artérielle est de 200/110 mmHg. L'analyse de sang ne révèle qu'un taux de créatinine de 2,5 mg/dl. Indiquez lequel des auto-anticorps suivants est le plus étroitement lié au processus décrit :", "full_answer": "Il s'agit d'un cas clinique clair de sclérose systémique (Raynaud, atteinte œsophagienne, induration cutanée) avec crise rénale. Outre les anticorps anti-protéinase 3, qui ne sont pas liés à la sclérodermie (réponse incorrecte 3), les anticorps anti ARN polymérase III, qui apparaissent chez des patients plus jeunes avec des crises rénales, sont les plus probables.", "type": "RHEUMATOLOGIE", "options": {"1": "Anticorps anti-ARN polymérase III.", "2": "Anticorps anti-centromères.", "3": "Anticorps anti-protéinase 3.", "4": "Anticorps anti-PM-Scl.", "5": null}, "correct_option": 1, "explanations": {"1": {"exist": true, "char_ranges": [[0, 366]], "word_ranges": [[0, 54]], "text": "Il s'agit d'un cas clinique clair de sclérose systémique (Raynaud, atteinte œsophagienne, induration cutanée) avec crise rénale. Outre les anticorps anti-protéinase 3, qui ne sont pas liés à la sclérodermie (réponse incorrecte 3), les anticorps anti ARN polymérase III, qui apparaissent chez des patients plus jeunes avec des crises rénales, sont les plus probables."}, "2": {"exist": false, "char_ranges": [], "word_ranges": [], "text": ""}, "3": {"exist": true, "char_ranges": [[135, 230]], "word_ranges": [[18, 33]], "text": "les anticorps anti-protéinase 3, qui ne sont pas liés à la sclérodermie (réponse incorrecte 3),"}, "4": {"exist": false, "char_ranges": [], "word_ranges": [], "text": ""}, "5": {"exist": false, "char_ranges": [], "word_ranges": [], "text": ""}}} +{"id": 273, "year": 2016, "question_id_specific": 71, "full_question": "Un jeune homme de 18 ans, ayant des antécédents d'asthme, d'allergie aux pollens, aux acariens et aux poils de chat, se présente au service des urgences en se plaignant d'une sensation de blocage alimentaire rétrosternal avec une incapacité pratique à avaler sa propre salive. Il rapporte des épisodes similaires à d'autres occasions qui se sont spontanément résorbés en quelques minutes. Lequel des diagnostics suivants est le plus probable ?", "full_answer": "Il s'agit d'une dysphagie intermittente. L'œsophage de Barrett n'implique pas nécessairement un rétrécissement peptique, mais si c'est le cas, il est progressif. L'œsophagite infectieuse est plus fréquente chez les patients immunodéprimés. Il reste donc l'anneau œsophagien distal et l'œsophagite à éosinophiles ; les deux sont possibles, mais l'accent mis sur le fardeau atopique du patient indique la probabilité de la seconde.", "type": "SYSTÈME DIGESTIF", "options": {"1": "Œsophage de Barrett.", "2": "Anneau œsophagien distal (Schatzki).", "3": "Oesophagite infectieuse.", "4": "Oesophagite à éosinophiles.", "5": null}, "correct_option": 4, "explanations": {"1": {"exist": true, "char_ranges": [[41, 161]], "word_ranges": [[5, 22]], "text": "L'œsophage de Barrett n'implique pas nécessairement un rétrécissement peptique, mais si c'est le cas, il est progressif."}, "2": {"exist": true, "char_ranges": [[240, 429]], "word_ranges": [[31, 61]], "text": "Il reste donc l'anneau œsophagien distal et l'œsophagite à éosinophiles ; les deux sont possibles, mais l'accent mis sur le fardeau atopique du patient indique la probabilité de la seconde."}, "3": {"exist": true, "char_ranges": [[162, 239]], "word_ranges": [[22, 31]], "text": "L'œsophagite infectieuse est plus fréquente chez les patients immunodéprimés."}, "4": {"exist": true, "char_ranges": [[240, 429]], "word_ranges": [[31, 61]], "text": "Il reste donc l'anneau œsophagien distal et l'œsophagite à éosinophiles ; les deux sont possibles, mais l'accent mis sur le fardeau atopique du patient indique la probabilité de la seconde."}, "5": {"exist": false, "char_ranges": [], "word_ranges": [], "text": ""}}} +{"id": 56, "year": 2011, "question_id_specific": 76, "full_question": "Jeune fille de 14 ans qui consulte pour une diminution de sa croissance depuis 2 ou 3 ans, alors qu'elle était normale auparavant (elle fournit des données) et que les autres filles de son âge ont un développement physique et sexuel plus important. Dernièrement, elle a eu des maux de tête et des problèmes visuels qu'elle remarque en classe et lorsqu'elle étudie. Elle n'a pas eu de ménarche, ni de polydipsie ou de polyurie. Les parents sont de taille normale. Examen : petite taille à -2,1 écarts-types, proportions corporelles normales, peu de poils pubiens et de développement mammaire. La campimétrie montre une hémianopsie partielle temporale gauche. Âge osseux : retard de 2 ans. Les tests de laboratoire généraux sont normaux. Les gonadotrophines (FSH et LH) et l'œstradiol sont faibles. Quelle est, selon vous, la réponse la plus appropriée ?", "full_answer": "La réponse 2 est fausse (syndrome de Turner : oestradiol bas et gonadotrophines élevées), les tumeurs cérébrales affectant l'hypothalamus-hypophyse ne donnent pas de gonadotrophines basses (5 faux), et il semble évident que les défauts de réfraction ne donnent pas d'hémianopsie (3 et 4 faux).", "type": "ENDOCRINOLOGIE", "options": {"1": "Le ralentissement de la croissance et du développement sexuel, le retard de l'âge osseux, les céphalées et les troubles visuels suggèrent des déficits hormonaux et une atteinte du chiasma optique.", "2": "En tant que fille en âge de puberté, son retard de croissance et son retard sexuel sont très probablement dus au syndrome de Turner.", "3": "Elle ne doit pas avoir de tumeur hypothalamique en raison de l'absence de polyurie et de polydipsie. Il a probablement un retard constitutionnel et son problème visuel est un problème de réfraction.", "4": "Un déficit en hormone de croissance peut expliquer le retard de développement et le faible taux d'œstradiol. Déterminez si elle a besoin de lunettes en raison de maux de tête et de troubles de la vue.", "5": "Elle pourrait avoir un craniopharyngiome, mais il serait étrange qu'elle n'ait pas présenté de symptômes auparavant. En outre, cela ne justifierait pas un taux de gonadotrophines et d'œstradiol faible."}, "correct_option": 1, "explanations": {"1": {"exist": false, "char_ranges": [], "word_ranges": [], "text": ""}, "2": {"exist": true, "char_ranges": [[0, 89]], "word_ranges": [[0, 14]], "text": "La réponse 2 est fausse (syndrome de Turner : oestradiol bas et gonadotrophines élevées),"}, "3": {"exist": true, "char_ranges": [[224, 293]], "word_ranges": [[32, 44]], "text": "les défauts de réfraction ne donnent pas d'hémianopsie (3 et 4 faux)."}, "4": {"exist": true, "char_ranges": [[224, 293]], "word_ranges": [[32, 44]], "text": "les défauts de réfraction ne donnent pas d'hémianopsie (3 et 4 faux)."}, "5": {"exist": true, "char_ranges": [[90, 198]], "word_ranges": [[14, 27]], "text": "les tumeurs cérébrales affectant l'hypothalamus-hypophyse ne donnent pas de gonadotrophines basses (5 faux),"}}} +{"id": 409, "year": 2018, "question_id_specific": 191, "full_question": "Il s'est rendu au domicile d'un patient oncologique dont la douleur était mal contrôlée depuis peu. L'examen physique n'a montré aucun signe de progression tumorale et n'a révélé aucune donnée d'intérêt connue antérieurement. Dans l'anamnèse, l'aidant principal déclare que le patient souffre 8 heures après avoir reçu la dose de base prescrite de morphine toutes les 12 heures. Cette situation clinique est appelée :", "full_answer": "La bonne réponse est 1. L'énoncé définit parfaitement un phénomène de tolérance où le patient s'est \"habitué\" à la dose et a besoin d'une dose plus importante pour obtenir le même effet analgésique.", "type": "SOINS PRIMAIRES", "options": {"1": "Tolérance", "2": "Hyperalgésie", "3": "Dépendance", "4": "Inefficacité", "5": null}, "correct_option": 1, "explanations": {"1": {"exist": true, "char_ranges": [[24, 198]], "word_ranges": [[5, 33]], "text": "L'énoncé définit parfaitement un phénomène de tolérance où le patient s'est \"habitué\" à la dose et a besoin d'une dose plus importante pour obtenir le même effet analgésique."}, "2": {"exist": false, "char_ranges": [], "word_ranges": [], "text": ""}, "3": {"exist": false, "char_ranges": [], "word_ranges": [], "text": ""}, "4": {"exist": false, "char_ranges": [], "word_ranges": [], "text": ""}, "5": {"exist": false, "char_ranges": [], "word_ranges": [], "text": ""}}} +{"id": 179, "year": 2013, "question_id_specific": 98, "full_question": "Une femme de 33 ans consulte pour des épistaxis répétées, des pétéchies et des ecchymoses. Les examens de laboratoire révèlent une thrombocytopénie avec une numération plaquettaire de 4000 plaquettes/microlitre. Le diagnostic initial présumé est un purpura thrombocytopénique immunitaire (PTI) chronique. Laquelle des affirmations suivantes est FAUSSE en ce qui concerne le diagnostic du PTI ?", "full_answer": "En cas de thrombopénie isolée, l'examen physique ne doit pas être négligé ; ainsi, la présence d'adénopathies oblige à écarter la présence d'un lymphome, par exemple. Le point 1 est correct. Une biopsie-aspiration de moelle osseuse montre que le nombre de mégacaryocytes est normal voire augmenté, la faute n'est pas dans la moelle mais dans le sang périphérique où les plaquettes sont détruites. Donc... La fausse réponse est 2.", "type": "HÉMATOLOGIE", "options": {"1": "La présence d'une lymphadénopathie ou d'une splénomégalie à l'examen physique suggère un autre diagnostic de PTI.", "2": "L'analyse de la moelle osseuse montre une diminution du nombre de mégacaryocytes sans autre altération.", "3": "La formule sanguine complète montre une thrombocytopénie isolée avec souvent de grandes plaquettes, sans anémie, sauf en cas d'hémorragie majeure ou d'hémolyse auto-immune associée (syndrome d'Evans).", "4": "Le diagnostic de PTI est établi par l'exclusion d'autres processus provoquant une thrombocytopénie.", "5": "La détermination des anticorps anti-plaque ne permet pas d'établir le diagnostic avec précision."}, "correct_option": 2, "explanations": {"1": {"exist": true, "char_ranges": [[0, 166]], "word_ranges": [[0, 26]], "text": "En cas de thrombopénie isolée, l'examen physique ne doit pas être négligé ; ainsi, la présence d'adénopathies oblige à écarter la présence d'un lymphome, par exemple."}, "2": {"exist": true, "char_ranges": [[191, 396]], "word_ranges": [[31, 63]], "text": "Une biopsie-aspiration de moelle osseuse montre que le nombre de mégacaryocytes est normal voire augmenté, la faute n'est pas dans la moelle mais dans le sang périphérique où les plaquettes sont détruites."}, "3": {"exist": false, "char_ranges": [], "word_ranges": [], "text": ""}, "4": {"exist": false, "char_ranges": [], "word_ranges": [], "text": ""}, "5": {"exist": false, "char_ranges": [], "word_ranges": [], "text": ""}}} +{"id": 353, "year": 2016, "question_id_specific": 161, "full_question": "Une femme de 27 ans a été adressée au service de gynécologie pour évaluation, signalant une dyspareunie depuis 8 mois, ainsi qu'une dyschésie et une rectorragie occasionnelle coïncidant avec les menstruations depuis 3-4 mois. Elle signale également une dysménorrhée depuis des années, qu'elle contrôle bien avec de l'ibuprofène. Elle essaie de tomber enceinte depuis 16 mois, mais n'y est pas encore parvenue. Lors de l'examen gynécologique, seule une douleur est notée à la pression du fornix vaginal postérieur. Quel examen vous permettrait, selon vous, de poser un diagnostic de certitude sur sa pathologie ?", "full_answer": "Laparoscopie diagnostique. Il s'agit d'un cas typique d'endométriose profonde, avec des implants dans différentes parties du bassin. Le diagnostic définitif sera donné par l'anatomie pathologique, nous devons donc faire une laparoscopie et l'envoyer à l'AP.", "type": "GYNÉCOLOGIE ET OBSTÉTRIQUE", "options": {"1": "Échographie transvaginale.", "2": "La laparoscopie diagnostique.", "3": "Imagerie par résonance magnétique.", "4": "Coloscopie.", "5": null}, "correct_option": 2, "explanations": {"1": {"exist": false, "char_ranges": [], "word_ranges": [], "text": ""}, "2": {"exist": true, "char_ranges": [[27, 257]], "word_ranges": [[2, 35]], "text": "Il s'agit d'un cas typique d'endométriose profonde, avec des implants dans différentes parties du bassin. Le diagnostic définitif sera donné par l'anatomie pathologique, nous devons donc faire une laparoscopie et l'envoyer à l'AP."}, "3": {"exist": false, "char_ranges": [], "word_ranges": [], "text": ""}, "4": {"exist": false, "char_ranges": [], "word_ranges": [], "text": ""}, "5": {"exist": false, "char_ranges": [], "word_ranges": [], "text": ""}}} +{"id": 438, "year": 2018, "question_id_specific": 119, "full_question": "Vétérinaire âgée de 38 ans, elle était chargée de surveiller les animaux sauvages et d'aider à l'accouchement du bétail domestique. Elle a commencé par présenter une forte fièvre accompagnée de frissons, de céphalées, de myalgies et d'une toux non productive, qu'elle a interprétée comme un syndrome grippal. Elle a présenté des douleurs thoraciques. La radiographie du thorax a montré des infiltrats pulmonaires bilatéraux dans les champs inférieurs. Un test sérologique a été effectué et a révélé des titres élevés d'anticorps dirigés contre les antigènes de phase II. Laquelle des affirmations suivantes est VRAIE ?", "full_answer": "Parmi les maladies possibles, je citerais la brucellose, typique des bovins (lait, avortement, sécrétions génitales...). Elle se manifeste par des symptômes grippaux (céphalées, arthralgies, myalgies, fièvre...) et d'autres symptômes en fonction de la voie d'entrée. Dans notre cas, lors de la mise bas d'animaux, l'entrée se fait par voie respiratoire, les infiltrats pulmonaires et les douleurs thoraciques sont donc dus à la bactérie. Le traitement varie en fonction de la maladie et la mortalité est très faible, voire nulle. La morbidité et la maladie chez les femmes enceintes sont plus préoccupantes. Important : la brucellose est très fréquente chez les travailleurs du secteur de l'élevage (vétérinaires, agriculteurs, abattoirs...) si vous lisez quelque chose comme \"assiste à la naissance d'animaux d'élevage\", \"travaille dans une ferme\", \"est vétérinaire\", soupçonnez la brucellose.", "type": "ÉPIDÉMIOLOGIE ET MÉDECINE PRÉVENTIVE", "options": {"1": "Elle est transmise par les tiques.", "2": "La doxycycline et l'hydroxychloroquine sont efficaces pour traiter les formes aiguës de la maladie.", "3": "Dans la forme aiguë, on observe également une élévation des anticorps dirigés contre les antigènes de la phase l.", "4": "La mortalité dans les formes aiguës est presque inexistante.", "5": null}, "correct_option": 4, "explanations": {"1": {"exist": false, "char_ranges": [], "word_ranges": [], "text": ""}, "2": {"exist": false, "char_ranges": [], "word_ranges": [], "text": ""}, "3": {"exist": false, "char_ranges": [], "word_ranges": [], "text": ""}, "4": {"exist": true, "char_ranges": [[267, 529]], "word_ranges": [[35, 79]], "text": "Dans notre cas, lors de la mise bas d'animaux, l'entrée se fait par voie respiratoire, les infiltrats pulmonaires et les douleurs thoraciques sont donc dus à la bactérie. Le traitement varie en fonction de la maladie et la mortalité est très faible, voire nulle."}, "5": {"exist": false, "char_ranges": [], "word_ranges": [], "text": ""}}} +{"id": 579, "year": 2022, "question_id_specific": 108, "full_question": "Un homme de 49 ans a été admis aux soins intensifs pour un traumatisme crânien consécutif à un accident de travail. À l'examen physique, il n'a pas ouvert les yeux, n'a pas émis de sons avant d'être intubé et a montré une extension des membres lors d'un stimulus nociceptif. Un capteur de pression intracrânienne est placé et une craniotomie décompressive doit être pratiquée en raison d'une hémorragie intraparenchymateuse. Laquelle des affirmations suivantes est correcte ?", "full_answer": "L'hypercapnie et l'acidose provoquent une vasodilatation entraînant une augmentation du débit sanguin cérébral. D'après l'examen décrit, le Glasgow est à 4 (O1V1M2). La pression de perfusion cérébrale est la pression artérielle moyenne moins la pression intracrânienne. L'œdème cérébral dans lequel se produit un œdème cellulaire dû à l'introduction de liquide extracellulaire dans le compartiment intracellulaire est cytotoxique.", "type": "NEUROLOGIE", "options": {"1": "À son arrivée, il est dans le coma sur l'échelle de Glasgow (7).", "2": "La pression de perfusion cérébrale correspond à la pression artérielle moyenne plus la pression intracrânienne.", "3": "L'œdème cérébral vasogénique est causé par l'œdème des cellules, la rupture de la membrane et la mort des cellules.", "4": "Le débit vasculaire cérébral augmente avec l'hypercapnie et l'acidose.", "5": null}, "correct_option": 4, "explanations": {"1": {"exist": true, "char_ranges": [[112, 165]], "word_ranges": [[13, 22]], "text": "D'après l'examen décrit, le Glasgow est à 4 (O1V1M2)."}, "2": {"exist": true, "char_ranges": [[166, 269]], "word_ranges": [[22, 36]], "text": "La pression de perfusion cérébrale est la pression artérielle moyenne moins la pression intracrânienne."}, "3": {"exist": true, "char_ranges": [[270, 430]], "word_ranges": [[36, 57]], "text": "L'œdème cérébral dans lequel se produit un œdème cellulaire dû à l'introduction de liquide extracellulaire dans le compartiment intracellulaire est cytotoxique."}, "4": {"exist": true, "char_ranges": [[0, 111]], "word_ranges": [[0, 13]], "text": "L'hypercapnie et l'acidose provoquent une vasodilatation entraînant une augmentation du débit sanguin cérébral."}, "5": {"exist": false, "char_ranges": [], "word_ranges": [], "text": ""}}} +{"id": 212, "year": 2014, "question_id_specific": 88, "full_question": "Un homme de 45 ans a subi une vagotomie tronculaire et une antrectomie avec reconstruction de Billroth II pour un ulcère gastroduodénal chronique avec sténose pyloro-duodénale. Six semaines après l'intervention chirurgicale, il a signalé que peu de temps après l'ingestion (moins d'une demi-heure), il avait des nausées, une asthénie et des sueurs, des vertiges et des crampes abdominales généralement accompagnées d'un écoulement diarrhéique. Laquelle des approches suivantes est la plus appropriée pour la prise en charge initiale de cet homme ?", "full_answer": "Les réponses 1, 2 et 5 sont des traitements appropriés pour le syndrome de dumping ou la postgastrectomie, mais la question porte sur la prise en charge initiale, de sorte que la réponse la plus appropriée semble être la réponse 2.", "type": "CHIRURGIE GÉNÉRALE", "options": {"1": "Appliquer un traitement avec un inhibiteur de la somatostatine (octréotide).", "2": "Suivre des mesures diététiques spécifiques.", "3": "Effectuer un traitement d'essai avec une benzodiazépine.", "4": "Recherche d'une tumeur neuroendocrine probable (par exemple, carcinoïde).", "5": "Indiquer le traitement chirurgical pour réaliser une gastrojéjunostomie Roux-en-Y antipéristaltique."}, "correct_option": 2, "explanations": {"1": {"exist": true, "char_ranges": [[0, 231]], "word_ranges": [[0, 41]], "text": "Les réponses 1, 2 et 5 sont des traitements appropriés pour le syndrome de dumping ou la postgastrectomie, mais la question porte sur la prise en charge initiale, de sorte que la réponse la plus appropriée semble être la réponse 2."}, "2": {"exist": true, "char_ranges": [[0, 231]], "word_ranges": [[0, 41]], "text": "Les réponses 1, 2 et 5 sont des traitements appropriés pour le syndrome de dumping ou la postgastrectomie, mais la question porte sur la prise en charge initiale, de sorte que la réponse la plus appropriée semble être la réponse 2."}, "3": {"exist": false, "char_ranges": [], "word_ranges": [], "text": ""}, "4": {"exist": false, "char_ranges": [], "word_ranges": [], "text": ""}, "5": {"exist": true, "char_ranges": [[0, 231]], "word_ranges": [[0, 41]], "text": "Les réponses 1, 2 et 5 sont des traitements appropriés pour le syndrome de dumping ou la postgastrectomie, mais la question porte sur la prise en charge initiale, de sorte que la réponse la plus appropriée semble être la réponse 2."}}} +{"id": 507, "year": 2020, "question_id_specific": 89, "full_question": "Un homme de 46 ans souffrant de troubles bipolaires est amené au service des urgences après une overdose de carbonate de lithium. L'examen révèle des tremblements sévères, une ataxie, une dysarthrie, une myoclonie et des fasciculations. La lithémie est de 4,1 mEq/L (toxicité > 1,6 mEq/L). Laquelle des options thérapeutiques suivantes serait la plus indiquée ?", "full_answer": "Soyez prudent avec le lithium. Des cas d'intoxication ont été observés avec le lithium normal. Ce n'est pas rare, surtout en raison des interactions. Un médicament très efficace si tout le monde est prévenu.", "type": "PSYCHIATRIE", "options": {"1": "Aminophylline associée à un cathartique.", "2": "Charbon actif.", "3": "Hémodialyse.", "4": "Diurèse forcée.", "5": null}, "correct_option": 3, "explanations": {"1": {"exist": false, "char_ranges": [], "word_ranges": [], "text": ""}, "2": {"exist": false, "char_ranges": [], "word_ranges": [], "text": ""}, "3": {"exist": false, "char_ranges": [], "word_ranges": [], "text": ""}, "4": {"exist": false, "char_ranges": [], "word_ranges": [], "text": ""}, "5": {"exist": false, "char_ranges": [], "word_ranges": [], "text": ""}}} +{"id": 552, "year": 2022, "question_id_specific": 89, "full_question": "Une fillette de 4 ans s'est présentée avec une forte fièvre depuis 6 jours. L'examen clinique a révélé une éruption maculopapuleuse érythémateuse sur le tronc et la région génitale, avec une tendance à la confluence, sans devenir scarlatiniforme ; une injection conjonctivale sans écoulement et des lèvres rouges avec une langue framboisée. Il y avait également un érythème avec œdème sur les mains et les pieds et une adénopathie cervicale unilatérale mesurant 2 cm de diamètre. Le diagnostic clinique de suspicion le plus probable est le suivant :", "full_answer": "Il ne fait aucun doute qu'ils ne répondent pas aux critères diagnostiques de la maladie de Kawasaki (fièvre de plusieurs jours, éruption cutanée, injection conjonctivale, lèvres rouges, langue framboisée, œdème des parties acrales et adénopathie).", "type": "DERMATOLOGIE", "options": {"1": "Maladie de Kawasaki.", "2": "La rougeole.", "3": "Rubéole.", "4": "Scarlatine.", "5": null}, "correct_option": 1, "explanations": {"1": {"exist": true, "char_ranges": [[0, 247]], "word_ranges": [[0, 35]], "text": "Il ne fait aucun doute qu'ils ne répondent pas aux critères diagnostiques de la maladie de Kawasaki (fièvre de plusieurs jours, éruption cutanée, injection conjonctivale, lèvres rouges, langue framboisée, œdème des parties acrales et adénopathie)."}, "2": {"exist": false, "char_ranges": [], "word_ranges": [], "text": ""}, "3": {"exist": false, "char_ranges": [], "word_ranges": [], "text": ""}, "4": {"exist": false, "char_ranges": [], "word_ranges": [], "text": ""}, "5": {"exist": false, "char_ranges": [], "word_ranges": [], "text": ""}}} +{"id": 598, "year": 2022, "question_id_specific": 181, "full_question": "Un homme de 67 ans s'est présenté avec trois mois d'asthénie et de fièvre fébrile, auxquels s'est ajoutée le dernier mois une obstruction nasale avec émission de mucus avec quelques caillots. Au cours des derniers jours, il a remarqué une douleur dans l'œil droit et une asymétrie par rapport à l'œil controlatéral. L'examen physique a révélé une proptose du globe oculaire droit et l'inspection des narines a révélé une muqueuse érythémateuse avec des croûtes sérohématiques. Le reste de l'examen était normal. Les analyses sanguines (hémogramme, fonctions rénale et hépatique) sont normales à l'exception d'une ESR de 65 mm/h ; l'analyse d'urine montre une microhématurie et une protéinurie de 520 mg/24h. Quel est le diagnostic initial le plus probable ?", "full_answer": "Ce patient présente une granulomatose avec polyangéite, anciennement connue sous le nom de granulomatose de Wegener. Dans le cas clinique, on nous parle de l'atteinte oto-rhino-laryngologique qui est présente chez 92% des patients. Au niveau pulmonaire, on nous signale la présence de mucus avec un caillot ; ce type d'atteinte est présent chez 85% des patients. Toujours à la fin du cas, l'analyse d'urine a montré une glomérulonéphrite, présente chez 77% des patients.", "type": "RHEUMATOLOGIE", "options": {"1": "Granulomatose éosinophile avec polyangéite.", "2": "Polyangéite microscopique.", "3": "Granulomatose avec polyangéite.", "4": "Polyarthrite noueuse.", "5": null}, "correct_option": 3, "explanations": {"1": {"exist": false, "char_ranges": [], "word_ranges": [], "text": ""}, "2": {"exist": false, "char_ranges": [], "word_ranges": [], "text": ""}, "3": {"exist": true, "char_ranges": [[0, 470]], "word_ranges": [[0, 73]], "text": "Ce patient présente une granulomatose avec polyangéite, anciennement connue sous le nom de granulomatose de Wegener. Dans le cas clinique, on nous parle de l'atteinte oto-rhino-laryngologique qui est présente chez 92% des patients. Au niveau pulmonaire, on nous signale la présence de mucus avec un caillot ; ce type d'atteinte est présent chez 85% des patients. Toujours à la fin du cas, l'analyse d'urine a montré une glomérulonéphrite, présente chez 77% des patients."}, "4": {"exist": false, "char_ranges": [], "word_ranges": [], "text": ""}, "5": {"exist": false, "char_ranges": [], "word_ranges": [], "text": ""}}} +{"id": 133, "year": 2012, "question_id_specific": 168, "full_question": "Une femme de 75 ans a été diagnostiquée il y a trois ans avec des drusen mous au fond de l'œil. Elle a signalé une métamorphopsie et une perte visuelle importante dans l'œil droit au cours des deux dernières semaines, ce qui l'empêche de lire. Indiquez le diagnostic le plus probable :", "full_answer": "Parmi les cinq options, quatre pourraient être compatibles avec le tableau clinique évoqué. Nous pourrions d'abord exclure la choriorétinopathie séreuse centrale car, par définition, elle survient chez les jeunes (jusqu'à l'âge de 55 ans environ). La clé de la différenciation entre les quatre autres est la présence de drusen mous dans le fond d'œil. Bien qu'il ne s'agisse pas d'un \"diagnostic\" (la question aurait pu être mieux formulée), il s'agit d'une constatation liée à la dégénérescence maculaire liée à l'âge, ou dégénérescence maculaire liée à l'âge. La forme sèche ou atrophique se manifeste par une perte de vision lentement progressive. La forme humide ou exsudative entraîne une perte visuelle sévère, aiguë ou rapidement progressive (généralement due à une hémorragie sous-rétinienne accompagnée d'une métamorphopsie (distorsion de l'image due au soulèvement de la rétine). Une membrane épirétinienne évolue généralement plus lentement. Dans le cas d'un trou maculaire, la perte visuelle est plus importante (une métamorphopsie peut également être signalée, mais elle est moins probable dans la phase aiguë). En cas de thrombose de la veine centrale de la rétine, la perte visuelle est plus importante et n'explique généralement pas la métamorphopsie (mais elle peut survenir en raison d'un œdème maculaire).", "type": "OPHTHALMOLOGIE", "options": {"1": "Membrane épirétinienne maculaire.", "2": "Trou maculaire.", "3": "Thrombose de la veine centrale de la rétine.", "4": "Dégénérescence maculaire liée à l'âge.", "5": "Choriorétinopathie séreuse centrale."}, "correct_option": 4, "explanations": {"1": {"exist": true, "char_ranges": [[890, 952]], "word_ranges": [[132, 139]], "text": "Une membrane épirétinienne évolue généralement plus lentement."}, "2": {"exist": true, "char_ranges": [[953, 1124]], "word_ranges": [[139, 166]], "text": "Dans le cas d'un trou maculaire, la perte visuelle est plus importante (une métamorphopsie peut également être signalée, mais elle est moins probable dans la phase aiguë)."}, "3": {"exist": true, "char_ranges": [[1125, 1324]], "word_ranges": [[166, 198]], "text": "En cas de thrombose de la veine centrale de la rétine, la perte visuelle est plus importante et n'explique généralement pas la métamorphopsie (mais elle peut survenir en raison d'un œdème maculaire)."}, "4": {"exist": true, "char_ranges": [[248, 561]], "word_ranges": [[35, 86]], "text": "La clé de la différenciation entre les quatre autres est la présence de drusen mous dans le fond d'œil. Bien qu'il ne s'agisse pas d'un \"diagnostic\" (la question aurait pu être mieux formulée), il s'agit d'une constatation liée à la dégénérescence maculaire liée à l'âge, ou dégénérescence maculaire liée à l'âge."}, "5": {"exist": true, "char_ranges": [[92, 247]], "word_ranges": [[13, 35]], "text": "Nous pourrions d'abord exclure la choriorétinopathie séreuse centrale car, par définition, elle survient chez les jeunes (jusqu'à l'âge de 55 ans environ)."}}} +{"id": 186, "year": 2013, "question_id_specific": 67, "full_question": "Une femme de 56 ans a consulté pour des douleurs dans la colonne dorso-lombaire et une difficulté progressive à effectuer des tâches normales. Au cours des cinq dernières années, elle avait pris du poids, présentait facilement des ecchymoses et une hypertension artérielle a été détectée. Examen physique : obésité centrale prédominante, faciès arrondi, augmentation de la graisse sus-claviculaire, diminution de la force musculaire proximale et quelques stries rougeâtres sur l'abdomen. Sa glycémie est de 136 mg/dL et l'étude radiologique montre une ostéoporose et un écrasement vertébral. Quelle est, selon vous, l'interprétation et l'attitude la plus cohérente ?", "full_answer": "Une question légèrement alambiquée sur le diagnostic du syndrome de Cushing, mais facile si l'on comprend bien les étapes. Nous sommes tous d'accord pour dire que le patient semble souffrir du syndrome de Cushing (nous excluons 1). Pour le diagnostic, il faut mesurer le cortisol libre urinaire, effectuer un test de suppression à la dexaméthasone ou mesurer le cortisol salivaire nocturne. Si l'hypercortisolisme est confirmé, l'origine doit être recherchée ; la mesure de l'ACTH indique l'origine : supprimée en cas d'hypercortisolisme d'origine surrénalienne ou d'utilisation prolongée de corticostéroïdes et élevée ou normale si l'origine est hypophysaire ou due à une sécrétion ectopique d'ACTH (par conséquent, les points 4 et 5 sont faux). Si l'ACTH est basse, un scanner de l'abdomen doit être effectué pour localiser l'origine. Si l'ACTH est normale ou élevée, une IRM hypophysaire doit être réalisée (les adénomes hypophysaires responsables du syndrome de Cushing sont très petits et le scanner hypophysaire est moins sensible que l'IRM). Il est très important d'établir un ordre correct afin de parvenir à un diagnostic correct : 1er : confirmer l'hypercortisolisme ou le syndrome de Cushing, 2ème : confirmer l'hypercortisolisme ou le syndrome de Cushing. Cushing, 2ème : dosage de l'ACTH pour orienter l'étiologie. 3ème : examen d'imagerie en fonction des taux d'ACTH. Par conséquent, la réponse 2 est fausse (le scanner abdominal n'est pas un examen qui permet d'exclure le s. Cushing) ; un examen d'imagerie ne doit être réalisé qu'une fois que le diagnostic d'hypercortisolisme a été clairement posé. Par conséquent, la vraie réponse est 3.", "type": "ENDOCRINOLOGIE", "options": {"1": "Ostéoporose post-ménopausique, diabète de type 2 et hypertension essentielle, avec diminution de la force due à la polyneuropathie diabétique.", "2": "La maladie de Cushing doit être exclue par un test de suppression de la dexaméthasone et une tomodensitométrie crânienne.", "3": "Suggérer une maladie de Cushing. Déterminer le cortisol libre urinaire et l'ACTH basale, ce qui permet d'orienter l'étiologie et de sélectionner la technique d'imagerie la plus appropriée.", "4": "Cela ressemble à la maladie de Cushing. Si l'ACTH basale est élevée, cela peut être dû à l'utilisation de corticostéroïdes ou à une tumeur surrénalienne, et une IRM doit être réalisée.", "5": "Il souffre probablement de la maladie de Cushing. Si l'ACTH basale est basse, il a probablement un micro-adénome hypophysaire et une tomodensitométrie crânienne doit être réalisée."}, "correct_option": 3, "explanations": {"1": {"exist": true, "char_ranges": [[123, 231]], "word_ranges": [[19, 37]], "text": "Nous sommes tous d'accord pour dire que le patient semble souffrir du syndrome de Cushing (nous excluons 1)."}, "2": {"exist": true, "char_ranges": [[1398, 1501]], "word_ranges": [[212, 231]], "text": "la réponse 2 est fausse (le scanner abdominal n'est pas un examen qui permet d'exclure le s. Cushing) ;"}, "3": {"exist": true, "char_ranges": [[1502, 1616]], "word_ranges": [[231, 248]], "text": "un examen d'imagerie ne doit être réalisé qu'une fois que le diagnostic d'hypercortisolisme a été clairement posé."}, "4": {"exist": true, "char_ranges": [[391, 746]], "word_ranges": [[61, 112]], "text": "Si l'hypercortisolisme est confirmé, l'origine doit être recherchée ; la mesure de l'ACTH indique l'origine : supprimée en cas d'hypercortisolisme d'origine surrénalienne ou d'utilisation prolongée de corticostéroïdes et élevée ou normale si l'origine est hypophysaire ou due à une sécrétion ectopique d'ACTH (par conséquent, les points 4 et 5 sont faux)."}, "5": {"exist": true, "char_ranges": [[391, 746]], "word_ranges": [[61, 112]], "text": "Si l'hypercortisolisme est confirmé, l'origine doit être recherchée ; la mesure de l'ACTH indique l'origine : supprimée en cas d'hypercortisolisme d'origine surrénalienne ou d'utilisation prolongée de corticostéroïdes et élevée ou normale si l'origine est hypophysaire ou due à une sécrétion ectopique d'ACTH (par conséquent, les points 4 et 5 sont faux)."}}} +{"id": 7, "year": 2011, "question_id_specific": 208, "full_question": "Lors d'une coloscopie, une tumeur de 5 cm a été détectée dans le côlon droit d'un homme de 48 ans. Aucune autre lésion n'a été décelée. Sa grand-mère maternelle a également souffert d'un cancer du côlon. Les biopsies sont superficielles et montrent une tumeur peu différenciée avec d'abondantes cellules inflammatoires dans le stroma, qui est diagnostiquée comme un carcinome de type médullaire.", "full_answer": "Question compliquée sur un adénocarcinome inhabituel. Pour écarter 1 qui serait facile à éliminer, le traitement du cancer du côlon est chirurgical (avec un traitement néoadjuvant pour le cancer du rectum). Si les biopsies confirment l'adénocarcinome, il n'est pas nécessaire de les répéter, car il est nécessaire d'avoir le prélèvement complet. La plupart des cancers du côlon se développent sur des adénomes, on peut donc éliminer celui-ci. Parmi les deux autres, il est important de savoir que le chiffre 5 est correct car ils présentent presque toujours une instabilité des microsatellites même s'ils ne répondent pas aux critères d'Amsterdam et ont un meilleur pronostic que d'autres cancers du côlon peu différenciés.", "type": "DIGESTIF", "options": {"1": "La chimiothérapie est le traitement de choix.", "2": "La biopsie étant superficielle, elle doit être répétée avant le traitement.", "3": "Le pronostic de la tumeur dépend principalement de son degré élevé d'anaplasie.", "4": "Il est peu probable que cette tumeur se soit développée sur un adénome antérieur.", "5": "L'instabilité des microsatellites et les gènes de réparation des erreurs de l'ADN devraient être étudiés."}, "correct_option": 5, "explanations": {"1": {"exist": true, "char_ranges": [[54, 206]], "word_ranges": [[6, 31]], "text": "Pour écarter 1 qui serait facile à éliminer, le traitement du cancer du côlon est chirurgical (avec un traitement néoadjuvant pour le cancer du rectum)."}, "2": {"exist": true, "char_ranges": [[207, 345]], "word_ranges": [[31, 51]], "text": "Si les biopsies confirment l'adénocarcinome, il n'est pas nécessaire de les répéter, car il est nécessaire d'avoir le prélèvement complet."}, "3": {"exist": true, "char_ranges": [[346, 442]], "word_ranges": [[51, 67]], "text": "La plupart des cancers du côlon se développent sur des adénomes, on peut donc éliminer celui-ci."}, "4": {"exist": true, "char_ranges": [[443, 723]], "word_ranges": [[67, 111]], "text": "Parmi les deux autres, il est important de savoir que le chiffre 5 est correct car ils présentent presque toujours une instabilité des microsatellites même s'ils ne répondent pas aux critères d'Amsterdam et ont un meilleur pronostic que d'autres cancers du côlon peu différenciés."}, "5": {"exist": true, "char_ranges": [[443, 723]], "word_ranges": [[67, 111]], "text": "Parmi les deux autres, il est important de savoir que le chiffre 5 est correct car ils présentent presque toujours une instabilité des microsatellites même s'ils ne répondent pas aux critères d'Amsterdam et ont un meilleur pronostic que d'autres cancers du côlon peu différenciés."}}} +{"id": 152, "year": 2012, "question_id_specific": 72, "full_question": "Un patient hyperuricémique qui prend habituellement 100 mg d'allopurinol par jour se présente au service des urgences avec une douleur aiguë et des signes inflammatoires au niveau du genou droit. Une arthrocentèse est pratiquée et l'on observe au microscope à lumière polarisée des cristaux intracellulaires à biréfringence négative. Parmi les approches thérapeutiques suivantes, laquelle est la plus appropriée dans ce cas ?", "full_answer": "En cas d'arthrite goutteuse aiguë (la présence de cristaux intracellulaires à biréfringence négative le confirme) chez un patient hyperuricémique préalablement traité par allopurinol, un AINS doit être ajouté jusqu'à la disparition de la crise. Le traitement hypouricémique ne doit pas être modifié pendant cette période car il altère la chaîne métabolique de l'acide urique et peut aggraver la situation.", "type": "RHEUMATOLOGIE", "options": {"1": "Arrêter l'allopurinol et commencer la colchicine.", "2": "Arrêter l'allopurinol et commencer les AINS.", "3": "Ajouter un AINS jusqu'à ce que la crise s'estompe.", "4": "Augmenter la dose d'allopurinol à 300 mg/jour.", "5": "Remplacer l'allopurinol par un uricosurique."}, "correct_option": 3, "explanations": {"1": {"exist": true, "char_ranges": [[245, 405]], "word_ranges": [[34, 59]], "text": "Le traitement hypouricémique ne doit pas être modifié pendant cette période car il altère la chaîne métabolique de l'acide urique et peut aggraver la situation."}, "2": {"exist": true, "char_ranges": [[245, 405]], "word_ranges": [[34, 59]], "text": "Le traitement hypouricémique ne doit pas être modifié pendant cette période car il altère la chaîne métabolique de l'acide urique et peut aggraver la situation."}, "3": {"exist": true, "char_ranges": [[0, 244]], "word_ranges": [[0, 34]], "text": "En cas d'arthrite goutteuse aiguë (la présence de cristaux intracellulaires à biréfringence négative le confirme) chez un patient hyperuricémique préalablement traité par allopurinol, un AINS doit être ajouté jusqu'à la disparition de la crise."}, "4": {"exist": true, "char_ranges": [[245, 405]], "word_ranges": [[34, 59]], "text": "Le traitement hypouricémique ne doit pas être modifié pendant cette période car il altère la chaîne métabolique de l'acide urique et peut aggraver la situation."}, "5": {"exist": true, "char_ranges": [[245, 405]], "word_ranges": [[34, 59]], "text": "Le traitement hypouricémique ne doit pas être modifié pendant cette période car il altère la chaîne métabolique de l'acide urique et peut aggraver la situation."}}} +{"id": 243, "year": 2014, "question_id_specific": 112, "full_question": "Une femme de 20 ans se présente avec une fièvre aiguë, une lymphadénopathie cervicale et une éruption cutanée. Selon la patiente, elle a eu, il y a trois semaines, une relation sexuelle qui aurait pu entraîner un risque de contracter le virus VIH. Laquelle des réponses suivantes est vraie ?", "full_answer": "Question simple si l'on connaît bien le processus de diagnostic du VIH. À première vue, nous pourrions directement exclure la réponse 2, car nous savons que la mononucléose peut être l'expression du premier stade de l'infection par le VIH, en particulier lorsqu'il y a des antécédents de contact à risque. Les réponses 3 et 4 sont également facilement éliminées ; ELISA est une technique très sensible mais peu spécifique et une confirmation par un test Western-blot est TOUJOURS nécessaire. Les réponses 1 et 5 se contredisent, ce qui signifie que l'une des deux est correcte. Dans ce cas, la fausse réponse est la réponse 1, c'est une réponse très catégorique (\"exclure\") qui ne correspond pas tout à fait à la réalité ; Comme nous l'avons dit, ELISA est une technique très sensible, de sorte qu'un résultat négatif pourrait nous laisser sereins, mais nous savons tous qu'aucun test médical n'exclut quoi que ce soit avec une certitude totale (à moins que sa sensibilité ne soit de 100 %, ce qui n'est pas le cas) et, d'autre part, nous pourrions dire que, dans ce cas, nous avons une forte suspicion clinique (le tableau est suggestif, et les antécédents sont là). En outre, trois semaines seulement se sont écoulées depuis le contact, de sorte que notre patient se trouve probablement dans la \"période de latence\" au cours de laquelle les techniques sérologiques peuvent ne pas être rentables. En conclusion, la réponse qui nous convient le mieux serait 5. Pendant la période fenêtre, une charge virale peut être indiquée pour le diagnostic (positive s'il y a plus de 10 000 copies), bien que ce test ne soit pas effectué en routine.", "type": "LES MALADIES INFECTIEUSES", "options": {"1": "Une sérologie VIH-1/VIH-2 négative par ELISA exclut la possibilité que le patient ait été infecté par le virus VIH.", "2": "Le processus clinique du patient ne correspond pas à une infection aiguë par le VIH.", "3": "Si le test ELISA du patient est positif, aucun autre diagnostic d'infection par le VIH n'est nécessaire.", "4": "La technique ELISA a une sensibilité élevée pour le diagnostic de l'infection par le VIH, mais sa spécificité est encore plus grande.", "5": "Si le test ELISA pour diagnostiquer le VIH chez le patient était négatif, nous pouvions déterminer la charge virale dans le sang par la technique PCR."}, "correct_option": 5, "explanations": {"1": {"exist": true, "char_ranges": [[747, 1015]], "word_ranges": [[126, 173]], "text": "ELISA est une technique très sensible, de sorte qu'un résultat négatif pourrait nous laisser sereins, mais nous savons tous qu'aucun test médical n'exclut quoi que ce soit avec une certitude totale (à moins que sa sensibilité ne soit de 100 %, ce qui n'est pas le cas)"}, "2": {"exist": true, "char_ranges": [[88, 305]], "word_ranges": [[15, 50]], "text": "nous pourrions directement exclure la réponse 2, car nous savons que la mononucléose peut être l'expression du premier stade de l'infection par le VIH, en particulier lorsqu'il y a des antécédents de contact à risque."}, "3": {"exist": true, "char_ranges": [[364, 491]], "word_ranges": [[60, 79]], "text": "ELISA est une technique très sensible mais peu spécifique et une confirmation par un test Western-blot est TOUJOURS nécessaire."}, "4": {"exist": true, "char_ranges": [[364, 491]], "word_ranges": [[60, 79]], "text": "ELISA est une technique très sensible mais peu spécifique et une confirmation par un test Western-blot est TOUJOURS nécessaire."}, "5": {"exist": true, "char_ranges": [[1413, 1637]], "word_ranges": [[236, 277]], "text": "la réponse qui nous convient le mieux serait 5. Pendant la période fenêtre, une charge virale peut être indiquée pour le diagnostic (positive s'il y a plus de 10 000 copies), bien que ce test ne soit pas effectué en routine."}}} +{"id": 458, "year": 2018, "question_id_specific": 126, "full_question": "Un homme de 47 ans, aux antécédents d'adénome pléomorphe de la parotide droite, traité par chirurgie (parotidectomie extrafaciale) il y a 6 mois, se présente à notre clinique en raison de douleurs à la mastication accompagnées de sueurs et d'une rougeur de la peau dans la région préauriculaire. Quel serait le meilleur traitement ?", "full_answer": "Votre question porte sur le syndrome auriculotemporal ou syndrome de Frey. C'est l'expression clinique d'une neuropathie végétative causée par une lésion mécanique ou irritative des fibres végétatives du nerf auriculotemporal dans son trajet infratemporal. Dans notre environnement, la cause la plus fréquente est la chirurgie de la glande fendue avec un pourcentage d'occurrence entre 10 et 80%. Le traitement conservateur par toxine botulique donne de bons résultats.", "type": "OTORHINOLARYNGOLOGIE ET CHIRURGIE MAXILLO-FACIALE", "options": {"1": "Parotidectomie totale élargie en cas de suspicion de récidive tumorale.", "2": "Prégabaline.", "3": "Injection intradermique de toxine botulique.", "4": "Antibiothérapie à large spectre.", "5": null}, "correct_option": 3, "explanations": {"1": {"exist": false, "char_ranges": [], "word_ranges": [], "text": ""}, "2": {"exist": false, "char_ranges": [], "word_ranges": [], "text": ""}, "3": {"exist": true, "char_ranges": [[257, 469]], "word_ranges": [[34, 67]], "text": "Dans notre environnement, la cause la plus fréquente est la chirurgie de la glande fendue avec un pourcentage d'occurrence entre 10 et 80%. Le traitement conservateur par toxine botulique donne de bons résultats."}, "4": {"exist": false, "char_ranges": [], "word_ranges": [], "text": ""}, "5": {"exist": false, "char_ranges": [], "word_ranges": [], "text": ""}}} +{"id": 123, "year": 2012, "question_id_specific": 223, "full_question": "Femme enceinte de 27 ans, 30 semaines de gestation. Elle s'est présentée au service des urgences hier après avoir constaté des douleurs dans la région lombaire gauche et une dysurie. Aucune sensation fébrile. Elle a signalé des infections urinaires répétées. L'analyse d'urine montre Hb 3+, leucocytes 3+, nitrites 2+, sédiment : 15-20 leucocytes par champ et 5-10 globules rouges par champ. Compte tenu de l'opportunité d'un traitement empirique, lequel des micro-organismes suivants est la cause la plus fréquente d'infection chez les femmes enceintes ?", "full_answer": "La bonne réponse est la première. La cause la plus fréquente des infections urinaires est Escherichia coli, y compris chez les femmes enceintes.", "type": "MICROBIOLOGIE", "options": {"1": "Escherichia coli.", "2": "Enterococcus faecalis.", "3": "Streptococcus agalactiae.", "4": "Proteus mirabilis.", "5": "Satphylococcus saprophyticus."}, "correct_option": 1, "explanations": {"1": {"exist": true, "char_ranges": [[34, 144]], "word_ranges": [[6, 23]], "text": "La cause la plus fréquente des infections urinaires est Escherichia coli, y compris chez les femmes enceintes."}, "2": {"exist": false, "char_ranges": [], "word_ranges": [], "text": ""}, "3": {"exist": false, "char_ranges": [], "word_ranges": [], "text": ""}, "4": {"exist": false, "char_ranges": [], "word_ranges": [], "text": ""}, "5": {"exist": false, "char_ranges": [], "word_ranges": [], "text": ""}}} +{"id": 522, "year": 2021, "question_id_specific": 165, "full_question": "Une femme de 66 ans a été diagnostiquée avec un diabète de type 2 il y a trois mois. Elle a un IMC de 31 kg/m2 et son contrôle glycémique est médiocre malgré un programme de mesures non pharmacologiques (alimentation saine, exercice physique). Parmi les médicaments hypoglycémiants suivants, lequel est associé à une prise de poids et doit être évité chez cette patiente ?", "full_answer": "Une question un peu plus compliquée que la précédente. En ce qui concerne le poids, la metformine, dans une moindre mesure, les inhibiteurs du SGLT-2 et, surtout, les agonistes du GLP-1 ont été associés à une diminution significative du poids chez les patients atteints de DM-2 (réponses 1, 3 et 4 incorrectes). En revanche, la pioglitazone, comme l'indique son étiquette, peut entraîner une prise de poids dose-dépendante, principalement due à l'accumulation de graisse et, dans certains cas, en plus de la rétention d'eau.", "type": "ENDOCRINOLOGIE", "options": {"1": "Metformine (biguanide).", "2": "Pioglitazone (thiazolidinedione).", "3": "Canagliflozin (inhibiteur du cotransporteur sodium-glucose 2 - iSGLT2).", "4": "Liraglutide (agoniste des récepteurs du GLP-1).", "5": null}, "correct_option": 2, "explanations": {"1": {"exist": true, "char_ranges": [[55, 311]], "word_ranges": [[9, 52]], "text": "En ce qui concerne le poids, la metformine, dans une moindre mesure, les inhibiteurs du SGLT-2 et, surtout, les agonistes du GLP-1 ont été associés à une diminution significative du poids chez les patients atteints de DM-2 (réponses 1, 3 et 4 incorrectes)."}, "2": {"exist": true, "char_ranges": [[312, 524]], "word_ranges": [[52, 83]], "text": "En revanche, la pioglitazone, comme l'indique son étiquette, peut entraîner une prise de poids dose-dépendante, principalement due à l'accumulation de graisse et, dans certains cas, en plus de la rétention d'eau."}, "3": {"exist": true, "char_ranges": [[55, 311]], "word_ranges": [[9, 52]], "text": "En ce qui concerne le poids, la metformine, dans une moindre mesure, les inhibiteurs du SGLT-2 et, surtout, les agonistes du GLP-1 ont été associés à une diminution significative du poids chez les patients atteints de DM-2 (réponses 1, 3 et 4 incorrectes)."}, "4": {"exist": true, "char_ranges": [[55, 311]], "word_ranges": [[9, 52]], "text": "En ce qui concerne le poids, la metformine, dans une moindre mesure, les inhibiteurs du SGLT-2 et, surtout, les agonistes du GLP-1 ont été associés à une diminution significative du poids chez les patients atteints de DM-2 (réponses 1, 3 et 4 incorrectes)."}, "5": {"exist": false, "char_ranges": [], "word_ranges": [], "text": ""}}} +{"id": 209, "year": 2014, "question_id_specific": 185, "full_question": "Une femme de 32 ans demande un conseil préconceptionnel. La patiente indique qu'elle a subi une conisation cervicale pour une lésion intraépithéliale de haut grade (LIEH) et qu'elle a ensuite fait trois fausses couches entre la 20e et la 22e semaine de gestation. Elle n'a pas d'enfant vivant. Les trois fois, elle s'est présentée au service des urgences avec une sensation de lourdeur dans l'hypogastre et on a constaté qu'elle était dilatée de 8 cm et que les membranes amniotiques étaient proéminentes. Elle n'avait jamais ressenti de contractions auparavant. Quels conseils lui donneriez-vous pour sa prochaine grossesse ?", "full_answer": "Il s'agit d'un cas typique d'incompétence cervicale (et cette patiente a la conisation comme facteur de risque). Cette pathologie consiste en une dilatation du col de l'utérus en l'absence de contractions, nécessitant un cerclage pour éviter une fausse couche ou un accouchement prématuré. Idéalement, le cerclage est effectué de manière élective, sur une base programmée, et non lorsque la patiente arrive au service des urgences \"in extemis\" (ce qui se fait également, mais avec un taux de réussite plus faible... idéalement, il devrait être effectué à froid).", "type": "GYNÉCOLOGIE ET OBSTÉTRIQUE", "options": {"1": "Je prescrirais une prophylaxie orale à base d'atosiban pendant toute la durée de la grossesse.", "2": "Je proposerais une maturation pulmonaire avec des corticostéroïdes à partir de 19-20 semaines de gestation.", "3": "Je recommanderais un cerclage du col de l'utérus à 14 semaines de gestation.", "4": "Je lui conseillerais de ne pas tenter d'autres grossesses en raison du risque élevé de récidive.", "5": "Je recommanderais les techniques de procréation assistée."}, "correct_option": 3, "explanations": {"1": {"exist": false, "char_ranges": [], "word_ranges": [], "text": ""}, "2": {"exist": false, "char_ranges": [], "word_ranges": [], "text": ""}, "3": {"exist": true, "char_ranges": [[0, 289]], "word_ranges": [[0, 43]], "text": "Il s'agit d'un cas typique d'incompétence cervicale (et cette patiente a la conisation comme facteur de risque). Cette pathologie consiste en une dilatation du col de l'utérus en l'absence de contractions, nécessitant un cerclage pour éviter une fausse couche ou un accouchement prématuré."}, "4": {"exist": false, "char_ranges": [], "word_ranges": [], "text": ""}, "5": {"exist": false, "char_ranges": [], "word_ranges": [], "text": ""}}} +{"id": 282, "year": 2016, "question_id_specific": 55, "full_question": "Une femme de 70 ans est admise aux soins intensifs après avoir subi un IAM antérieur traité par angioplastie coronaire et pose d'un stent sur l'artère descendante antérieure. Quatre jours plus tard, elle présente soudainement une hypotension nécessitant un soutien volumétrique vigoureux, l'instauration de médicaments vasoactifs, une intubation orotrachéale et la mise en place d'une ventilation mécanique. L'examen physique a révélé un souffle qui n'était pas présent auparavant. La suspicion d'une complication mécanique de l'infarctus a conduit à une échocardiographie transthoracique montrant un épanchement péricardique. Cochez la bonne réponse :", "full_answer": "En cas de rupture de la paroi libre, il n'y a pas de frémito palpable.", "type": "CARDIOLOGIE ET CHIRURGIE VASCULAIRE", "options": {"1": "La mortalité sous traitement médical est de 20 %.", "2": "En cas de rupture de la paroi libre, on observe un saut oxymétrique dans le ventricule droit lors du cathétérisme de Swan-Ganz.", "3": "En cas de rupture de la paroi libre, il n'y a pas de frémito palpable.", "4": "Les complications mécaniques surviennent généralement le premier jour suivant l'infarctus.", "5": null}, "correct_option": 3, "explanations": {"1": {"exist": false, "char_ranges": [], "word_ranges": [], "text": ""}, "2": {"exist": false, "char_ranges": [], "word_ranges": [], "text": ""}, "3": {"exist": true, "char_ranges": [[0, 70]], "word_ranges": [[0, 15]], "text": "En cas de rupture de la paroi libre, il n'y a pas de frémito palpable."}, "4": {"exist": false, "char_ranges": [], "word_ranges": [], "text": ""}, "5": {"exist": false, "char_ranges": [], "word_ranges": [], "text": ""}}} +{"id": 464, "year": 2019, "question_id_specific": 155, "full_question": "Un homme de 58 ans, trois semaines après une grave entorse de la cheville, présente une dyspnée au repos, des vertiges et une syncope rapidement progressifs. À son arrivée à l'hôpital, il présente une hypotension (TA systolique 80 mmHg, diastolique 40 mmHg) et une mauvaise perfusion. Il est intubé et relié à la ventilation mécanique et la noradrénaline est administrée. L'échocardiographie montre des signes d'hypertension pulmonaire. L'angio-TDM montre de multiples défauts de réplétion occupant les deux artères pulmonaires principales. Parmi les traitements suivants, lequel serait associé à l'amélioration hémodynamique la plus rapide dans ce cas ?", "full_answer": "Le patient a souffert d'une thromboembolie pulmonaire qui, en plus d'être bilatérale, a provoqué une altération hémodynamique sévère, un état de choc et a nécessité une ventilation mécanique. On ne nous dit pas qu'il présente des contre-indications à la fibrinolyse, qui serait donc l'option la plus appropriée en raison de sa rapidité d'administration et d'action. L'héparine sodique est réservée aux cas où le patient est stable. La thromboendartériectomie pourrait être pratiquée en urgence dans des centres sélectionnés (pas disponibles dans tous) chez les patients pour lesquels la fibrinolyse systémique est contre-indiquée, ou dans des centres où l'expérience de cette technique est avérée et où elle peut être pratiquée rapidement. Le filtre cave inférieur est un traitement indiqué en phase aiguë chez les patients stables, lorsque l'anticoagulation est contre-indiquée ; ou a posteriori, en prophylaxie, dans ce groupe de patients. Dans le dernier document de consensus SEPAR, les indications de fibrinolyse comprennent les patients à risque intermédiaire (qui pourraient bénéficier d'une fibrinolyse) et les patients à haut risque. Ils classent le risque intermédiaire comme des PESI ≥ 1 ou des PESI III-IV. Dans ce sous-groupe, les patients présentant une dysfonction ventriculaire droite, un taux de troponine ou de BNP supérieur au seuil et une thrombose veineuse profonde semblent bénéficier le plus de la fibrinolyse. Dans le sous-groupe des patients à haut risque (défini par des critères d'hypotension ou de choc cardiogénique), l'utilisation de la fibrinolyse systémique est beaucoup plus claire, comme dans le cas présenté dans la question.", "type": "SOINS CRITIQUES ET D'URGENCE", "options": {"1": "Perfusion intraveineuse d'héparine sodique.", "2": "Thromboendartectomie.", "3": "Fibrinolyse systémique avec rt-PA (alteplase) 100 mg par voie intraveineuse.", "4": "Filtre pour la veine cave inférieure.", "5": null}, "correct_option": 3, "explanations": {"1": {"exist": true, "char_ranges": [[366, 431]], "word_ranges": [[55, 66]], "text": "L'héparine sodique est réservée aux cas où le patient est stable."}, "2": {"exist": true, "char_ranges": [[432, 739]], "word_ranges": [[66, 109]], "text": "La thromboendartériectomie pourrait être pratiquée en urgence dans des centres sélectionnés (pas disponibles dans tous) chez les patients pour lesquels la fibrinolyse systémique est contre-indiquée, ou dans des centres où l'expérience de cette technique est avérée et où elle peut être pratiquée rapidement."}, "3": {"exist": true, "char_ranges": [[0, 365]], "word_ranges": [[0, 55]], "text": "Le patient a souffert d'une thromboembolie pulmonaire qui, en plus d'être bilatérale, a provoqué une altération hémodynamique sévère, un état de choc et a nécessité une ventilation mécanique. On ne nous dit pas qu'il présente des contre-indications à la fibrinolyse, qui serait donc l'option la plus appropriée en raison de sa rapidité d'administration et d'action."}, "4": {"exist": true, "char_ranges": [[740, 941]], "word_ranges": [[109, 139]], "text": "Le filtre cave inférieur est un traitement indiqué en phase aiguë chez les patients stables, lorsque l'anticoagulation est contre-indiquée ; ou a posteriori, en prophylaxie, dans ce groupe de patients."}, "5": {"exist": false, "char_ranges": [], "word_ranges": [], "text": ""}}} +{"id": 502, "year": 2020, "question_id_specific": 77, "full_question": "Un nourrisson de six mois se présente aux urgences pour détresse respiratoire. Examen : température axillaire 37,2°C, fréquence respiratoire 40 rpm, fréquence cardiaque 160 bpm, pression artérielle 90/45 mmHg, SatO2 95 % à l'air ambiant. Il présente une détresse respiratoire modérée avec rétraction intercostale et sous-costale. Auscultation pulmonaire : rhonchies expiratoires éparses, expiration allongée et entrée d'air légèrement diminuée dans les deux champs pulmonaires. Auscultation cardiaque : pas de souffle. Il est décidé de garder le patient en observation à l'hôpital pendant quelques heures. Quelle est, selon vous, l'attitude la plus appropriée à ce moment-là en ce qui concerne les examens complémentaires ?", "full_answer": "Le patient présente probablement une bronchiolite. À ce stade, il n'est pas nécessaire de procéder à des examens complémentaires, sauf en cas d'aggravation clinique.", "type": "PÉDIATRIE", "options": {"1": "Demander les gaz du sang veineux, la numération des globules blancs et les réactifs de phase aiguë.", "2": "Demander une radiographie pulmonaire.", "3": "Commander les gaz du sang artériel et les réactifs de phase aiguë.", "4": "Ne demandez pas de tests complémentaires.", "5": null}, "correct_option": 4, "explanations": {"1": {"exist": false, "char_ranges": [], "word_ranges": [], "text": ""}, "2": {"exist": false, "char_ranges": [], "word_ranges": [], "text": ""}, "3": {"exist": false, "char_ranges": [], "word_ranges": [], "text": ""}, "4": {"exist": true, "char_ranges": [[0, 165]], "word_ranges": [[0, 24]], "text": "Le patient présente probablement une bronchiolite. À ce stade, il n'est pas nécessaire de procéder à des examens complémentaires, sauf en cas d'aggravation clinique."}, "5": {"exist": false, "char_ranges": [], "word_ranges": [], "text": ""}}} +{"id": 71, "year": 2012, "question_id_specific": 50, "full_question": "Un patient ayant des antécédents de maladie fébrile et de douleur thoracique se présente à l'hôpital avec une dyspnée et une tachypnée. À l'examen physique, la tension artérielle est basse, la pression veineuse jugulaire est élevée avec un sinus descendant profond X et il a un pouls paradoxal. Quelle pathologie doit-on suspecter ?", "full_answer": "Une tension artérielle basse avec une pression jugulaire élevée doit toujours faire suspecter une tamponnade cardiaque. Une fièvre et une douleur thoracique accompagnées d'une dyspnée et d'une tachypnée doivent faire suspecter un épanchement péricardique accompagné d'un trouble hémodynamique. Le pouls paradoxal est un signe typique de tamponnade cardiaque. La réponse correcte est donc 5.", "type": "ANESTHÉSIOLOGIE, SOINS INTENSIFS ET MÉDECINE D'URGENCE", "options": {"1": "Maladie cardiaque ischémique.", "2": "Cardiomyopathie dilatée.", "3": "Sténose aortique sévère.", "4": "Péricardite constrictive.", "5": "Épanchement péricardique avec tamponnade cardiaque."}, "correct_option": 5, "explanations": {"1": {"exist": false, "char_ranges": [], "word_ranges": [], "text": ""}, "2": {"exist": false, "char_ranges": [], "word_ranges": [], "text": ""}, "3": {"exist": false, "char_ranges": [], "word_ranges": [], "text": ""}, "4": {"exist": false, "char_ranges": [], "word_ranges": [], "text": ""}, "5": {"exist": true, "char_ranges": [[0, 358]], "word_ranges": [[0, 48]], "text": "Une tension artérielle basse avec une pression jugulaire élevée doit toujours faire suspecter une tamponnade cardiaque. Une fièvre et une douleur thoracique accompagnées d'une dyspnée et d'une tachypnée doivent faire suspecter un épanchement péricardique accompagné d'un trouble hémodynamique. Le pouls paradoxal est un signe typique de tamponnade cardiaque."}}} +{"id": 486, "year": 2020, "question_id_specific": 45, "full_question": "Une jeune femme de 15 ans présentant un retard de la ménarche et une petite taille n'a pas de déficience intellectuelle. Lequel des tests génétiques suivants serait systématiquement utilisé pour diagnostiquer cette patiente ?", "full_answer": "Cette question est peut-être un peu plus difficile, car le cas clinique qui nous est présenté est très vague. Une adolescente avec un retard de puberté et une petite taille, rien de plus. Avec ces informations, elle pourrait avoir plusieurs pathologies d'origine génétique, mais il s'agit du MIR et on nous dit aussi que la patiente n'a pas de déficience intellectuelle. Dans le MIR, un adolescent présentant un retard pubertaire et une petite taille doit être un Turner jusqu'à preuve du contraire (il y a eu deux questions en 2017). Par conséquent, s'il s'agit de notre première suspicion diagnostique, le test HABITUEL (comme le dit l'énoncé) pour le diagnostic devrait être un caryotype conventionnel (formule : 45, X0). La maladie de Turner peut également être diagnostiquée à l'aide d'un FISH (il n'y aurait qu'un seul signal pour l'X au lieu des deux que les femmes ont habituellement) ou d'un array. Cependant, l'array et le NGS sont généralement réservés aux patients présentant des tableaux cliniques non évidents, une déficience intellectuelle ou une suspicion de syndrome génétique nécessitant ces tests spécifiques (par exemple, un array pour diagnostiquer le syndrome de délétion 22q11 ou un panel NGS pour diagnostiquer le syndrome de Noonan).", "type": "GENETIQUE", "options": {"1": "Séquençage en masse (NGS).", "2": "POISSONS.", "3": "Microréseaux d'ADN et/ou d'ARN.", "4": "Caryotype.", "5": null}, "correct_option": 4, "explanations": {"1": {"exist": true, "char_ranges": [[919, 1127]], "word_ranges": [[151, 179]], "text": "l'array et le NGS sont généralement réservés aux patients présentant des tableaux cliniques non évidents, une déficience intellectuelle ou une suspicion de syndrome génétique nécessitant ces tests spécifiques"}, "2": {"exist": true, "char_ranges": [[725, 892]], "word_ranges": [[118, 147]], "text": "La maladie de Turner peut également être diagnostiquée à l'aide d'un FISH (il n'y aurait qu'un seul signal pour l'X au lieu des deux que les femmes ont habituellement)"}, "3": {"exist": true, "char_ranges": [[919, 1127]], "word_ranges": [[151, 179]], "text": "l'array et le NGS sont généralement réservés aux patients présentant des tableaux cliniques non évidents, une déficience intellectuelle ou une suspicion de syndrome génétique nécessitant ces tests spécifiques"}, "4": {"exist": true, "char_ranges": [[384, 724]], "word_ranges": [[64, 118]], "text": "un adolescent présentant un retard pubertaire et une petite taille doit être un Turner jusqu'à preuve du contraire (il y a eu deux questions en 2017). Par conséquent, s'il s'agit de notre première suspicion diagnostique, le test HABITUEL (comme le dit l'énoncé) pour le diagnostic devrait être un caryotype conventionnel (formule : 45, X0)."}, "5": {"exist": false, "char_ranges": [], "word_ranges": [], "text": ""}}} +{"id": 216, "year": 2014, "question_id_specific": 121, "full_question": "Un homme de 38 ans consulte pour dyspnée et hémoptysie. Les analyses sanguines montrent une créatinine à 7 mg/dl, une urée à 250 mg/dl et un titre élevé d'anticorps anti-MBG (anticorps anti-membrane basale glomérulaire). Une biopsie rénale a montré des croissants dans 75 % des glomérules et l'immunofluorescence a montré un dépôt linéaire d'Ig. Laquelle des réponses suivantes est la bonne ?", "full_answer": "La suspicion diagnostique basée sur les données fournies (en particulier les anticorps anti-GBM et la présence d'un syndrome rénopulmonaire) est celle d'un syndrome de Goodpasture. Le reste des données (séminules dans la biopsie, dépôt linéaire d'IgG) soutient le diagnostic. Sachant cela, les options sont facilement écartées : l'option 1 parle d'une néphropathie à IgA (ce n'est pas le cas car les dépôts sont des IgG) ; l'option 3 est également écartée, car il ne s'agit pas d'une GMN primitive, comme la GMN membraneuse, mais d'une glomérulopathie secondaire ; l'option 4 est également fausse : le traitement initial consiste en l'administration de corticostéroïdes et de cyclophosphamide associés à une plasmaphérèse ; et l'option 5 est également fausse, car les lésions ne sont pas causées par des immunocomplexes circulants, mais par des anticorps déposés dans la membrane basale glomérulaire. L'option 2 est donc vraie : comme indiqué ci-dessus, le traitement consisterait à associer des corticostéroïdes, du cyclophosphamide et des plasmaphérèses.", "type": "NEPHROLOGIE", "options": {"1": "Il s'agit d'une néphropathie à IgA avec insuffisance rénale aiguë.", "2": "Une plasmaphérèse serait indiquée.", "3": "Il s'agit d'une glomérulonéphrite membraneuse.", "4": "Le mycophénolate mofétil est le traitement initial de choix.", "5": "L'atteinte glomérulaire est due à la présence d'immunocomplexes circulants."}, "correct_option": 2, "explanations": {"1": {"exist": true, "char_ranges": [[329, 422]], "word_ranges": [[47, 66]], "text": "l'option 1 parle d'une néphropathie à IgA (ce n'est pas le cas car les dépôts sont des IgG) ;"}, "2": {"exist": true, "char_ranges": [[0, 180]], "word_ranges": [[0, 25]], "text": "La suspicion diagnostique basée sur les données fournies (en particulier les anticorps anti-GBM et la présence d'un syndrome rénopulmonaire) est celle d'un syndrome de Goodpasture."}, "3": {"exist": true, "char_ranges": [[423, 564]], "word_ranges": [[66, 88]], "text": "l'option 3 est également écartée, car il ne s'agit pas d'une GMN primitive, comme la GMN membraneuse, mais d'une glomérulopathie secondaire ;"}, "4": {"exist": true, "char_ranges": [[565, 723]], "word_ranges": [[88, 110]], "text": "l'option 4 est également fausse : le traitement initial consiste en l'administration de corticostéroïdes et de cyclophosphamide associés à une plasmaphérèse ;"}, "5": {"exist": true, "char_ranges": [[727, 900]], "word_ranges": [[111, 137]], "text": "l'option 5 est également fausse, car les lésions ne sont pas causées par des immunocomplexes circulants, mais par des anticorps déposés dans la membrane basale glomérulaire."}}} +{"id": 384, "year": 2016, "question_id_specific": 151, "full_question": "Une femme est venue à la clinique avec sa fille de 3 ans parce qu'elle avait détecté un léger développement des seins trois mois auparavant, sans prendre aucun médicament ni aucun antécédent pertinent. L'examen physique a révélé un stade IV de Tanner, sans croissance des poils pubiens ou axillaires. Les organes génitaux externes sont normaux. L'échographie révèle un petit utérus et la radiologie un âge osseux de 3 ans. Quelle attitude adopter ?", "full_answer": "Il semble que l'on veuille nous présenter une puberté précoce (ou une télarche prématurée), mais on ne fournit aucune donnée analytique et les données échographiques sont ambiguës (nous devrions supposer que par petit utérus, on fait référence à un utérus prépubère, mais on ne fournit aucune donnée sur la taille des ovaires). Nous sommes confrontés au cas d'une fillette de trois ans présentant un développement mammaire avancé, en principe sans cause associée (en principe, elle ne prend pas de médicaments susceptibles d'augmenter le taux d'œstrogènes dans le sang, elle ne semble pas utiliser de crèmes pour le corps ni manger beaucoup de viande de poulet). Si l'on suit le schéma diagnostique en cas de télarche précoce ou de suspicion de puberté précoce, on demande l'âge osseux et l'échographie abdominale (l'OE n'est pas avancé comme cela se produit dans la puberté précoce, et nous supposons que par petit utérus on entend un utérus prépubère) ; d'après les examens complémentaires qui nous sont proposés, il ne semble pas s'agir d'une puberté précoce, sauf sur le plan clinique (Tanner IV). En toute rigueur, sans données hormonales analytiques, il semble que l'on puisse retenir l'option 1, avec nécessité d'une surveillance rapprochée de la jeune fille. Si l'on considère toutes les données ci-dessus comme acquises, on peut écarter l'option 4, qui serait le traitement de la puberté précoce centrale. En ce qui concerne l'option de la mammographie, l'échographie mammaire est utilisée en pédiatrie et, dans ce cas, elle serait indiquée si l'on nous disait qu'il y avait une asymétrie mammaire (l'option 3 est exclue). La biopsie mammaire ne serait indiquée qu'en cas de signes d'alerte.", "type": "PÉDIATRIE", "options": {"1": "Suivi tous les 3-4 mois, car il s'agit d'une affection temporaire qui disparaît souvent d'elle-même.", "2": "Biopsie du sein.", "3": "Mammographie.", "4": "Administration d'analogues de la GnRh.", "5": null}, "correct_option": 1, "explanations": {"1": {"exist": true, "char_ranges": [[663, 1062]], "word_ranges": [[105, 170]], "text": "Si l'on suit le schéma diagnostique en cas de télarche précoce ou de suspicion de puberté précoce, on demande l'âge osseux et l'échographie abdominale (l'OE n'est pas avancé comme cela se produit dans la puberté précoce, et nous supposons que par petit utérus on entend un utérus prépubère) ; d'après les examens complémentaires qui nous sont proposés, il ne semble pas s'agir d'une puberté précoce,"}, "2": {"exist": true, "char_ranges": [[1632, 1700]], "word_ranges": [[259, 270]], "text": "La biopsie mammaire ne serait indiquée qu'en cas de signes d'alerte."}, "3": {"exist": true, "char_ranges": [[1415, 1631]], "word_ranges": [[224, 259]], "text": "En ce qui concerne l'option de la mammographie, l'échographie mammaire est utilisée en pédiatrie et, dans ce cas, elle serait indiquée si l'on nous disait qu'il y avait une asymétrie mammaire (l'option 3 est exclue)."}, "4": {"exist": true, "char_ranges": [[663, 1062]], "word_ranges": [[105, 170]], "text": "Si l'on suit le schéma diagnostique en cas de télarche précoce ou de suspicion de puberté précoce, on demande l'âge osseux et l'échographie abdominale (l'OE n'est pas avancé comme cela se produit dans la puberté précoce, et nous supposons que par petit utérus on entend un utérus prépubère) ; d'après les examens complémentaires qui nous sont proposés, il ne semble pas s'agir d'une puberté précoce,"}, "5": {"exist": false, "char_ranges": [], "word_ranges": [], "text": ""}}} +{"id": 238, "year": 2014, "question_id_specific": 145, "full_question": "Une femme de 40 ans a consulté pour une vingtaine d'épisodes par jour de douleur périoculaire gauche intense durant 15 minutes, accompagnée d'un larmoiement intense et d'une rhinorrhée. L'examen et l'IRM sont normaux. Le traitement de choix est le suivant :", "full_answer": "Dans cette question, on nous présente un cas afin d'en déduire un diagnostic et d'indiquer ensuite le traitement de choix. Les caractéristiques du cas et les réponses suggèrent qu'il s'agit d'une céphalée trigémino-autonomique (douleur intense et périoculaire avec larmoiement et rhinorrhée). L'idée est de faire un diagnostic différentiel principalement entre une céphalée en grappe (dont les traitements incluent les réponses 2, 3, 4 et 5), une hémicranie paroxystique (dont le traitement de choix est l'indométhacine de la réponse 1) et un SUNCT (céphalée névralgiforme unilatérale avec injection conjonctivale et larmoiement). La céphalée en grappe prédomine chez les hommes et sa durée peut varier entre 15 et 180 minutes, entre une fois tous les 2 jours et jusqu'à 8 fois par jour. L'hémicranie paroxystique prédomine chez les femmes, avec des épisodes de douleur similaires aux céphalées en grappe, mais d'une durée plus courte (2-30 minutes) et d'une fréquence plus élevée (5-30 épisodes par jour). Dans le cas du syndrome SUNCT, les crises sont beaucoup plus courtes, elles durent quelques secondes (5-240 secondes) et sont généralement réfractaires au traitement. Par conséquent, le diagnostic serait l'hémicranie paroxystique et le traitement de choix serait l'indométhacine (qui est également un critère de diagnostic). La réponse correcte est donc 1 (Indométhacine). Toutes les données fournies sont basées sur les critères de diagnostic de l'International Headache Society.", "type": "NEUROLOGIE", "options": {"1": "Indométhacine.", "2": "Lamotrigine.", "3": "Verapamil.", "4": "Prednisone.", "5": "Carbonate de lithium."}, "correct_option": 1, "explanations": {"1": {"exist": true, "char_ranges": [[788, 1331]], "word_ranges": [[121, 198]], "text": "L'hémicranie paroxystique prédomine chez les femmes, avec des épisodes de douleur similaires aux céphalées en grappe, mais d'une durée plus courte (2-30 minutes) et d'une fréquence plus élevée (5-30 épisodes par jour). Dans le cas du syndrome SUNCT, les crises sont beaucoup plus courtes, elles durent quelques secondes (5-240 secondes) et sont généralement réfractaires au traitement. Par conséquent, le diagnostic serait l'hémicranie paroxystique et le traitement de choix serait l'indométhacine (qui est également un critère de diagnostic)."}, "2": {"exist": true, "char_ranges": [[631, 787]], "word_ranges": [[90, 121]], "text": "La céphalée en grappe prédomine chez les hommes et sa durée peut varier entre 15 et 180 minutes, entre une fois tous les 2 jours et jusqu'à 8 fois par jour."}, "3": {"exist": true, "char_ranges": [[631, 787]], "word_ranges": [[90, 121]], "text": "La céphalée en grappe prédomine chez les hommes et sa durée peut varier entre 15 et 180 minutes, entre une fois tous les 2 jours et jusqu'à 8 fois par jour."}, "4": {"exist": true, "char_ranges": [[631, 787]], "word_ranges": [[90, 121]], "text": "La céphalée en grappe prédomine chez les hommes et sa durée peut varier entre 15 et 180 minutes, entre une fois tous les 2 jours et jusqu'à 8 fois par jour."}, "5": {"exist": true, "char_ranges": [[631, 787]], "word_ranges": [[90, 121]], "text": "La céphalée en grappe prédomine chez les hommes et sa durée peut varier entre 15 et 180 minutes, entre une fois tous les 2 jours et jusqu'à 8 fois par jour."}}} +{"id": 47, "year": 2011, "question_id_specific": 157, "full_question": "Un garçon de 6 ans vient à la clinique accompagné du moniteur d'un centre d'accueil de notre quartier en raison d'une tumeur douloureuse de 3 cm de diamètre à la palpation dans la région occipitale droite du cuir chevelu. Il présente une alopécie dans cette zone et 3 grosseurs de consistance assez dure dans la région cervicale postérieure droite. Quel serait le traitement le plus approprié ?", "full_answer": "La bonne réponse est 3. Si je ne me trompe pas, vous décrivez un kérion de Celso pour lequel le traitement de choix est la griséofulvine orale.", "type": "PÉDIATRIE", "options": {"1": "Incision et drainage.", "2": "Mupirocine topique.", "3": "La griséofulvine par voie orale.", "4": "Céphazoline intraveineuse.", "5": "Kétoconazole topique."}, "correct_option": 3, "explanations": {"1": {"exist": false, "char_ranges": [], "word_ranges": [], "text": ""}, "2": {"exist": false, "char_ranges": [], "word_ranges": [], "text": ""}, "3": {"exist": true, "char_ranges": [[24, 143]], "word_ranges": [[5, 27]], "text": "Si je ne me trompe pas, vous décrivez un kérion de Celso pour lequel le traitement de choix est la griséofulvine orale."}, "4": {"exist": false, "char_ranges": [], "word_ranges": [], "text": ""}, "5": {"exist": false, "char_ranges": [], "word_ranges": [], "text": ""}}} +{"id": 381, "year": 2016, "question_id_specific": 139, "full_question": "Une femme de 70 ans ayant des antécédents d'anorexie, de perte de poids, de gêne au niveau de la musculature et des articulations proximales et de douleur dans la région temporo-mandibulaire se présente aux urgences pour une perte unilatérale de la vision (mouvement de la main), d'apparition soudaine et indolore (anomalie pupillaire afférente). Quel examen demanderiez-vous en premier lieu à des fins de diagnostic ?", "full_answer": "Nous sommes en présence d'un cas d'amaurose monoculaire avec un profil vasculaire, également chez une femme âgée avec des antécédents de perte de poids et ce qui semble être des symptômes de claudication mandibulaire et de polymyalgie rhumatismale. La première cause à envisager est donc l'artérite à cellules géantes comme cause de l'AIN, de sorte que, parmi les options, la bonne serait l'option 2. L'option 4 serait envisagée si l'on nous parlait d'une NIA mais avec des caractéristiques non artéritiques (sans tous les symptômes qui l'accompagnent).", "type": "NEUROLOGIE", "options": {"1": "Ponction lombaire.", "2": "Protéine C-Réactive.", "3": "Angiographie par résonance magnétique.", "4": "Échographie carotidienne.", "5": null}, "correct_option": 2, "explanations": {"1": {"exist": false, "char_ranges": [], "word_ranges": [], "text": ""}, "2": {"exist": true, "char_ranges": [[249, 400]], "word_ranges": [[38, 64]], "text": "La première cause à envisager est donc l'artérite à cellules géantes comme cause de l'AIN, de sorte que, parmi les options, la bonne serait l'option 2."}, "3": {"exist": false, "char_ranges": [], "word_ranges": [], "text": ""}, "4": {"exist": true, "char_ranges": [[401, 553]], "word_ranges": [[64, 86]], "text": "L'option 4 serait envisagée si l'on nous parlait d'une NIA mais avec des caractéristiques non artéritiques (sans tous les symptômes qui l'accompagnent)."}, "5": {"exist": false, "char_ranges": [], "word_ranges": [], "text": ""}}} +{"id": 32, "year": 2011, "question_id_specific": 64, "full_question": "Chez un patient dont l'épilepsie a été diagnostiquée et qui présente des épisodes d'insensibilité aux stimuli externes, des mouvements irréguliers des quatre membres, des yeux fermés, des pleurs et des mouvements pelviens, d'une durée de cinq à vingt secondes et ne répondant pas au traitement par des médicaments antiépileptiques, quelle est l'étude complémentaire la plus susceptible de clarifier le diagnostic ?", "full_answer": "Réponse correcte 1 : Le tableau décrit est très évocateur d'une pseudocrisie avec des mouvements asynchrones des membres, des mouvements pelviens, des pleurs et une faible réponse aux médicaments antiépileptiques.", "type": "NEUROLOGIE ET NEUROCHIRURGIE", "options": {"1": "Surveillance vidéo-EEG pour le diagnostic des pseudo-crises (crises psychogènes).", "2": "Holter ECG pour le diagnostic des maladies cardiaques arythmiques.", "3": "EEG de routine pour le diagnostic du type d'épilepsie (généralisée ou focale).", "4": "IRM cérébrale pour détecter les lésions épileptogènes (dysplasie corticale, tumeur, sclérose temporale médiane).", "5": "Détermination de la glycémie capillaire pour le diagnostic de l'hypoglycémie."}, "correct_option": 1, "explanations": {"1": {"exist": true, "char_ranges": [[21, 213]], "word_ranges": [[4, 30]], "text": "Le tableau décrit est très évocateur d'une pseudocrisie avec des mouvements asynchrones des membres, des mouvements pelviens, des pleurs et une faible réponse aux médicaments antiépileptiques."}, "2": {"exist": false, "char_ranges": [], "word_ranges": [], "text": ""}, "3": {"exist": false, "char_ranges": [], "word_ranges": [], "text": ""}, "4": {"exist": false, "char_ranges": [], "word_ranges": [], "text": ""}, "5": {"exist": false, "char_ranges": [], "word_ranges": [], "text": ""}}} +{"id": 106, "year": 2012, "question_id_specific": 135, "full_question": "Un garçon de 2 ans. Ses antécédents personnels comprennent : 3 épisodes d'otite moyenne aiguë, 1 méningite à méningocoques et 2 pneumonies (une du lobe moyen et une du lobe supérieur gauche). Elle a été admise à trois reprises pour un purpura thrombocytopénique (à trois reprises, les anticorps antiplaquettaires étaient négatifs et la moelle osseuse montrait des mégacaryocytes normaux). Plusieurs hommes de la famille maternelle sont décédés dans l'enfance à la suite de processus infectieux. L'examen physique a révélé des lésions typiques de la dermatite atopique. L'étude immunologique a montré une légère diminution des sous-populations de lymphocytes T, une élévation des IgA et des IgE, une diminution des IgM et des IgG à la limite inférieure de la normale. Quel est le diagnostic le plus probable ?", "full_answer": "Pour répondre correctement à cette question, il convient de noter que les symptômes suivants font partie des symptômes décrits : - Enfant de 2 ans. - Infections ORL. - Infections pulmonaires. - Hospitalisations. - PTI. - Antécédents familiaux d'infections avec décès dus à des infections chez les hommes (famille maternelle). - Dermatite atopique. Toutes ces données, ainsi que les données analytiques, décrivent un déficit immunitaire qui, en raison des antécédents familiaux et maternels, semble être lié au chromosome X, car plusieurs hommes sont décédés d'un état clinique similaire (le père du patient a apporté le chromosome Y, la mère le chromosome X). Parmi les immunodéficiences liées à l'X figure le syndrome de Wiskott-Aldrich, une entité décrite par une triade initiale de symptômes consistant en des saignements (typiques MAIS absents dans le cas présent : saignements abondants après la circoncision, diarrhée sanglante), des infections récurrentes et de l'eczéma. Outre la thrombocytopénie, ils présentent un risque accru de phénomènes auto-immuns et de néoplasmes lymphoïdes. L'option Hyper-IgE manque de données cliniques très caractéristiques telles que des altérations osseuses et des lésions cutanées, qui ne sont pas des dermatites atopiques, car elles suivent un schéma différent (éruption papulo-pustuleuse sur le visage et le cuir chevelu). L'option de l'hypogammaglobulinémie transitoire du nourrisson et le déficit immunitaire combiné sévère et variable commun échouent, entre autres caractéristiques, à la détermination des immunoglobulines G et M, qui sont à la limite inférieure, mais dans les limites de la normalité.", "type": "GÉNÉTIQUE ET IMMUNOLOGIE", "options": {"1": "Syndrome de Wiskott-Aldrich.", "2": "Syndrome d'hyper IgE.", "3": "Hypogammaglobulinémie transitoire de l'enfance.", "4": "Déficit immunitaire combiné sévère lié au chromosome X.", "5": "Déficit immunitaire commun variable."}, "correct_option": 4, "explanations": {"1": {"exist": true, "char_ranges": [[660, 978]], "word_ranges": [[102, 147]], "text": "Parmi les immunodéficiences liées à l'X figure le syndrome de Wiskott-Aldrich, une entité décrite par une triade initiale de symptômes consistant en des saignements (typiques MAIS absents dans le cas présent : saignements abondants après la circoncision, diarrhée sanglante), des infections récurrentes et de l'eczéma."}, "2": {"exist": true, "char_ranges": [[1092, 1364]], "word_ranges": [[162, 201]], "text": "L'option Hyper-IgE manque de données cliniques très caractéristiques telles que des altérations osseuses et des lésions cutanées, qui ne sont pas des dermatites atopiques, car elles suivent un schéma différent (éruption papulo-pustuleuse sur le visage et le cuir chevelu)."}, "3": {"exist": true, "char_ranges": [[1365, 1647]], "word_ranges": [[201, 241]], "text": "L'option de l'hypogammaglobulinémie transitoire du nourrisson et le déficit immunitaire combiné sévère et variable commun échouent, entre autres caractéristiques, à la détermination des immunoglobulines G et M, qui sont à la limite inférieure, mais dans les limites de la normalité."}, "4": {"exist": true, "char_ranges": [[403, 659]], "word_ranges": [[61, 102]], "text": "décrivent un déficit immunitaire qui, en raison des antécédents familiaux et maternels, semble être lié au chromosome X, car plusieurs hommes sont décédés d'un état clinique similaire (le père du patient a apporté le chromosome Y, la mère le chromosome X)."}, "5": {"exist": false, "char_ranges": [], "word_ranges": [], "text": ""}}} +{"id": 62, "year": 2011, "question_id_specific": 119, "full_question": "Nous avons été consultés pour évaluer une femme de 83 ans admise au service de traumatologie pour une fracture de la hanche il y a 6 heures. Elle souffre d'hypertension, de lombalgie, de démence modérée et vit dans une maison de retraite. Son traitement habituel est le thiazide, l'atorvastatine, le donépézil, le calcium et la vitamine D. FE : Le patient est confus, son pouls est de 90 bpm, sa fréquence respiratoire est de 20 rpm, sa tension artérielle est de 170/88, sa pression veineuse jugulaire est normale. L'hémogramme et la radiographie pulmonaire sont normaux et l'ECG montre un rythme sinusal sans altération ischémique. Lequel des éléments suivants est l'approche thérapeutique la plus correcte ?", "full_answer": "Le tableau confusionnel est déterminé par la démence existante du patient, de sorte que nous aurons beau retarder l'intervention chirurgicale, nous n'obtiendrons aucun résultat. L'augmentation de la pression artérielle est d'abord due à la douleur (la première option est donc un analgésique), puis à la situation de stress qui conduit une patiente déjà hypertendue à augmenter sa pression artérielle, de sorte que le labétalol pourrait nous aider. La dernière réponse n'est pas correcte car elle dépend du type de fracture, ce qui n'est pas précisé dans l'énoncé : une fracture pertrochantérienne nécessiterait une ostéosynthèse fermée et une fracture sous-capitale nécessiterait une prothèse.", "type": "ANESTHÉSIOLOGIE ET SOINS INTENSIFS", "options": {"1": "Retarder l'intervention chirurgicale jusqu'à ce que le tableau confusionnel ait disparu.", "2": "Retarder l'intervention chirurgicale et effectuer un échocardiogramme.", "3": "Retarder l'intervention chirurgicale jusqu'à ce que la tension artérielle soit bien contrôlée.", "4": "Mettre en place un bêta-bloquant et initier une intervention chirurgicale.", "5": "Effectuer une ostéosynthèse fermée, en évitant dans tous les cas l'implantation de prothèses."}, "correct_option": 4, "explanations": {"1": {"exist": true, "char_ranges": [[0, 177]], "word_ranges": [[0, 24]], "text": "Le tableau confusionnel est déterminé par la démence existante du patient, de sorte que nous aurons beau retarder l'intervention chirurgicale, nous n'obtiendrons aucun résultat."}, "2": {"exist": false, "char_ranges": [], "word_ranges": [], "text": ""}, "3": {"exist": true, "char_ranges": [[178, 448]], "word_ranges": [[24, 67]], "text": "L'augmentation de la pression artérielle est d'abord due à la douleur (la première option est donc un analgésique), puis à la situation de stress qui conduit une patiente déjà hypertendue à augmenter sa pression artérielle, de sorte que le labétalol pourrait nous aider."}, "4": {"exist": false, "char_ranges": [], "word_ranges": [], "text": ""}, "5": {"exist": true, "char_ranges": [[449, 694]], "word_ranges": [[67, 102]], "text": "La dernière réponse n'est pas correcte car elle dépend du type de fracture, ce qui n'est pas précisé dans l'énoncé : une fracture pertrochantérienne nécessiterait une ostéosynthèse fermée et une fracture sous-capitale nécessiterait une prothèse."}}} +{"id": 592, "year": 2022, "question_id_specific": 76, "full_question": "Une femme de 40 ans consulte parce qu'elle a remarqué une grosseur dans le quadrant supéro-externe du sein droit depuis un mois. Elle présente un rapport de mammographie décrivant une lésion de type BIRADS 3. Quelle est la meilleure conduite à tenir ?", "full_answer": "BI-RADS Breast Imaging Reporting and Data System (système de rapports et de données sur l'imagerie mammaire). BI-RADS 3 est défini par la réponse 3.", "type": "ONCOLOGIE", "options": {"1": "Rassurez-le, car un examen d'imagerie a déjà été effectué et une tumeur maligne a été exclue.", "2": "Cette classification implique probablement une intervention chirurgicale étant donné que la probabilité de cancer est supérieure à 10 %. Il le lui explique et l'oriente préférentiellement vers l'unité du sein.", "3": "Il s'agit probablement d'une découverte bénigne, le risque de cancer étant inférieur à 2 %. Il explique que cela nécessite un suivi tous les 6 à 12 mois jusqu'à 24 mois ou une biopsie.", "4": "Les résultats sont peu suspects de cancer (2-10%), mais une biopsie est nécessaire.", "5": null}, "correct_option": 3, "explanations": {"1": {"exist": false, "char_ranges": [], "word_ranges": [], "text": ""}, "2": {"exist": false, "char_ranges": [], "word_ranges": [], "text": ""}, "3": {"exist": true, "char_ranges": [[0, 148]], "word_ranges": [[0, 24]], "text": "BI-RADS Breast Imaging Reporting and Data System (système de rapports et de données sur l'imagerie mammaire). BI-RADS 3 est défini par la réponse 3."}, "4": {"exist": false, "char_ranges": [], "word_ranges": [], "text": ""}, "5": {"exist": false, "char_ranges": [], "word_ranges": [], "text": ""}}} +{"id": 26, "year": 2011, "question_id_specific": 117, "full_question": "Un patient immunodéficient présentant une pneumonie à la radiographie pulmonaire ou au scanner avec un halo méniscal ou un contour croissant suggère une infection par.. :", "full_answer": "Le signe du halo est caractéristique de l'aspergillose pulmonaire, et plus encore chez un patient immuodéprimé. Mais il n'est pas pathognomonique ; il a également été associé à la tuberculose, à certains néoplasmes et à la granulomatose de Wegener.", "type": "INFECTIEUX", "options": {"1": "Staphylococcus aureus.", "2": "Streptococcus pneumoniae.", "3": "Candida albicans.", "4": "Pseudomonas aeruginosa.", "5": "Aspergillus fumigatus."}, "correct_option": 5, "explanations": {"1": {"exist": false, "char_ranges": [], "word_ranges": [], "text": ""}, "2": {"exist": false, "char_ranges": [], "word_ranges": [], "text": ""}, "3": {"exist": false, "char_ranges": [], "word_ranges": [], "text": ""}, "4": {"exist": false, "char_ranges": [], "word_ranges": [], "text": ""}, "5": {"exist": true, "char_ranges": [[0, 111]], "word_ranges": [[0, 16]], "text": "Le signe du halo est caractéristique de l'aspergillose pulmonaire, et plus encore chez un patient immuodéprimé."}}} +{"id": 435, "year": 2018, "question_id_specific": 121, "full_question": "Un homme de 45 ans consulte pour une toux productive, une douleur pleurétique au flanc droit et de la fièvre depuis 48 heures. Une saturation en O2 de 88 % et des râles à la base droite ont été observés. La radiographie du thorax a montré une consolidation de la base droite. Il a des antécédents d'infection par le VIH bien contrôlée par des médicaments antirétroviraux (lymphocytes CD4 550 ce/uL et charge virale VIH indétectable). Parmi les traitements antimicrobiens empiriques suivants, lequel vous semble le plus approprié ?", "full_answer": "La première option semble être la bonne, en raison de la faible probabilité d'infection par P. jirovecii chez les patients ayant plus de 500 lymphocytes CD4 et une charge virale indétectable. Il est plus probable qu'il s'agisse d'une infection pneumococcique que nous couvrons avec la ceftriaxone et avec l'azithromycine pour les infections dites \"atypiques\". Le méropénem est un antibiotique à trop large spectre, qui pourrait être utilisé en cas de pneumonie hospitalière due à P. aeruginosa, un agent bactérien qui peut également provoquer une pneumonie chez les patients séropositifs, mais cette situation n'est pas courante.", "type": "MALADIES INFECTIEUSES ET MICROBIOLOGIE", "options": {"1": "Cefiriaxone 2 g et azithromycine 500 mg toutes les 24 heures.", "2": "Cefiriaxone 2 g, azithornicine 500 mg toutes les 24 heures et triméthoprime-sulfaméthoxazole 5 mg/kg/8 h (sur la base de la dose de triméthoprime).", "3": "Méthyl-prednisolone 40 mg/jour, céfiriaxone 2 g IV 124 h et triméthoprime-sulfaméthoxazole 5 mg/kg/8 h (sur la base de la dose de triméthoprime).", "4": "Méropénem I g/8 h et vancomycine I g/l2 h.", "5": null}, "correct_option": 1, "explanations": {"1": {"exist": true, "char_ranges": [[192, 359]], "word_ranges": [[31, 54]], "text": "Il est plus probable qu'il s'agisse d'une infection pneumococcique que nous couvrons avec la ceftriaxone et avec l'azithromycine pour les infections dites \"atypiques\"."}, "2": {"exist": false, "char_ranges": [], "word_ranges": [], "text": ""}, "3": {"exist": false, "char_ranges": [], "word_ranges": [], "text": ""}, "4": {"exist": true, "char_ranges": [[360, 629]], "word_ranges": [[54, 95]], "text": "Le méropénem est un antibiotique à trop large spectre, qui pourrait être utilisé en cas de pneumonie hospitalière due à P. aeruginosa, un agent bactérien qui peut également provoquer une pneumonie chez les patients séropositifs, mais cette situation n'est pas courante."}, "5": {"exist": false, "char_ranges": [], "word_ranges": [], "text": ""}}} +{"id": 474, "year": 2020, "question_id_specific": 142, "full_question": "Une femme de 70 ans, diabétique et hypertendue, fait une chute à son domicile et présente une plaie de 9 cm communiquant avec un foyer de fracture du tibia droit. La radiographie montre une fracture oblique courte du tiers médio-distal du tibia. Il a subi une intervention chirurgicale d'urgence avec nettoyage (Friederich) et mise en place d'un clou endomédullaire en acier. À l'âge de 11 mois, il a présenté une pseudarthrose atrophique du tibia avec suppuration dans la zone de la plaie. Quelle est la meilleure option de traitement immédiat ?", "full_answer": "La première chose à faire est de traiter l'infection et pour cela nous devons retirer tout le matériel d'ostéosynthèse, débrider et administrer une antibiothérapie. Dans ce cas, il est préférable de procéder à une fixation externe afin d'éviter d'avoir tout le matériel autour de la zone affectée. Le point 4 est donc correct et les points 1 et 2 ne le sont pas. Le point 3 serait envisagé une fois le tableau infectieux terminé. TRAITEMENT - Traitement antibiotique suppressif : indiqué chez les patients atteints de Ciemy de type C, il consiste en une antibiothérapie orale prolongée pendant au moins 6 mois pour \"refroidir\" l'exacerbation du tableau clinique. - Traitement curatif : il comprend une première intervention chirurgicale avec débridement agressif de tous les tissus affectés, irrigation abondante, stabilisation avec un fixateur externe si la stabilité est compromise, et remplissage éventuel des cavités avec des substances libérant des antibiotiques. Après des périodes prolongées d'antibiothérapie sélectionnées en fonction des antibiogrammes des cultures, et une fois qu'il est certain que l'infection a été guérie, des techniques spéciales doivent être envisagées pour la reconstruction du défaut osseux et pour obtenir une couverture adéquate des tissus mous. Nous avons affaire à une infection associée à un implant (clou intramédullaire) et à une pseudarthrose de la fracture du tibia (absence d'union après 11 mois). L'infection est comprise par les manifestations cliniques (suppuration, non-union) et les antécédents de diabète et de fracture ouverte. La prise en charge se confond avec celle de l'ostéomyélite chronique. L'antibiothérapie isolée (option 2) n'est indiquée que chez les patients présentant une comorbidité sévère et pour lesquels un traitement chirurgical serait plus agressif que la poursuite de la maladie. La dynamisation des ongles associée à une antibiothérapie à large spectre (option 1) n'est pas non plus indiquée, car (a) elle n'a pas montré de bénéfice dans les pseudarthroses établies et (b) nous avons le même problème qu'avec l'option 1, nous n'éliminons pas le biofilm. L'option 3, discutée, ne serait envisagée qu'en l'absence d'infection associée.", "type": "CHIRURGIE ORTHOPÉDIQUE ET TRAUMATOLOGIE", "options": {"1": "Triple antibiothérapie (gram positif, gram négatif et anaérobie) et nettoyage de la plaie chirurgicale, suppression des verrous distaux pour favoriser la cicatrisation osseuse.", "2": "Attente vigilante et traitement antibiotique à base de quinolones.", "3": "Apport d'un greffon autologue et de facteurs de croissance (BMP 2 et 7) pour stimuler le processus de consolidation osseuse qui se ralentit.", "4": "Retrait du clou, débridement, fixation externe et antibiothérapie en fonction des résultats de la culture.", "5": null}, "correct_option": 4, "explanations": {"1": {"exist": true, "char_ranges": [[165, 362]], "word_ranges": [[24, 63]], "text": "Dans ce cas, il est préférable de procéder à une fixation externe afin d'éviter d'avoir tout le matériel autour de la zone affectée. Le point 4 est donc correct et les points 1 et 2 ne le sont pas."}, "2": {"exist": true, "char_ranges": [[165, 362]], "word_ranges": [[24, 63]], "text": "Dans ce cas, il est préférable de procéder à une fixation externe afin d'éviter d'avoir tout le matériel autour de la zone affectée. Le point 4 est donc correct et les points 1 et 2 ne le sont pas."}, "3": {"exist": true, "char_ranges": [[363, 429]], "word_ranges": [[63, 74]], "text": "Le point 3 serait envisagé une fois le tableau infectieux terminé."}, "4": {"exist": true, "char_ranges": [[165, 362]], "word_ranges": [[24, 63]], "text": "Dans ce cas, il est préférable de procéder à une fixation externe afin d'éviter d'avoir tout le matériel autour de la zone affectée. Le point 4 est donc correct et les points 1 et 2 ne le sont pas."}, "5": {"exist": false, "char_ranges": [], "word_ranges": [], "text": ""}}} +{"id": 17, "year": 2011, "question_id_specific": 137, "full_question": "Une patiente de 14 ans en bon état général a présenté il y a 4 jours une éruption cutanée généralisée très prurigineuse constituée de plaques érythémateuses oedémateuses de 2 à 15 cm de diamètre sans desquamation avec une tendance à acquérir une morphologie annulaire, qui disparaissent individuellement en moins de 24 heures. Les muqueuses sont épargnées. La première impression diagnostique serait :", "full_answer": "Urticaire aiguë : caractérisée par des lésions érythémateuses, oedémateuses, évanescentes, prurigineuses, évanescentes, durant moins de 24 heures, sans desquamation. L'état général est généralement conservé. La forme annulaire est plus fréquente chez les enfants. Dans la rubéole et la toxicodermie, l'état général n'est pas conservé. Dans le choc staphylococcique, l'état général est altéré et les lésions sont vésiculeuses. Les lésions de gale sont de préférence interdigitales, sous forme de papulocoses qui peuvent suivre des trajectoires linéaires.", "type": "DERMATOLOGIE", "options": {"1": "Urticaire.", "2": "Rubéole.", "3": "Toxicoderme.", "4": "Choc toxique staphylococcique.", "5": "La gale."}, "correct_option": 1, "explanations": {"1": {"exist": true, "char_ranges": [[0, 334]], "word_ranges": [[0, 44]], "text": "Urticaire aiguë : caractérisée par des lésions érythémateuses, oedémateuses, évanescentes, prurigineuses, évanescentes, durant moins de 24 heures, sans desquamation. L'état général est généralement conservé. La forme annulaire est plus fréquente chez les enfants. Dans la rubéole et la toxicodermie, l'état général n'est pas conservé."}, "2": {"exist": false, "char_ranges": [], "word_ranges": [], "text": ""}, "3": {"exist": false, "char_ranges": [], "word_ranges": [], "text": ""}, "4": {"exist": false, "char_ranges": [], "word_ranges": [], "text": ""}, "5": {"exist": false, "char_ranges": [], "word_ranges": [], "text": ""}}} +{"id": 425, "year": 2018, "question_id_specific": 92, "full_question": "Une femme de 34 ans a été admise à l'hôpital pour polyurie et polydipsie. Au cours des 24 premières heures d'admission, une diurèse de 8,2 litres a été observée et un test sanguin a montré une glycémie de 96 mg/dL, une natrémie de 148 mEq/L et une osmolalité plasmatique de 309 mOsm/kg avec une osmolalité urinaire de 89 mOsml/kg. Quel test diagnostique doit être effectué ensuite ?", "full_answer": "Nous avons été confrontés à une polyurie. Dans un premier temps, nous avons écarté l'hypothèse d'un diabète sucré (notre patient présentait une glycémie normale de 96 mg/dl). Les données analytiques suggèrent un diabète insipide (osm plasmatique élevé et osm urinaire faible). Il faut alors différencier le diabète insipide central (absence d'ADH) du diabète insipide néphrogénique (l'ADH n'exerce pas son action au niveau rénal). Ceci est possible grâce au test à la vasopressine (administration intraveineuse d'ADH et nouvelle mesure de l'osmolarité urinaire). La bonne réponse est donc l'option 3.", "type": "ENDOCRINOLOGIE", "options": {"1": "Test de perfusion de solution saline hypertonique pour la détermination en série de l'hormone antidiurétique.", "2": "Test de déshydratation (test de Miller).", "3": "Administration de desmopressine avec contrôle en série de l'osmolalité de l'urine.", "4": "Détermination de l'hormone antidiurétique dans le plasma.", "5": null}, "correct_option": 3, "explanations": {"1": {"exist": false, "char_ranges": [], "word_ranges": [], "text": ""}, "2": {"exist": false, "char_ranges": [], "word_ranges": [], "text": ""}, "3": {"exist": true, "char_ranges": [[175, 600]], "word_ranges": [[27, 88]], "text": "Les données analytiques suggèrent un diabète insipide (osm plasmatique élevé et osm urinaire faible). Il faut alors différencier le diabète insipide central (absence d'ADH) du diabète insipide néphrogénique (l'ADH n'exerce pas son action au niveau rénal). Ceci est possible grâce au test à la vasopressine (administration intraveineuse d'ADH et nouvelle mesure de l'osmolarité urinaire). La bonne réponse est donc l'option 3."}, "4": {"exist": false, "char_ranges": [], "word_ranges": [], "text": ""}, "5": {"exist": false, "char_ranges": [], "word_ranges": [], "text": ""}}} +{"id": 8, "year": 2011, "question_id_specific": 232, "full_question": "Une femme de 52 ans consulte pour avoir remarqué une coloration jaunâtre des conjonctives au cours de la semaine précédente. Elle ne signale aucun comportement sexuel à risque ni aucun antécédent épidémiologique de risque d'hépatite virale. Elle ne consomme pas d'alcool ni de médicaments hépatotoxiques. Elle signale un an d'antécédents de prurit généralisé, d'asthénie, de sécheresse buccale et d'absence de larmoiement pour une cause sans rapport avec l'hépatite virale. Le reste de l'anamnèse n'a révélé aucun signe pathologique. L'examen physique a révélé des lésions de grattage, un ictère conjonctival et une hépatomégalie non douloureuse. Le patient a fourni des analyses de sang effectuées dans son entreprise avec les résultats pathologiques suivants : Bilirubine 3 mg/dl, FA 400 IU/ ESR 40mm 1 heure. Indiquez quelle serait la recommandation la plus appropriée pour établir le diagnostic étiologique des symptômes du patient :", "full_answer": "Une image classique de la cirrhose biliaire primitive, le genre qui ne se produit pas dans la vie réelle. Le diagnostic serait pratiquement posé avec l'AMA (1). Avec 2, nous exclurions l'hémochromatose, avec 3, la maladie de Wilson. Avec 4, nous pourrions exclure des maladies rares telles que les malformations des voies biliaires ou la maladie de Caroli, et avec 5, nous pourrions exclure une hépatite virale.", "type": "DIGESTIF", "options": {"1": "Anticorps anti-mitochondriaux.", "2": "Étude du métabolisme du Fe.", "3": "Étude du métabolisme du cuivre.", "4": "Imagerie par résonance magnétique du foie.", "5": "Sérologie des virus B et C."}, "correct_option": 1, "explanations": {"1": {"exist": true, "char_ranges": [[106, 160]], "word_ranges": [[19, 27]], "text": "Le diagnostic serait pratiquement posé avec l'AMA (1)."}, "2": {"exist": true, "char_ranges": [[161, 202]], "word_ranges": [[27, 32]], "text": "Avec 2, nous exclurions l'hémochromatose,"}, "3": {"exist": true, "char_ranges": [[203, 232]], "word_ranges": [[32, 38]], "text": "avec 3, la maladie de Wilson."}, "4": {"exist": true, "char_ranges": [[233, 355]], "word_ranges": [[38, 58]], "text": "Avec 4, nous pourrions exclure des maladies rares telles que les malformations des voies biliaires ou la maladie de Caroli,"}, "5": {"exist": true, "char_ranges": [[360, 411]], "word_ranges": [[59, 67]], "text": "avec 5, nous pourrions exclure une hépatite virale."}}} +{"id": 621, "year": 2022, "question_id_specific": 60, "full_question": "Un patient de 20 ans s'est présenté aux urgences après un accident de vélo avec un traumatisme facial. Un scanner crânien a été réalisé et a montré une fracture du tiers moyen de la face affectant la région orbito-malaire. L'une des complications les plus fréquentes de ce type de fracture est.. :", "full_answer": "Les fractures du tiers moyen de la face dans la région orbito-malaire peuvent affecter le plancher et, dans une moindre mesure, la paroi latérale de l'orbite. En supposant donc que l'on nous demande quelle est l'une des complications les plus fréquentes du plancher de l'orbite, il faut toujours souligner deux complications qui peuvent indiquer un traitement chirurgical, même urgent : la diplopie, due à la dislocation du muscle droit inférieur sur le sinus maxillaire sous-jacent (voire à son emprisonnement) ; et l'énophtalmie, qui peut entraîner d'autres complications associées à moyen et à long terme, comme la pseudo-ptose palpébrale supérieure due à la perte de volume orbitaire (option 4 correcte). L'ankylose temporo-mandibulaire n'est pas envisagée car, bien que relativement proche de la cavité orbitaire, elle ne fait pas partie du complexe orbito-malaire (option 1 écartée). L'atteinte du maxillaire peut entraîner une malocclusion dentaire, mais elle survient généralement dans les fractures situées plus bas que la cavité orbitaire (option 2 écartée). Les fractures naso-ethmoïdales sont incluses dans les fractures de la face moyenne, mais les os du nez sont situés plus en avant du rebord orbital médian, et donc en dehors de l'orbite (option 3 écartée).", "type": "OPHTALMOLOGIE (ECTOPIQUE)", "options": {"1": "Ankylose temporo-mandibulaire.", "2": "Malocclusion dentaire.", "3": "Pseudarthrose naso-ethmoïdale.", "4": "Enophtalmie.", "5": null}, "correct_option": 4, "explanations": {"1": {"exist": true, "char_ranges": [[709, 889]], "word_ranges": [[109, 134]], "text": "L'ankylose temporo-mandibulaire n'est pas envisagée car, bien que relativement proche de la cavité orbitaire, elle ne fait pas partie du complexe orbito-malaire (option 1 écartée)."}, "2": {"exist": true, "char_ranges": [[890, 1068]], "word_ranges": [[134, 159]], "text": "L'atteinte du maxillaire peut entraîner une malocclusion dentaire, mais elle survient généralement dans les fractures situées plus bas que la cavité orbitaire (option 2 écartée)."}, "3": {"exist": true, "char_ranges": [[1069, 1273]], "word_ranges": [[159, 194]], "text": "Les fractures naso-ethmoïdales sont incluses dans les fractures de la face moyenne, mais les os du nez sont situés plus en avant du rebord orbital médian, et donc en dehors de l'orbite (option 3 écartée)."}, "4": {"exist": true, "char_ranges": [[159, 708]], "word_ranges": [[26, 109]], "text": "En supposant donc que l'on nous demande quelle est l'une des complications les plus fréquentes du plancher de l'orbite, il faut toujours souligner deux complications qui peuvent indiquer un traitement chirurgical, même urgent : la diplopie, due à la dislocation du muscle droit inférieur sur le sinus maxillaire sous-jacent (voire à son emprisonnement) ; et l'énophtalmie, qui peut entraîner d'autres complications associées à moyen et à long terme, comme la pseudo-ptose palpébrale supérieure due à la perte de volume orbitaire (option 4 correcte)."}, "5": {"exist": false, "char_ranges": [], "word_ranges": [], "text": ""}}} +{"id": 470, "year": 2020, "question_id_specific": 138, "full_question": "Femme de 73 ans ayant des antécédents d'obésité, de diabète sucré de type 2, d'hypertension et de dyslipidémie. Elle consulte pour une douleur insupportable du genou droit évoluant depuis 5 jours, sans antécédent traumatique. Examen : genou globuleux, varus modéré, extension et flexion limitées par la douleur, douleur médiale diffuse. La radiographie montre des ostéophytes et un léger conflit de l'interligne médial. Quelle serait sa prise en charge initiale ?", "full_answer": "Nous décrivons une crise douloureuse aiguë chez un patient atteint de gonarthrose. Dans cette situation, la première chose à faire est de résoudre la crise douloureuse et d'envisager un traitement conservateur approprié pour cette arthrose (1 exact). L'arthroplastie du genou n'est pas envisagée d'emblée, le point 2 est donc faux. On ne nous parle pas de clinique infectieuse pour suspecter une arthrite qui justifierait un débridement et un lavage, donc 3 est faux. Chez un patient atteint de gonarthrose, il y aura toujours une méniscopathie, elle fait partie des changements dégénératifs. Le kyste de Baker ne nous intéresse qu'en cas de crise douloureuse sévère, dans le cadre du diagnostic différentiel avec une thrombose profonde, et il est évalué avec un écho-Doppler, dans le cas d'une gonarthrose, il n'y a aucun intérêt à détecter un kyste de Baker. La tendinite est diagnostiquée par l'examen et non par l'IRM. Par conséquent, le point 4 est faux.", "type": "CHIRURGIE ORTHOPÉDIQUE ET TRAUMATOLOGIE", "options": {"1": "Explication du diagnostic, repos relatif, paracétamol 1g/8h plus métamizole 500 mg/ 8 h sauvetage naproxen.", "2": "Orientation préférentielle des patients ambulatoires en traumatologie pour l'évaluation des prothèses totales cimentées.", "3": "Orientation préférentielle des patients ambulatoires en traumatologie pour un débridement arthroscopique.", "4": "Demande d'IRM préférentielle pour l'évaluation de la méniscopathie, du kyste de Baker et/ou de la tendinite.", "5": null}, "correct_option": 1, "explanations": {"1": {"exist": true, "char_ranges": [[0, 250]], "word_ranges": [[0, 37]], "text": "Nous décrivons une crise douloureuse aiguë chez un patient atteint de gonarthrose. Dans cette situation, la première chose à faire est de résoudre la crise douloureuse et d'envisager un traitement conservateur approprié pour cette arthrose (1 exact)."}, "2": {"exist": true, "char_ranges": [[251, 331]], "word_ranges": [[37, 50]], "text": "L'arthroplastie du genou n'est pas envisagée d'emblée, le point 2 est donc faux."}, "3": {"exist": true, "char_ranges": [[332, 467]], "word_ranges": [[50, 73]], "text": "On ne nous parle pas de clinique infectieuse pour suspecter une arthrite qui justifierait un débridement et un lavage, donc 3 est faux."}, "4": {"exist": true, "char_ranges": [[593, 959]], "word_ranges": [[91, 154]], "text": "Le kyste de Baker ne nous intéresse qu'en cas de crise douloureuse sévère, dans le cadre du diagnostic différentiel avec une thrombose profonde, et il est évalué avec un écho-Doppler, dans le cas d'une gonarthrose, il n'y a aucun intérêt à détecter un kyste de Baker. La tendinite est diagnostiquée par l'examen et non par l'IRM. Par conséquent, le point 4 est faux."}, "5": {"exist": false, "char_ranges": [], "word_ranges": [], "text": ""}}} +{"id": 298, "year": 2016, "question_id_specific": 103, "full_question": "Un homme de 25 ans, sans antécédents particuliers, se présente aux urgences avec de la fièvre, des céphalées, des myalgies, des nausées, des vomissements, des douleurs abdominales, un ictère et une injection conjonctivale, deux semaines après s'être rendu en Thaïlande pour participer à une régate en eau douce. Quel est le diagnostic le plus probable ?", "full_answer": "Les antécédents de contact avec de l'eau douce dans une zone endémique orientent vers la leptospirose, et dans le tableau clinique, on trouve également une jaunisse et une injection conjonctivale, ce qui oriente également vers la leptospirose et n'est pas aussi caractéristique des trois autres agents pathogènes.", "type": "LES MALADIES INFECTIEUSES", "options": {"1": "Le paludisme.", "2": "Schistosomiase.", "3": "Leptospirose.", "4": "Rabia.", "5": null}, "correct_option": 3, "explanations": {"1": {"exist": true, "char_ranges": [[0, 313]], "word_ranges": [[0, 47]], "text": "Les antécédents de contact avec de l'eau douce dans une zone endémique orientent vers la leptospirose, et dans le tableau clinique, on trouve également une jaunisse et une injection conjonctivale, ce qui oriente également vers la leptospirose et n'est pas aussi caractéristique des trois autres agents pathogènes."}, "2": {"exist": true, "char_ranges": [[0, 313]], "word_ranges": [[0, 47]], "text": "Les antécédents de contact avec de l'eau douce dans une zone endémique orientent vers la leptospirose, et dans le tableau clinique, on trouve également une jaunisse et une injection conjonctivale, ce qui oriente également vers la leptospirose et n'est pas aussi caractéristique des trois autres agents pathogènes."}, "3": {"exist": true, "char_ranges": [[0, 313]], "word_ranges": [[0, 47]], "text": "Les antécédents de contact avec de l'eau douce dans une zone endémique orientent vers la leptospirose, et dans le tableau clinique, on trouve également une jaunisse et une injection conjonctivale, ce qui oriente également vers la leptospirose et n'est pas aussi caractéristique des trois autres agents pathogènes."}, "4": {"exist": true, "char_ranges": [[0, 313]], "word_ranges": [[0, 47]], "text": "Les antécédents de contact avec de l'eau douce dans une zone endémique orientent vers la leptospirose, et dans le tableau clinique, on trouve également une jaunisse et une injection conjonctivale, ce qui oriente également vers la leptospirose et n'est pas aussi caractéristique des trois autres agents pathogènes."}, "5": {"exist": false, "char_ranges": [], "word_ranges": [], "text": ""}}} +{"id": 365, "year": 2016, "question_id_specific": 100, "full_question": "Un homme de 64 ans, agriculteur, ex-fumeur (5 ans), atteint de BPCO et souffrant de polyarthrite rhumatoïde sous traitement corticoïde. Il a consulté le service des urgences pour des céphalées sévères évoluant depuis 2 jours avec déviation de la commissure des lèvres. En toile de fond, il rapporte qu'après un mois de grippe, il persiste avec une toux, une expectoration purulente et parfois hémoptotique, une fièvre fébrile, une anorexie, une asthénie et un amaigrissement. À son arrivée, il présente une fièvre de 38,2 °C, de multiples abcès cutanés sur les mains, le dos et les fesses (certains avec des voies fistuleuses) et une paralysie faciale centrale droite, des infiltrats apicaux avec un petit épanchement pleural associé sur la radiographie du thorax et une leucocytose avec neutrophilie. Parmi les diagnostics suspects suivants, je considérerais qu'ils sont les plus probables :", "full_answer": "Nocardia affecte généralement les personnes immunodéprimées, en particulier celles dont l'immunité à médiation cellulaire est altérée, comme celle produite par les stéroïdes, et peut entraîner une atteinte pulmonaire, des abcès cérébraux et des abcès cutanés. La tuberculose peut affecter les poumons et le cerveau, mais pas les abcès cutanés. L'aspergillus n'affecte pas non plus la peau.", "type": "PNEUMOLOGIE ET CHIRURGIE THORACIQUE", "options": {"1": "Tumeur pulmonaire avec métastases cérébrales.", "2": "Tuberculose disséminée.", "3": "Nocardiose.", "4": "Aspergillose.", "5": null}, "correct_option": 4, "explanations": {"1": {"exist": false, "char_ranges": [], "word_ranges": [], "text": ""}, "2": {"exist": true, "char_ranges": [[260, 343]], "word_ranges": [[35, 49]], "text": "La tuberculose peut affecter les poumons et le cerveau, mais pas les abcès cutanés."}, "3": {"exist": true, "char_ranges": [[0, 259]], "word_ranges": [[0, 35]], "text": "Nocardia affecte généralement les personnes immunodéprimées, en particulier celles dont l'immunité à médiation cellulaire est altérée, comme celle produite par les stéroïdes, et peut entraîner une atteinte pulmonaire, des abcès cérébraux et des abcès cutanés."}, "4": {"exist": true, "char_ranges": [[344, 389]], "word_ranges": [[49, 56]], "text": "L'aspergillus n'affecte pas non plus la peau."}, "5": {"exist": false, "char_ranges": [], "word_ranges": [], "text": ""}}} +{"id": 392, "year": 2016, "question_id_specific": 223, "full_question": "Une femme de 68 ans, ayant des antécédents de deux épisodes dépressifs majeurs dans sa vie, a consulté pour des symptômes de tristesse, de dépression, d'anhédonie, d'asthénie et d'anorexie compatibles avec un nouvel épisode dépressif. On lui a prescrit 10 mg d'escitalopram et elle a été évaluée deux semaines plus tard. Lors de cet examen, la patiente a déclaré se sentir très bien, se réveiller tôt le matin très hyperactive et avec un \"grand désir de faire des choses\", elle a dit qu'elle avait beaucoup d'énergie et qu'elle était plus bavarde que d'habitude. Elle ne se dit pas irritable et peut dormir 6 heures d'affilée. Compte tenu de cette situation, que pensez-vous que la patiente a ?", "full_answer": "Dans cette question, on pourrait soupçonner la possibilité d'un changement affectif après l'introduction du médicament antidépresseur, sur la base de ce que le patient mentionne, mais si nous regardons la capacité du patient à maintenir le sommeil, il y a un fait clair qui \"exclut\" un changement hypo/maniaque. Deux semaines est une courte période pour que le médicament ait un effet complet sur l'humeur, mais il peut y avoir une activation initiale que les patients ne supportent pas toujours très bien (ce qui ne semble pas être le cas). Chez ce patient, il est indiqué de maintenir le traitement et de le réévaluer dans un mois au maximum.", "type": "PSYCHIATRIE", "options": {"1": "Trouble bipolaire de type I.", "2": "Hypomanie induite par les médicaments.", "3": "Réponse normale à l'escitalopram.", "4": "Démence frontale.", "5": null}, "correct_option": 3, "explanations": {"1": {"exist": false, "char_ranges": [], "word_ranges": [], "text": ""}, "2": {"exist": true, "char_ranges": [[202, 311]], "word_ranges": [[29, 48]], "text": "la capacité du patient à maintenir le sommeil, il y a un fait clair qui \"exclut\" un changement hypo/maniaque."}, "3": {"exist": false, "char_ranges": [], "word_ranges": [], "text": ""}, "4": {"exist": false, "char_ranges": [], "word_ranges": [], "text": ""}, "5": {"exist": false, "char_ranges": [], "word_ranges": [], "text": ""}}} +{"id": 285, "year": 2016, "question_id_specific": 59, "full_question": "Une femme de 73 ans a été admise pour une dyspnée progressive jusqu'au repos, une orthopnée et une prise de poids de 4 kg. L'examen physique a révélé une pression artérielle de 150/84 mm Hg, une fréquence cardiaque de 100 battements/minute, une augmentation de la pression veineuse jugulaire, des crépitants dans les deux bases et un œdème malléolaire. Traitement habituel : énalapril 5 mg toutes les 12 heures, furosémide 80 mg par jour. Quel est le traitement le plus approprié à ce stade ?", "full_answer": "Augmenter la dose d'énalapril en fonction de la tolérance et administrer du turosémide par voie intraveineuse.", "type": "CARDIOLOGIE ET CHIRURGIE VASCULAIRE", "options": {"1": "Administrer le fiirosemide par voie intraveineuse.", "2": "Augmenter la dose d'énalapril en fonction de la tolérance et administrer du furosémide par voie intraveineuse.", "3": "Commencez à prendre un bêta-bloquant.", "4": "Ajouter un traitement à l'amlodipine.", "5": null}, "correct_option": 2, "explanations": {"1": {"exist": false, "char_ranges": [], "word_ranges": [], "text": ""}, "2": {"exist": true, "char_ranges": [[0, 110]], "word_ranges": [[0, 16]], "text": "Augmenter la dose d'énalapril en fonction de la tolérance et administrer du turosémide par voie intraveineuse."}, "3": {"exist": false, "char_ranges": [], "word_ranges": [], "text": ""}, "4": {"exist": false, "char_ranges": [], "word_ranges": [], "text": ""}, "5": {"exist": false, "char_ranges": [], "word_ranges": [], "text": ""}}} +{"id": 301, "year": 2016, "question_id_specific": 232, "full_question": "Une femme de 24 ans consulte après avoir constaté une lymphadénopathie inguinale. L'interrogatoire n'a pas révélé de gêne locale ni d'éléments évocateurs d'une infection sexuellement transmissible. L'examen révèle deux adénopathies, une dans chaque aine, de 1 cm de diamètre, molles, mobiles, non douloureuses. Aucune lésion cutanée n'est observée au niveau des membres inférieurs, de l'anus ou du périnée. Quel examen vous semble indispensable ?", "full_answer": "Chez l'adulte en bonne santé, des ganglions lymphatiques inguinaux palpables d'une taille allant jusqu'à 2 centimètres peuvent être présents et considérés comme normaux. Un examen plus approfondi de ces ganglions lymphatiques normaux n'est pas justifié.", "type": "LES MALADIES INFECTIEUSES", "options": {"1": "Une sérologie pour la syphilis, car il s'agit très probablement d'une infection à Treponema pallidum.", "2": "Un examen gynécologique pour écarter la possibilité d'un cancer de l'ovaire.", "3": "D'après les caractéristiques cliniques, il semble que les ganglions lymphatiques soient normaux et qu'il n'y ait pas lieu de procéder à d'autres examens.", "4": "Un test de Paul-Bunnell doit être effectué pour exclure une mononucléose infectieuse.", "5": null}, "correct_option": 3, "explanations": {"1": {"exist": false, "char_ranges": [], "word_ranges": [], "text": ""}, "2": {"exist": false, "char_ranges": [], "word_ranges": [], "text": ""}, "3": {"exist": true, "char_ranges": [[0, 253]], "word_ranges": [[0, 35]], "text": "Chez l'adulte en bonne santé, des ganglions lymphatiques inguinaux palpables d'une taille allant jusqu'à 2 centimètres peuvent être présents et considérés comme normaux. Un examen plus approfondi de ces ganglions lymphatiques normaux n'est pas justifié."}, "4": {"exist": false, "char_ranges": [], "word_ranges": [], "text": ""}, "5": {"exist": false, "char_ranges": [], "word_ranges": [], "text": ""}}} +{"id": 23, "year": 2011, "question_id_specific": 113, "full_question": "Femme de 71 ans ayant des antécédents de polyarthrite rhumatoïde sous traitement par sulfasalazine, prednisone et étanercept. Elle s'est présentée aux urgences pendant 72 heures avec des symptômes cliniques compatibles avec un zona facial touchant le côté droit du visage, le pavillon de l'oreille, respectant le front et un chémosis conjonctival. Quel serait le traitement approprié ?", "full_answer": "Je pense que cette question ne figure pas clairement dans le programme des maladies infectieuses et qu'elle peut faire double emploi avec l'OFT et le DERMA, mais si je comprends bien, chez un patient immunodéprimé et avec des données sur l'atteinte oculaire, l'admission pour un traitement intraveineux serait indiquée en raison du risque élevé de complications possibles.", "type": "INFECTIEUX", "options": {"1": "Traitement symptomatique de la douleur uniquement.", "2": "Traitement topique à l'acyclovir.", "3": "Traitement ambulatoire par acyclovir, valacyclovir ou famciclovir oral.", "4": "Admission à l'hôpital et traitement par acyclovir ou famciclovir iv.", "5": "Ig parentérale et vaccination."}, "correct_option": 4, "explanations": {"1": {"exist": false, "char_ranges": [], "word_ranges": [], "text": ""}, "2": {"exist": false, "char_ranges": [], "word_ranges": [], "text": ""}, "3": {"exist": false, "char_ranges": [], "word_ranges": [], "text": ""}, "4": {"exist": true, "char_ranges": [[184, 372]], "word_ranges": [[31, 57]], "text": "chez un patient immunodéprimé et avec des données sur l'atteinte oculaire, l'admission pour un traitement intraveineux serait indiquée en raison du risque élevé de complications possibles."}, "5": {"exist": false, "char_ranges": [], "word_ranges": [], "text": ""}}} +{"id": 564, "year": 2022, "question_id_specific": 126, "full_question": "Un homme de 58 ans, souffrant d'hypertension depuis 6 ans, consulte pour un mauvais contrôle de sa pression artérielle malgré un traitement par inhibiteur de l'enzyme de conversion de l'angiotensine, diurétique et antagoniste du calcium. Lors de la consultation, il présente une tension artérielle de 149/100 mmHg. Examens de laboratoire : créatinine 1,2 mg/dl, potassium 2,2 mEq/l et alcalose métabolique compensée ; le reste de l'étude biochimique, l'hémogramme, la coagulation et le sédiment urinaire sont normaux. Cochez la bonne réponse :", "full_answer": "On nous présente un patient souffrant d'hypertension artérielle résistante, supposant un hyperaldostéronisme primaire dû à une alcalose métabolique hypokaliémique. Lorsque le diagnostic est confirmé biochimiquement, l'examen suivant à effectuer est une tomodensitométrie pour déterminer le sous-type et exclure la présence d'un carcinome surrénalien (option 3 correcte). Il est donc nécessaire d'attendre l'examen d'imagerie pour confirmer l'étiologie (option 1 incorrecte). La cause la plus fréquente est l'adénome producteur d'aldostérone (option 2 incorrecte). La spironolactone est le traitement médical de choix (option 4 incorrecte).", "type": "NEPHROLOGIE", "options": {"1": "L'origine de l'hypertension dans ce cas est une sécrétion excessive d'aldostérone causée par un hyperfonctionnement autonome de la médullosurrénale.", "2": "Dans la plupart des cas, le substrat anatomique est une hyperplasie bilatérale du cortex surrénalien.", "3": "La tomodensitométrie fait partie du bilan diagnostique en cas de confirmation biochimique.", "4": "La spironolactone est contre-indiquée dans la prise en charge de cette pathologie.", "5": null}, "correct_option": 3, "explanations": {"1": {"exist": true, "char_ranges": [[371, 474]], "word_ranges": [[46, 59]], "text": "Il est donc nécessaire d'attendre l'examen d'imagerie pour confirmer l'étiologie (option 1 incorrecte)."}, "2": {"exist": true, "char_ranges": [[475, 563]], "word_ranges": [[59, 71]], "text": "La cause la plus fréquente est l'adénome producteur d'aldostérone (option 2 incorrecte)."}, "3": {"exist": true, "char_ranges": [[0, 370]], "word_ranges": [[0, 46]], "text": "On nous présente un patient souffrant d'hypertension artérielle résistante, supposant un hyperaldostéronisme primaire dû à une alcalose métabolique hypokaliémique. Lorsque le diagnostic est confirmé biochimiquement, l'examen suivant à effectuer est une tomodensitométrie pour déterminer le sous-type et exclure la présence d'un carcinome surrénalien (option 3 correcte)."}, "4": {"exist": true, "char_ranges": [[564, 639]], "word_ranges": [[71, 82]], "text": "La spironolactone est le traitement médical de choix (option 4 incorrecte)."}, "5": {"exist": false, "char_ranges": [], "word_ranges": [], "text": ""}}} +{"id": 292, "year": 2016, "question_id_specific": 213, "full_question": "Gustavo s'est présenté au service des urgences avec des lésions cutanées et un malaise général évoluant depuis plusieurs jours. Il présentait des lésions psoriasiformes sur le tronc avec une atteinte de la paume des mains et de la plante des pieds. Il présentait également une inflammation articulaire asymétrique non suppurative, une rougeur oculaire bilatérale ainsi que des érosions sur le gland du pénis. Dans l'anamnèse qui suit, Gustavo reconnaît un contact sexuel à risque 20 jours plus tôt. Quel est son diagnostic ?", "full_answer": "Il décrit un syndrome de Reiter classique : kératodermie palmo-plantaire, arthrite et manifestations oculaires, ainsi qu'une urétrite probablement à chlamydia, peut-être asymptomatique (ils n'expliquent pas l'exsudat urétral). Bien entendu, le patient peut également présenter une infection par le VIH ou même une syphilis secondaire (en plus de la maladie de Reiter).", "type": "DERMATOLOGIE, VÉNÉRÉOLOGIE ET CHIRURGIE PLASTIQUE", "options": {"1": "L'infection par le VIH.", "2": "Syphilis secondaire.", "3": "Syndrome de Reiter.", "4": "Erythème polymorphe.", "5": null}, "correct_option": 3, "explanations": {"1": {"exist": false, "char_ranges": [], "word_ranges": [], "text": ""}, "2": {"exist": false, "char_ranges": [], "word_ranges": [], "text": ""}, "3": {"exist": true, "char_ranges": [[0, 226]], "word_ranges": [[0, 27]], "text": "Il décrit un syndrome de Reiter classique : kératodermie palmo-plantaire, arthrite et manifestations oculaires, ainsi qu'une urétrite probablement à chlamydia, peut-être asymptomatique (ils n'expliquent pas l'exsudat urétral)."}, "4": {"exist": false, "char_ranges": [], "word_ranges": [], "text": ""}, "5": {"exist": false, "char_ranges": [], "word_ranges": [], "text": ""}}} +{"id": 477, "year": 2020, "question_id_specific": 128, "full_question": "Un homme de 35 ans a été admis pour un traumatisme thoracique grave avec de multiples fractures de côtes. Après avoir répondu favorablement au traitement par analgésiques et oxygène, il a commencé à présenter une hypoxémie sévère. Indiquez la cause la plus probable de cette détérioration :", "full_answer": "La contusion pulmonaire est la lésion la plus grave et celle dont le pronostic est le plus sombre lors d'un traumatisme thoracique. C'est aussi la lésion qui provoque le plus tôt une hypoxémie. Contrairement à ce que l'on pourrait penser, un thorax instable dû à des fractures multiples des côtes (volet costal) provoquerait une hypoventilation progressive avec atélectasie du parenchyme pulmonaire, qui évoluerait d'une part vers l'hypercapnie et l'acidose respiratoire, et d'autre part vers une hypoxémie ultérieure due à l'infection associée à l'atélectasie. L'infection respiratoire par aspiration est également plus tardive, et n'apparaît pas dans tous les cas de traumatismes thoraciques graves, seulement s'il y a eu une diminution du niveau de conscience (TBI associé, intubation incontrôlée avec bronchoaspiration...). L'hypovolémie post-traumatique (dans ce cas, comme il y a de multiples fractures de côtes, elle pourrait être due à un hémothorax) surviendrait plus tôt et serait associée en premier lieu à une instabilité hémodynamique.", "type": "SOINS INTENSIFS", "options": {"1": "Instabilité de la paroi thoracique due à des fractures multiples.", "2": "Infection respiratoire par aspiration.", "3": "Altération des échanges gazeux due à une contusion pulmonaire.", "4": "Hypovolémie post-traumatique.", "5": null}, "correct_option": 3, "explanations": {"1": {"exist": true, "char_ranges": [[239, 561]], "word_ranges": [[40, 83]], "text": "un thorax instable dû à des fractures multiples des côtes (volet costal) provoquerait une hypoventilation progressive avec atélectasie du parenchyme pulmonaire, qui évoluerait d'une part vers l'hypercapnie et l'acidose respiratoire, et d'autre part vers une hypoxémie ultérieure due à l'infection associée à l'atélectasie."}, "2": {"exist": true, "char_ranges": [[562, 827]], "word_ranges": [[83, 119]], "text": "L'infection respiratoire par aspiration est également plus tardive, et n'apparaît pas dans tous les cas de traumatismes thoraciques graves, seulement s'il y a eu une diminution du niveau de conscience (TBI associé, intubation incontrôlée avec bronchoaspiration...)."}, "3": {"exist": true, "char_ranges": [[0, 193]], "word_ranges": [[0, 33]], "text": "La contusion pulmonaire est la lésion la plus grave et celle dont le pronostic est le plus sombre lors d'un traumatisme thoracique. C'est aussi la lésion qui provoque le plus tôt une hypoxémie."}, "4": {"exist": true, "char_ranges": [[828, 1048]], "word_ranges": [[119, 153]], "text": "L'hypovolémie post-traumatique (dans ce cas, comme il y a de multiples fractures de côtes, elle pourrait être due à un hémothorax) surviendrait plus tôt et serait associée en premier lieu à une instabilité hémodynamique."}, "5": {"exist": false, "char_ranges": [], "word_ranges": [], "text": ""}}} +{"id": 572, "year": 2022, "question_id_specific": 195, "full_question": "Une femme de 66 ans souffre de diabète de type 2. Lors de l'évaluation de sa fonction rénale, elle a présenté un stade G3a/A1. À quelles valeurs correspond ce stade, le plus fréquent chez les patients atteints de néphropathie diabétique ?", "full_answer": "Question facile. Le stade G3a correspond à un débit de filtration compris entre 45 et 59 ml/min. Le stade A1 correspond à une albuminurie inférieure à 30 mg/ml. La bonne option est donc 1.", "type": "NEPHROLOGIE", "options": {"1": "Débit de filtration glomérulaire 45-59 ml/min/1,73 m² et albuminurie <30 mg/ml.", "2": "Débit de filtration glomérulaire 30-44 ml/min/1,73 m² et albuminurie <30 mg/ml.", "3": "Taux de filtration glomérulaire de 45 à 59 ml/min/1,73 m² et albuminurie de 30 à 300 mg/ml.", "4": "Débit de filtration glomérulaire de 15 à 29 ml/min/1,73 m² et albuminurie <30 mg/ml.", "5": null}, "correct_option": 1, "explanations": {"1": {"exist": true, "char_ranges": [[17, 188]], "word_ranges": [[2, 34]], "text": "Le stade G3a correspond à un débit de filtration compris entre 45 et 59 ml/min. Le stade A1 correspond à une albuminurie inférieure à 30 mg/ml. La bonne option est donc 1."}, "2": {"exist": false, "char_ranges": [], "word_ranges": [], "text": ""}, "3": {"exist": false, "char_ranges": [], "word_ranges": [], "text": ""}, "4": {"exist": false, "char_ranges": [], "word_ranges": [], "text": ""}, "5": {"exist": false, "char_ranges": [], "word_ranges": [], "text": ""}}} +{"id": 180, "year": 2013, "question_id_specific": 232, "full_question": "Un patient de 29 ans se présente à votre cabinet avec un diagnostic d'aplasie médullaire sévère. Quel est le traitement de choix ?", "full_answer": "Transfuser ce patient et le bourrer d'antibiotiques, c'est lui donner du pain pour aujourd'hui et de la nourriture pour demain. Nous voulons quelque chose de plus durable. Les androgènes et les transfusions de plaquettes ne règlent pas non plus le problème. Il ne nous reste que les trois autres options. Une greffe autologue n'est pas raisonnable, car la moelle osseuse d'un patient atteint d'aplasie médullaire est inférieure à 25 %, de sorte qu'il est impossible d'en obtenir beaucoup. Par conséquent, nous préférerions une greffe de moelle osseuse allogénique si le patient a un frère ou une sœur HLA-identique, car c'est le traitement de choix selon le protocole de la Société espagnole d'hématologie et d'hémothérapie pour les patients de moins de 40 ans atteints d'aplasie médullaire sévère. Réponse correcte, 3.", "type": "HÉMATOLOGIE", "options": {"1": "Transfusions périodiques et antibiotiques.", "2": "Androgènes et transfusions de plaquettes.", "3": "Greffe de moelle osseuse allogénique si frère ou sœur HLA-identique.", "4": "Greffe autologue de moelle osseuse pour éviter le rejet.", "5": "Cyclosporine A et globuline antithymocyte."}, "correct_option": 3, "explanations": {"1": {"exist": true, "char_ranges": [[0, 127]], "word_ranges": [[0, 20]], "text": "Transfuser ce patient et le bourrer d'antibiotiques, c'est lui donner du pain pour aujourd'hui et de la nourriture pour demain."}, "2": {"exist": true, "char_ranges": [[172, 257]], "word_ranges": [[27, 41]], "text": "Les androgènes et les transfusions de plaquettes ne règlent pas non plus le problème."}, "3": {"exist": true, "char_ranges": [[505, 798]], "word_ranges": [[80, 126]], "text": "nous préférerions une greffe de moelle osseuse allogénique si le patient a un frère ou une sœur HLA-identique, car c'est le traitement de choix selon le protocole de la Société espagnole d'hématologie et d'hémothérapie pour les patients de moins de 40 ans atteints d'aplasie médullaire sévère."}, "4": {"exist": true, "char_ranges": [[305, 488]], "word_ranges": [[50, 78]], "text": "Une greffe autologue n'est pas raisonnable, car la moelle osseuse d'un patient atteint d'aplasie médullaire est inférieure à 25 %, de sorte qu'il est impossible d'en obtenir beaucoup."}, "5": {"exist": false, "char_ranges": [], "word_ranges": [], "text": ""}}} +{"id": 270, "year": 2015, "question_id_specific": 135, "full_question": "Un jeune homme se présente aux urgences pour une brûlure par flamme du deuxième degré sur 10 % de la surface corporelle, affectant le bras droit de façon extensive et circulaire, sans pouls artériel dans la main mesuré par Doppler. Il n'y a pas de pouls artériel dans la main mesuré par Doppler. Quel est le traitement de choix ?", "full_answer": "La question est délicate : il est question d'une brûlure de 10 % de la surface corporelle et le traitement de choix pour les brûlures du second degré de moins de 20 % de la surface corporelle devrait être topique et sous surveillance. Cependant, il y a un signe d'alerte : il n'y a pas de pouls artériel et la brûlure est circonférentielle au membre. Il s'agit d'une urgence qui, si on la laisse évoluer, compromettra la viabilité du membre affecté et conduira à l'amputation : une escarotomie d'urgence doit être pratiquée afin de soulager la pression du troisième espace sur le tronc vasculaire artériel.", "type": "DERMATOLOGIE ET CHIRURGIE PLASTIQUE", "options": {"1": "Cures occlusives de sulfadiazine-arginine et évaluation de la profondeur à une semaine.", "2": "Drainage lymphatique et évaluation d'une dérivation vasculaire.", "3": "Incisions d'escarotomie ou de décompression d'urgence.", "4": "Gestion des attentes.", "5": "Amputation du membre."}, "correct_option": 3, "explanations": {"1": {"exist": false, "char_ranges": [], "word_ranges": [], "text": ""}, "2": {"exist": false, "char_ranges": [], "word_ranges": [], "text": ""}, "3": {"exist": true, "char_ranges": [[246, 606]], "word_ranges": [[44, 105]], "text": "il y a un signe d'alerte : il n'y a pas de pouls artériel et la brûlure est circonférentielle au membre. Il s'agit d'une urgence qui, si on la laisse évoluer, compromettra la viabilité du membre affecté et conduira à l'amputation : une escarotomie d'urgence doit être pratiquée afin de soulager la pression du troisième espace sur le tronc vasculaire artériel."}, "4": {"exist": false, "char_ranges": [], "word_ranges": [], "text": ""}, "5": {"exist": false, "char_ranges": [], "word_ranges": [], "text": ""}}} +{"id": 368, "year": 2016, "question_id_specific": 121, "full_question": "Alors que vous êtes de garde au service des urgences de votre hôpital, vous devez vous occuper d'un patient de 64 ans souffrant d'insuffisance respiratoire aiguë. Son état clinique est critique, avec une faible saturation en oxygène et une instabilité hémodynamique. Une radiographie thoracique urgente a montré une atélectasie des 2/3 du poumon droit. Une intubation orotrachéale et une ventilation assistée ont été réalisées, avec une Fi02 de 1,0. L'analyse ultérieure des gaz du sang artériel montre un pH de 7,23, une Pa02 de 60 mmHg et une PaC02 de 30 mmHg. Quelle est la cause de l'hypoxémie ?", "full_answer": "Une FiO2 de 1 et une hyperventilation par ventilation mécanique, démontrée par une PCO2 basse, ne parviennent pas à augmenter suffisamment la PO2, en raison d'un trouble V/Q, avec un shunt probable dans la zone d'atélectasie pulmonaire, probablement aiguë, car le poumon n'a pas pu créer des mécanismes compensatoires pour limiter la perfusion dans les zones mal ventilées.", "type": "PNEUMOLOGIE ET CHIRURGIE THORACIQUE", "options": {"1": "Court-circuit.", "2": "Hypoventilation.", "3": "Faible pression inspirée 02.", "4": "Maladie neuromusculaire.", "5": null}, "correct_option": 1, "explanations": {"1": {"exist": true, "char_ranges": [[0, 373]], "word_ranges": [[0, 58]], "text": "Une FiO2 de 1 et une hyperventilation par ventilation mécanique, démontrée par une PCO2 basse, ne parviennent pas à augmenter suffisamment la PO2, en raison d'un trouble V/Q, avec un shunt probable dans la zone d'atélectasie pulmonaire, probablement aiguë, car le poumon n'a pas pu créer des mécanismes compensatoires pour limiter la perfusion dans les zones mal ventilées."}, "2": {"exist": false, "char_ranges": [], "word_ranges": [], "text": ""}, "3": {"exist": false, "char_ranges": [], "word_ranges": [], "text": ""}, "4": {"exist": false, "char_ranges": [], "word_ranges": [], "text": ""}, "5": {"exist": false, "char_ranges": [], "word_ranges": [], "text": ""}}} +{"id": 167, "year": 2013, "question_id_specific": 85, "full_question": "Si, chez un patient souffrant d'insuffisance cardiaque chronique, nous détectons des ondes v proéminentes dans le pouls veineux jugulaire et, à l'auscultation cardiaque, un souffle holosystolique dans la région de l'appendice xiphoïde qui s'accentue à l'inspiration profonde, quelle est la valvulopathie responsable de cet examen physique ?", "full_answer": "Sémiologie de base. Mais raisonnement d'urgence : souffle systolique, quelque chose qui en systole devrait être fermé et ne l'est pas, ou devrait s'ouvrir et ne le fait pas : options 1, 3 et 5. Et si l'on voit des répercussions dans le pouls jugulaire, ce doit être dans les cavités droites. Outre le fait que l'onde v apparaît pendant la systole, alors que les oreillettes se remplissent : si le flux du VR monte vers le RA, ce qui se passera, c'est que l'onde v sera très importante...", "type": "CARDIOLOGIE ET CHIRURGIE CARDIOVASCULAIRE", "options": {"1": "Insuffisance mitrale.", "2": "Insuffisance pulmonaire.", "3": "Insuffisance tricuspide.", "4": "Insuffisance aortique.", "5": "Sténose aortique."}, "correct_option": 3, "explanations": {"1": {"exist": false, "char_ranges": [], "word_ranges": [], "text": ""}, "2": {"exist": false, "char_ranges": [], "word_ranges": [], "text": ""}, "3": {"exist": true, "char_ranges": [[197, 487]], "word_ranges": [[36, 89]], "text": "si l'on voit des répercussions dans le pouls jugulaire, ce doit être dans les cavités droites. Outre le fait que l'onde v apparaît pendant la systole, alors que les oreillettes se remplissent : si le flux du VR monte vers le RA, ce qui se passera, c'est que l'onde v sera très importante..."}, "4": {"exist": false, "char_ranges": [], "word_ranges": [], "text": ""}, "5": {"exist": false, "char_ranges": [], "word_ranges": [], "text": ""}}} +{"id": 154, "year": 2012, "question_id_specific": 75, "full_question": "Une femme de 42 ans signale une douleur à caractère inflammatoire et un gonflement des deux poignets, des 2e et 3e articulations métacarpophalangiennes et interphalangiennes proximales bilatérales et de la cheville gauche depuis 4 mois, accompagnés d'une raideur matinale de plus d'une heure. La radiographie de la main a montré une érosion de l'apophyse styloïde du cubitus dans le carpe droit. Les tests de laboratoire ont montré une Hb : 10 g/dL avec ESR de 45 mm dans la première heure, CRP 16 mg/L, facteur rhumatoïde 160 IU/ML. Après 6 mois de traitement à l'indométhacine et au méthotrexate, le patient continue de présenter une douleur et un gonflement des deux carpes, une raideur matinale durant 30 minutes et un test sanguin montrant une ESR de 30 mm dans la première heure et une CRP de 9 mg/dL. Lequel des énoncés suivants est vrai en ce qui concerne l'approche à adopter ?", "full_answer": "Il s'agit d'une patiente atteinte de polyarthrite rhumatoïde. A mon avis, comme elle a montré une nette amélioration clinique et biologique au cours des six derniers mois (diminution des réactifs de phase aiguë), je maintiendrais l'approche thérapeutique adoptée et attendrais une nouvelle évaluation. Cependant, je pense que cette question pourrait avoir une autre réponse valable, la numéro 3. Cela dépend un peu de l'attitude de chaque rhumatologue. Si ce que vous recherchez est une rémission de la maladie le plus rapidement possible, vous pourriez choisir d'envisager l'association d'un anti-TNF alpha...", "type": "RHEUMATOLOGIE", "options": {"1": "Interrompre le traitement en raison de l'absence de réponse et instaurer une dose élevée de prednisone pour le contrôle des symptômes uniquement.", "2": "Maintenir l'approche thérapeutique adoptée étant donné que nous ne sommes dans le programme que depuis 6 mois et que nous devrions attendre un minimum de 9 mois pour évaluer la réponse thérapeutique.", "3": "En l'absence de contre-indication médicale, il est possible d'envisager l'ajout d'un anti-TNF alpha au traitement.", "4": "Commencer un deuxième traitement de fond dès que possible, car il ne serait pas possible de commencer un traitement biologique après le méthotrexate seul.", "5": "Envisager l'instauration d'un traitement anti-CD20 associé au méthotrexate."}, "correct_option": 2, "explanations": {"1": {"exist": false, "char_ranges": [], "word_ranges": [], "text": ""}, "2": {"exist": true, "char_ranges": [[74, 301]], "word_ranges": [[11, 43]], "text": "comme elle a montré une nette amélioration clinique et biologique au cours des six derniers mois (diminution des réactifs de phase aiguë), je maintiendrais l'approche thérapeutique adoptée et attendrais une nouvelle évaluation."}, "3": {"exist": true, "char_ranges": [[453, 610]], "word_ranges": [[67, 90]], "text": "Si ce que vous recherchez est une rémission de la maladie le plus rapidement possible, vous pourriez choisir d'envisager l'association d'un anti-TNF alpha..."}, "4": {"exist": false, "char_ranges": [], "word_ranges": [], "text": ""}, "5": {"exist": false, "char_ranges": [], "word_ranges": [], "text": ""}}} +{"id": 176, "year": 2013, "question_id_specific": 57, "full_question": "Femme de 56 ans, avec des antécédents de schizophrénie bien contrôlée, sans habitudes toxiques. Elle a été admise pour une pneumonie du lobe moyen avec un petit épanchement pleural associé et a été traitée par lévofloxacine 500 mg/24h. Elle a eu une bonne évolution clinique à l'exception d'une fièvre persistante et d'une leucocytose au sixième jour de traitement. Aucune étude microbiologique n'est disponible. La conduite à tenir la plus appropriée est la suivante :", "full_answer": "Tant que la fièvre persiste dans une pneumonie en évolution, il existe un risque d'empyématisation, surtout si un petit épanchement s'est déjà produit au début. Le traitement antibiotique est en principe correct et avant d'envisager des changements de traitement, il convient d'évaluer la possibilité d'une empyématisation.", "type": "PNEUMOLOGIE", "options": {"1": "L'évolution est normale, le traitement doit être maintenu jusqu'à ce que 10 jours se soient écoulés.", "2": "Elle est considérée comme un échec thérapeutique et le traitement antibiotique doit être modifié.", "3": "Effectuer une thoracentèse pour éliminer un empyème.", "4": "Ajouter des corticostéroïdes à une dose de 0,5 mg/kg/jour au traitement antibiotique.", "5": "Effectuer une bronchoscopie avec biopsie, aspiration et lavage broncho-alvéolaire."}, "correct_option": 3, "explanations": {"1": {"exist": false, "char_ranges": [], "word_ranges": [], "text": ""}, "2": {"exist": false, "char_ranges": [], "word_ranges": [], "text": ""}, "3": {"exist": true, "char_ranges": [[161, 323]], "word_ranges": [[25, 46]], "text": "Le traitement antibiotique est en principe correct et avant d'envisager des changements de traitement, il convient d'évaluer la possibilité d'une empyématisation."}, "4": {"exist": false, "char_ranges": [], "word_ranges": [], "text": ""}, "5": {"exist": false, "char_ranges": [], "word_ranges": [], "text": ""}}} +{"id": 130, "year": 2012, "question_id_specific": 82, "full_question": "Une patiente de 18 ans avec des antécédents d'absences entre 6 et 9 ans, de crises généralisées tonico-cloniques d'apparition récente et de sauts violents des membres supérieurs au petit déjeuner. Les manifestations cliniques s'aggravent lors des sorties nocturnes du week-end. L'EEG montre des décharges aiguës de pointes de polypes à 6 cycles/seconde. Le diagnostic le plus probable est le suivant :", "full_answer": "Un patient d'âge adolescent présentant des myoclonies (\"sauts violents\") au moment du petit déjeuner nous orientera presque toujours vers une épilepsie myoclonique juvénile (réponse 5 correcte), une entité très bien caractérisée. Les autres caractéristiques sont des antécédents de crises ou d'absences, une aggravation lors des sorties nocturnes et un EEG avec des décharges généralisées en polyspike à une fréquence plus élevée que les absences. Parmi les réponses restantes, Lennox Gastaut se développe à un âge précoce avec d'autres symptômes, 4 est identique aux absences, et 3 se présente généralement avec des crises partielles complexes et, dans tous les cas, l'EEG est clairement distinct d'un foyer temporal.", "type": "NEUROLOGIE ET NEUROCHIRURGIE", "options": {"1": "Grand mal épileptique.", "2": "Syndrome de Lennox-Gastaut.", "3": "Épilepsie symptomatique due à une sclérose temporale mésiale.", "4": "Petite maladie atypique.", "5": "Épilepsie myoclonique juvénile."}, "correct_option": 5, "explanations": {"1": {"exist": false, "char_ranges": [], "word_ranges": [], "text": ""}, "2": {"exist": true, "char_ranges": [[478, 547]], "word_ranges": [[68, 79]], "text": "Lennox Gastaut se développe à un âge précoce avec d'autres symptômes,"}, "3": {"exist": true, "char_ranges": [[579, 644]], "word_ranges": [[84, 94]], "text": "et 3 se présente généralement avec des crises partielles complexes"}, "4": {"exist": true, "char_ranges": [[548, 577]], "word_ranges": [[79, 84]], "text": "4 est identique aux absences,"}, "5": {"exist": true, "char_ranges": [[0, 194]], "word_ranges": [[0, 26]], "text": "Un patient d'âge adolescent présentant des myoclonies (\"sauts violents\") au moment du petit déjeuner nous orientera presque toujours vers une épilepsie myoclonique juvénile (réponse 5 correcte),"}}} +{"id": 276, "year": 2016, "question_id_specific": 75, "full_question": "Indiquez la situation clinique en relation avec l'infection par le virus de l'hépatite B chez un patient de 5 ans originaire du Nigeria, avec un examen physique normal et les sérologies suivantes pour l'hépatite B : HBsAg + / ANTI-HBs - / HbeAg - / ANTI-HBe + / ANTI-HBc IgM - / ANTI-HBc IgG + / HBV DNA + :", "full_answer": "La présence d'anticorps contre le noyau implique un contact naturel et des IgG, ce qui n'est pas aigu. L'absence d'antigène e exclut une réplication active. La persistance de l'antigène de surface (HBsAg) et de l'ADN du virus indique que le virus est toujours présent. L'ensemble de ces éléments indique un porteur asymptomatique. Les porteurs du mutant pré-core présentent généralement des périodes symptomatiques récurrentes et une inflammation accrue du foie. Il n'y a pas assez de données pour l'exclure complètement, mais il ne fait pas partie des réponses et indique un examen physique tout à fait normal.", "type": "SYSTÈME DIGESTIF", "options": {"1": "Infection aiguë.", "2": "Infection chronique.", "3": "Patient vacciné.", "4": "Porteurs asymptomatiques.", "5": null}, "correct_option": 4, "explanations": {"1": {"exist": false, "char_ranges": [], "word_ranges": [], "text": ""}, "2": {"exist": false, "char_ranges": [], "word_ranges": [], "text": ""}, "3": {"exist": false, "char_ranges": [], "word_ranges": [], "text": ""}, "4": {"exist": true, "char_ranges": [[0, 330]], "word_ranges": [[0, 52]], "text": "La présence d'anticorps contre le noyau implique un contact naturel et des IgG, ce qui n'est pas aigu. L'absence d'antigène e exclut une réplication active. La persistance de l'antigène de surface (HBsAg) et de l'ADN du virus indique que le virus est toujours présent. L'ensemble de ces éléments indique un porteur asymptomatique."}, "5": {"exist": false, "char_ranges": [], "word_ranges": [], "text": ""}}} +{"id": 183, "year": 2013, "question_id_specific": 64, "full_question": "Un patient sous assistance nutritionnelle entérale présente, 72 heures après le début de la nutrition entérale, un bilan sanguin montrant une hypophosphatémie et une hypokaliémie, avec des signes cliniques d'insuffisance cardiaque. Le patient est diagnostiqué comme souffrant du syndrome de réalimentation. Indiquez lequel des facteurs suivants n'est PAS considéré comme un facteur de risque pour un patient présentant cette pathologie :", "full_answer": "Le syndrome de réalimentation survient chez des patients précédemment malnutris exposés à une thérapie de nutrition orale, entérale ou parentérale. Toutes les réponses sont des causes de malnutrition, sauf 3.", "type": "ENDOCRINOLOGIE", "options": {"1": "Malnutrition calorique antérieure.", "2": "Anorexie mentale.", "3": "Obésité non morbide.", "4": "Personnes âgées.", "5": "Vomissements et diarrhées prolongés."}, "correct_option": 3, "explanations": {"1": {"exist": true, "char_ranges": [[0, 208]], "word_ranges": [[0, 30]], "text": "Le syndrome de réalimentation survient chez des patients précédemment malnutris exposés à une thérapie de nutrition orale, entérale ou parentérale. Toutes les réponses sont des causes de malnutrition, sauf 3."}, "2": {"exist": true, "char_ranges": [[0, 208]], "word_ranges": [[0, 30]], "text": "Le syndrome de réalimentation survient chez des patients précédemment malnutris exposés à une thérapie de nutrition orale, entérale ou parentérale. Toutes les réponses sont des causes de malnutrition, sauf 3."}, "3": {"exist": true, "char_ranges": [[0, 208]], "word_ranges": [[0, 30]], "text": "Le syndrome de réalimentation survient chez des patients précédemment malnutris exposés à une thérapie de nutrition orale, entérale ou parentérale. Toutes les réponses sont des causes de malnutrition, sauf 3."}, "4": {"exist": true, "char_ranges": [[0, 208]], "word_ranges": [[0, 30]], "text": "Le syndrome de réalimentation survient chez des patients précédemment malnutris exposés à une thérapie de nutrition orale, entérale ou parentérale. Toutes les réponses sont des causes de malnutrition, sauf 3."}, "5": {"exist": true, "char_ranges": [[0, 208]], "word_ranges": [[0, 30]], "text": "Le syndrome de réalimentation survient chez des patients précédemment malnutris exposés à une thérapie de nutrition orale, entérale ou parentérale. Toutes les réponses sont des causes de malnutrition, sauf 3."}}} +{"id": 115, "year": 2012, "question_id_specific": 98, "full_question": "Une femme de 32 ans atteinte de paralysie cérébrale à la suite d'un accouchement s'est présentée au service des urgences pour quelques jours d'urines foncées en relation avec un épisode de forte fièvre et de toux sèche. A l'admission, l'hémogramme montre 16900 leucocytes/mm3 (85% S, 11% L, 4% M), hémoglobine 6,3 g/dl ; MCV 109 fl, 360000 plaquettes/mm3. En biochimie, LDH 2408 ; bilirubine 6,8 mg/dl, (bilirubine non conjuguée 6,1 mg/dl), GOT et GPT normaux. L'étude morphologique du sang montre une anisocytose macrocytaire avec des formes sphérocytaires fréquentes et une polychromatophilie sans blastes. L'étude des anticorps irréguliers est positive pour la panagglutinine, ce qui rend le crossmatch difficile. Quelle serait votre suspicion et le traitement le plus approprié ?", "full_answer": "Une longue déclaration contenant des données plutôt confuses. Nous pouvons aborder cette question de deux manières : en examinant les données clés et en allant directement au diagnostic que nous envisageons ou en les écartant une à une. La cholurie, les LDH, la présence de panagglutinine et le frottis du sang périphérique \"sentent\" l'anémie hémolytique. La réponse 1 ne peut pas l'être car une aplasie médullaire n'explique pas la cholurie, l'élévation des LDH, ni la recherche irrégulière d'anticorps positifs à la panagglutinine ; une aplasie est une insuffisance médullaire caractérisée par une disparition totale ou partielle des progéniteurs hématopoïétiques. De plus, on n'observe pas de pancytopénie, ce qui nous ferait plutôt pencher pour cette pathologie. La réponse 2 n'est pas non plus possible car la sphérocytose ne justifie pas la présence de panagglutinine. La réponse 3 est celle qui me semble correcte : une anémie hémolytique auto-immune justifierait les données fournies : LDH et bilirubinémie élevées dues à la destruction des globules rouges, polychromatophilie, sphérocytose et anisocytose car la moelle travaille beaucoup pour essayer de compenser l'anémie, qui est régénérative. L'étude des anticorps irréguliers et la présence de panagglutinine confirment également cette réponse, car la fixation d'un anticorps sur l'hémocyte entraîne sa lyse et sa destruction. La fillette tousse et a de la fièvre, ce qui correspond à une infection respiratoire. Le traitement initial consiste en l'administration de corticoïdes. Ceci est confirmé par Sans Sabrafens dans son livre \"Hématologie clinique\" [1]. La réponse 4 suscite beaucoup de doutes ; elle n'est pas possible car une anémie pernicieuse ne justifie pas la présence de panagglutinine alors qu'elle justifierait l'élévation des LDH et de la bilirubine ; de plus, il s'agit d'une hémolyse intramédullaire, arégénérative, il n'y a pas de réticulocytose ni de libération de formes immatures dans le sang pour tenter de compenser et d'arranger la situation. La réponse 5 est fausse car il n'y a pas de blastes dans le sang et elle n'explique pas la présence de panagglutinine.", "type": "HÉMATOLOGIE", "options": {"1": "Aplasie de la moelle épinière et immunothérapie par thymoglobuline et cyclosporine.", "2": "Sphérocytose héréditaire et splénectomie.", "3": "Anémie hémolytique auto-immune associée à une infection respiratoire et à des corticostéroïdes.", "4": "Anémie pernicieuse et injections régulières de vitamine B12.", "5": "Leucémie aiguë et chimiothérapie."}, "correct_option": 3, "explanations": {"1": {"exist": true, "char_ranges": [[356, 534]], "word_ranges": [[55, 83]], "text": "La réponse 1 ne peut pas l'être car une aplasie médullaire n'explique pas la cholurie, l'élévation des LDH, ni la recherche irrégulière d'anticorps positifs à la panagglutinine ;"}, "2": {"exist": true, "char_ranges": [[767, 874]], "word_ranges": [[115, 133]], "text": "La réponse 2 n'est pas non plus possible car la sphérocytose ne justifie pas la présence de panagglutinine."}, "3": {"exist": true, "char_ranges": [[875, 1204]], "word_ranges": [[133, 180]], "text": "La réponse 3 est celle qui me semble correcte : une anémie hémolytique auto-immune justifierait les données fournies : LDH et bilirubinémie élevées dues à la destruction des globules rouges, polychromatophilie, sphérocytose et anisocytose car la moelle travaille beaucoup pour essayer de compenser l'anémie, qui est régénérative."}, "4": {"exist": true, "char_ranges": [[1623, 2030]], "word_ranges": [[241, 306]], "text": "La réponse 4 suscite beaucoup de doutes ; elle n'est pas possible car une anémie pernicieuse ne justifie pas la présence de panagglutinine alors qu'elle justifierait l'élévation des LDH et de la bilirubine ; de plus, il s'agit d'une hémolyse intramédullaire, arégénérative, il n'y a pas de réticulocytose ni de libération de formes immatures dans le sang pour tenter de compenser et d'arranger la situation."}, "5": {"exist": true, "char_ranges": [[2031, 2149]], "word_ranges": [[306, 329]], "text": "La réponse 5 est fausse car il n'y a pas de blastes dans le sang et elle n'explique pas la présence de panagglutinine."}}} +{"id": 75, "year": 2012, "question_id_specific": 65, "full_question": "Un patient admis pour une pancréatite aiguë commence par une tachypnée, une tachycardie, des sueurs et une cyanose progressive. La PaO2 est de 55 mm Hg (rapport PaO2/FiO2<200). La CXR montre des infiltrats pulmonaires alvéolaires bilatéraux. La pression de coin capillaire pulmonaire est normale. L'oxygénothérapie n'améliore pas la situation. Quel est le diagnostic le plus probable ?", "full_answer": "Question à réponse directe. La question raconte l'histoire d'un patient atteint de pancréatite aiguë qui développe une insuffisance respiratoire aiguë. L'énoncé énumère les critères du SDRA (syndrome de détresse respiratoire aiguë) : PaO2/fiO2 < 200 + infiltrats pulmonaires bilatéraux + PCP<18. Le diagnostic est donc celui d'une détresse respiratoire et la bonne réponse est donc l'option 5.", "type": "ANESTHÉSIOLOGIE, SOINS INTENSIFS ET MÉDECINE D'URGENCE", "options": {"1": "Pneumonie nosocomiale.", "2": "Insuffisance cardiaque.", "3": "Lymphangite carcinomateuse.", "4": "Thromboembolie pulmonaire.", "5": "Détresse respiratoire."}, "correct_option": 5, "explanations": {"1": {"exist": false, "char_ranges": [], "word_ranges": [], "text": ""}, "2": {"exist": false, "char_ranges": [], "word_ranges": [], "text": ""}, "3": {"exist": false, "char_ranges": [], "word_ranges": [], "text": ""}, "4": {"exist": false, "char_ranges": [], "word_ranges": [], "text": ""}, "5": {"exist": true, "char_ranges": [[152, 393]], "word_ranges": [[20, 57]], "text": "L'énoncé énumère les critères du SDRA (syndrome de détresse respiratoire aiguë) : PaO2/fiO2 < 200 + infiltrats pulmonaires bilatéraux + PCP<18. Le diagnostic est donc celui d'une détresse respiratoire et la bonne réponse est donc l'option 5."}}} +{"id": 191, "year": 2013, "question_id_specific": 125, "full_question": "Un nourrisson de 13 mois s'est présenté au service des urgences avec une fièvre allant jusqu'à 39 ºC pendant 48 heures d'évolution, sans autres symptômes associés. L'examen des organes et de l'appareil ne révèle rien de significatif et met en évidence un bon état général. Vous alliez le renvoyer chez lui mais le pédiatre de garde demande une culture d'urine et une culture d'urine par cathétérisme. Les urines montrent une leucocyturie ++, une hématurie ++ et des nitrites ++ et les urines des bacilles Gram négatif. L'analyse de sang ne montre pas de leucocytose et la protéine C-réactive est à 50 mg/l. Le médecin traitant vous dit maintenant qu'il n'est pas nécessaire d'hospitaliser l'enfant et de lui prescrire un antibiotique par voie orale. Indiquez le traitement antibiotique empirique le moins approprié dans ce cas :", "full_answer": "Tous les autres médicaments constituent un bon choix dans les infections urinaires à bacilles largement négatifs (le plus souvent E. coli). La présence de leucocytes et de nitrites dans l'urine est très évocatrice d'une infection urinaire. Et si le test de Gram de l'urine détecte des bactéries G-, il s'agit probablement d'une infection urinaire due à E. coli. De nombreux E. coli sont résistants à l'amoxicilline.", "type": "PÉDIATRIE", "options": {"1": "Amoxicilline.", "2": "Amoxicilline-acide clavulanique.", "3": "Céfuroxime-axétil.", "4": "Cotrimoxazole.", "5": "Cefixime."}, "correct_option": 1, "explanations": {"1": {"exist": true, "char_ranges": [[140, 415]], "word_ranges": [[21, 66]], "text": "La présence de leucocytes et de nitrites dans l'urine est très évocatrice d'une infection urinaire. Et si le test de Gram de l'urine détecte des bactéries G-, il s'agit probablement d'une infection urinaire due à E. coli. De nombreux E. coli sont résistants à l'amoxicilline."}, "2": {"exist": true, "char_ranges": [[0, 112]], "word_ranges": [[0, 16]], "text": "Tous les autres médicaments constituent un bon choix dans les infections urinaires à bacilles largement négatifs"}, "3": {"exist": true, "char_ranges": [[0, 112]], "word_ranges": [[0, 16]], "text": "Tous les autres médicaments constituent un bon choix dans les infections urinaires à bacilles largement négatifs"}, "4": {"exist": true, "char_ranges": [[0, 112]], "word_ranges": [[0, 16]], "text": "Tous les autres médicaments constituent un bon choix dans les infections urinaires à bacilles largement négatifs"}, "5": {"exist": true, "char_ranges": [[0, 112]], "word_ranges": [[0, 16]], "text": "Tous les autres médicaments constituent un bon choix dans les infections urinaires à bacilles largement négatifs"}}} +{"id": 323, "year": 2016, "question_id_specific": 150, "full_question": "Un garçon de 12 mois qui, d'après les examens de santé effectués depuis sa naissance, a un testicule droit dans le canal inguinal qui ne peut pas descendre dans le scrotum. Cochez la bonne réponse :", "full_answer": "Il s'agit de la cryptorchidie. Elle doit être traitée avant l'âge de 2 ans et le traitement hormonal n'est actuellement pas utilisé. Le traitement de premier choix est l'orchidopexie.", "type": "UROLOGIE", "options": {"1": "Le diagnostic le plus probable est celui d'un testicule rétractile.", "2": "Il faut attendre l'âge de deux ans pour que la descente spontanée se produise.", "3": "La gonadotrophine chorionique humaine est le traitement de premier choix.", "4": "L'indication d'une orchidopexie ne doit pas être différée.", "5": null}, "correct_option": 4, "explanations": {"1": {"exist": false, "char_ranges": [], "word_ranges": [], "text": ""}, "2": {"exist": false, "char_ranges": [], "word_ranges": [], "text": ""}, "3": {"exist": false, "char_ranges": [], "word_ranges": [], "text": ""}, "4": {"exist": true, "char_ranges": [[0, 183]], "word_ranges": [[0, 29]], "text": "Il s'agit de la cryptorchidie. Elle doit être traitée avant l'âge de 2 ans et le traitement hormonal n'est actuellement pas utilisé. Le traitement de premier choix est l'orchidopexie."}, "5": {"exist": false, "char_ranges": [], "word_ranges": [], "text": ""}}} +{"id": 311, "year": 2016, "question_id_specific": 217, "full_question": "Une femme de 84 ans s'est présentée avec une perte de vision de l'œil gauche depuis 4 jours, accompagnée d'une métamorphopsie. La macula présente d'abondants exsudats durs, deux petites hémorragies profondes et un décollement de rétine neurosensoriel localisé. L'œil controlatéral présente d'abondants drusen mous. Parmi les diagnostics suivants, lequel vous semble le plus probable ?", "full_answer": "Aucune des trois autres options n'entraîne de tels symptômes (le VAPD est généralement asymptomatique, l'OVCR entraîne une perte brutale et totale ou quasi-totale de la vision avec la \"tache rouge cerise\" typique au fond d'œil, et l'AINO artéritique et non artéritique entraîne un œdème papillaire généralement accompagné d'hémorragies péripapillaires mais non maculaires, et aucune de ces trois options n'est associée à des drusen dans l'œil controlatéral). La DMLA dans sa forme exsudative présente à la fois ce tableau clinique (perte de vision et métamorphopsie) et le fond d'œil caractéristique décrit dans la question, et des drusen sont généralement observés dans l'œil controlatéral.", "type": "OPHTHALMOLOGIE", "options": {"1": "Décollement postérieur aigu du vitré.", "2": "Dégénérescence maculaire liée à l'âge (DMLA) exsudative.", "3": "Obstruction de l'artère centrale de la rétine.", "4": "Neuropathie optique ischémique antérieure non artéritique.", "5": null}, "correct_option": 2, "explanations": {"1": {"exist": true, "char_ranges": [[376, 458]], "word_ranges": [[53, 66]], "text": "aucune de ces trois options n'est associée à des drusen dans l'œil controlatéral)."}, "2": {"exist": true, "char_ranges": [[459, 691]], "word_ranges": [[66, 102]], "text": "La DMLA dans sa forme exsudative présente à la fois ce tableau clinique (perte de vision et métamorphopsie) et le fond d'œil caractéristique décrit dans la question, et des drusen sont généralement observés dans l'œil controlatéral."}, "3": {"exist": true, "char_ranges": [[376, 458]], "word_ranges": [[53, 66]], "text": "aucune de ces trois options n'est associée à des drusen dans l'œil controlatéral)."}, "4": {"exist": true, "char_ranges": [[376, 458]], "word_ranges": [[53, 66]], "text": "aucune de ces trois options n'est associée à des drusen dans l'œil controlatéral)."}, "5": {"exist": false, "char_ranges": [], "word_ranges": [], "text": ""}}} +{"id": 137, "year": 2012, "question_id_specific": 164, "full_question": "Face à un traumatisme direct du côté droit de la face après lequel le patient présente un hématome palpébral unilatéral, une diplopie dans le regard vertical et une difficulté à ouvrir la bouche, laquelle des affirmations suivantes est vraie ?", "full_answer": "Une diplopie dans le regard vertical est très évocatrice d'une atteinte des parois osseuses de l'orbite. La fracture de Lefort de type I n'affecte pas le contenu orbital. Compte tenu de ces informations, la seule réponse correcte est 2. Si nous avions des doutes sur la question de savoir si la fracture de Lefort de type I affecte ou non l'orbite, il suffit de regarder les réponses. La personne qui a posé la question voulait s'assurer que la seule réponse valable était le numéro 2 ; c'est pourquoi elle a ajouté la mention \"mais au cas où, nous le vérifierons par un scanner\".", "type": "OTORHINOLARYNGOLOGIE ET CHIRURGIE MAXILLO-FACIALE", "options": {"1": "Il s'agit très probablement d'une fracture de la partie moyenne de la face de type Lefort I.", "2": "Il s'agit probablement d'une fracture orbitomalaire unilatérale. Le diagnostic devrait idéalement être vérifié par un scanner (tomographie axiale informatisée).", "3": "Il s'agit d'une fracture de la base du crâne au niveau du foramen carotidien.", "4": "Le diagnostic probable est une fracture avec luxation du condyle mandibulaire.", "5": "Une fracture mandibulaire est probablement associée à une fracture moyenne de la face de type Lefort I."}, "correct_option": 2, "explanations": {"1": {"exist": true, "char_ranges": [[105, 170]], "word_ranges": [[16, 28]], "text": "La fracture de Lefort de type I n'affecte pas le contenu orbital."}, "2": {"exist": true, "char_ranges": [[0, 104]], "word_ranges": [[0, 16]], "text": "Une diplopie dans le regard vertical est très évocatrice d'une atteinte des parois osseuses de l'orbite."}, "3": {"exist": false, "char_ranges": [], "word_ranges": [], "text": ""}, "4": {"exist": false, "char_ranges": [], "word_ranges": [], "text": ""}, "5": {"exist": false, "char_ranges": [], "word_ranges": [], "text": ""}}} +{"id": 309, "year": 2016, "question_id_specific": 215, "full_question": "Un homme de 47 ans atteint de myopie magna, opéré de la cataracte il y a deux ans, se présente aux urgences en signalant une perte de vision profonde et indolore de l'œil droit. Parmi les diagnostics suivants, lequel peut être à l'origine de cette symptomatologie ?", "full_answer": "Cette question pourrait être controversée. L'option 1 est fortement écartée car l'endophtalmie se manifeste généralement par une forte douleur. L'option 3, à savoir une forme humide de dégénérescence maculaire liée à l'âge, est également exclue, car elle survient généralement chez des patients plus âgés, bien qu'un patient atteint de myopie magna puisse présenter une dégénérescence maculaire myopique (dont le tableau clinique est similaire à celui de la DMLA, mais il s'agit d'une pathologie différente). Un décollement postérieur du vitré (DPV) n'est généralement pas très symptomatique, produisant une myodésopsie mais pas de perte de vision ni de douleur ; cependant, certains DPV sont hémorragiques, car les vaisseaux sont tirés, produisant une hémorragie du vitré qui entraînerait une perte de vision. Dans tous les cas, étant donné les antécédents de myopie et de chirurgie intraoculaire, le premier diagnostic à exclure serait le décollement de la rétine, car il s'agit de deux facteurs de risque pour cette pathologie. Techniquement, les options 2 et 4 (un DPV hémorragique aigu) pourraient toutes deux provoquer ces symptômes, mais l'option la plus correcte est l'option 2 (et je suis sûr que c'est celle à laquelle l'examinateur souhaite que l'on réponde).", "type": "OPHTHALMOLOGIE", "options": {"1": "Endophtalmie post-chirurgicale.", "2": "Décollement de la rétine.", "3": "Dégénérescence maculaire liée à l'âge, forme humide.", "4": "Décollement du vitré postérieur.", "5": null}, "correct_option": 2, "explanations": {"1": {"exist": true, "char_ranges": [[43, 143]], "word_ranges": [[5, 19]], "text": "L'option 1 est fortement écartée car l'endophtalmie se manifeste généralement par une forte douleur."}, "2": {"exist": true, "char_ranges": [[1031, 1270]], "word_ranges": [[155, 193]], "text": "Techniquement, les options 2 et 4 (un DPV hémorragique aigu) pourraient toutes deux provoquer ces symptômes, mais l'option la plus correcte est l'option 2 (et je suis sûr que c'est celle à laquelle l'examinateur souhaite que l'on réponde)."}, "3": {"exist": true, "char_ranges": [[144, 305]], "word_ranges": [[19, 44]], "text": "L'option 3, à savoir une forme humide de dégénérescence maculaire liée à l'âge, est également exclue, car elle survient généralement chez des patients plus âgés,"}, "4": {"exist": true, "char_ranges": [[1031, 1270]], "word_ranges": [[155, 193]], "text": "Techniquement, les options 2 et 4 (un DPV hémorragique aigu) pourraient toutes deux provoquer ces symptômes, mais l'option la plus correcte est l'option 2 (et je suis sûr que c'est celle à laquelle l'examinateur souhaite que l'on réponde)."}, "5": {"exist": false, "char_ranges": [], "word_ranges": [], "text": ""}}} +{"id": 279, "year": 2016, "question_id_specific": 32, "full_question": "Une femme de 59 ans s'est présentée avec une diarrhée aqueuse chronique de 4 mois. À l'endoscopie, la muqueuse ne présentait aucun aspect pertinent. En particulier, aucun ulcère ou zone friable n'a été observé. Une biopsie du côlon transverse a été réalisée. L'histopathologie a révélé une zone épaissie sous l'épithélium de revêtement superficiel, qui était plus évidente par la technique du trichrome de Masson et impliquait une atrophie et une dénudation de l'épithélium. Le diagnostic de la lésion intestinale est ?", "full_answer": "Dans ce cas, la présence d'un épaississement de la couche de collagène sous-épithéliale, plus évident avec le trichrome de Masson (une coloration spéciale qui permet de différencier les fibres de collagène), est pathognomonique de la colite collagénique. L'atrophie et la dénudation épithéliale sont caractéristiques de cette maladie, qui ne provoque généralement pas de modifications macroscopiques de la muqueuse. Une densité accrue de lymphocytes intra-épithéliaux et l'absence d'altérations architecturales des cryptes (caractéristiques de la colite ulcéreuse et de la maladie de Crohn) confirment ce diagnostic.", "type": "ANATOMIE PATHOLOGIQUE", "options": {"1": "Colite ulcéreuse chronique.", "2": "Colite pseudomembraneuse.", "3": "Colite collagénique.", "4": "Maladie de Crohn fibrosante.", "5": null}, "correct_option": 3, "explanations": {"1": {"exist": false, "char_ranges": [], "word_ranges": [], "text": ""}, "2": {"exist": false, "char_ranges": [], "word_ranges": [], "text": ""}, "3": {"exist": true, "char_ranges": [[13, 254]], "word_ranges": [[3, 37]], "text": "la présence d'un épaississement de la couche de collagène sous-épithéliale, plus évident avec le trichrome de Masson (une coloration spéciale qui permet de différencier les fibres de collagène), est pathognomonique de la colite collagénique."}, "4": {"exist": false, "char_ranges": [], "word_ranges": [], "text": ""}, "5": {"exist": false, "char_ranges": [], "word_ranges": [], "text": ""}}} +{"id": 437, "year": 2018, "question_id_specific": 118, "full_question": "Luis est un jeune homme de 25 ans qui a subi une splénectomie après un accident de vélo il y a un an. Il y a 24 heures, un chien l'a mordu, lui causant une petite blessure à la main droite. Il s'est rendu à son centre de santé (situé à 3 heures de l'hôpital le plus proche) avec une fièvre de 39ºC, une douleur au niveau de la plaie et un malaise général. À l'examen, la tension artérielle est de 100/60 mm Hg, la fréquence cardiaque est de 110 battements par minute, la plaie est légèrement enflée sans pus. Laquelle des mesures suivantes est la plus indiquée à ce moment-là ?", "full_answer": "Lorsqu'un patient splénectomisé est considéré comme un patient \"spécial\", il doit recevoir un traitement antibiotique dès que possible, même si la plaie ne semble pas infectée. Vous devez vous assurer que le chien vous appartient (vous devez demander si l'animal est correctement vacciné) et que le patient est correctement vacciné (5 doses de tétanos, aucun tétanos ne serait nécessaire) ; faire un rapport d'agression animale, pour l'observation de l'animal et prendre des mesures si cela s'avère nécessaire.", "type": "ÉPIDÉMIOLOGIE ET MÉDECINE PRÉVENTIVE", "options": {"1": "Envoyer à l'hôpital pour la vaccination contre la rage et le tétanos et garder en observation.", "2": "Nettoyer la plaie et administrer de la ganunaglobuline non spécifique par voie intramusculaire.", "3": "Administrer 400 mg de moxifloxacine par voie orale et envoyer le patient à l'hôpital.", "4": "Administrer 600 mg de clindamycine par voie orale toutes les 8 heures et observer.", "5": null}, "correct_option": 3, "explanations": {"1": {"exist": false, "char_ranges": [], "word_ranges": [], "text": ""}, "2": {"exist": false, "char_ranges": [], "word_ranges": [], "text": ""}, "3": {"exist": true, "char_ranges": [[0, 176]], "word_ranges": [[0, 26]], "text": "Lorsqu'un patient splénectomisé est considéré comme un patient \"spécial\", il doit recevoir un traitement antibiotique dès que possible, même si la plaie ne semble pas infectée."}, "4": {"exist": false, "char_ranges": [], "word_ranges": [], "text": ""}, "5": {"exist": false, "char_ranges": [], "word_ranges": [], "text": ""}}} +{"id": 112, "year": 2012, "question_id_specific": 154, "full_question": "Patiente de 67 ans avec des antécédents de ménopause à 55 ans, 3 grossesses avec 3 accouchements euthyroïdiens, diabète de type 2 depuis 6 ans, traitement à la nifédipine pour l'hypertension. Elle consulte pour des symptômes de métrorragies intermittentes et peu abondantes évoluant depuis 2 mois. L'examen gynécologique révèle des organes génitaux externes sans lésion, un col atrophié, un utérus et des annexes normaux à la palpation et une étude cytologique du col normale. L'échographie transvaginale montre un endomètre hyperéchogène de 7 mm. Parmi les examens suivants, lequel est le plus approprié et le plus sensible pour établir un diagnostic ?", "full_answer": "La bonne réponse est 3. L'étude qui nous fournit le plus d'informations sur la pathologie endométriale est l'hystéroscopie, qui nous permet de réaliser une biopsie endométriale ciblée. La cytologie de l'endomètre est réalisée à l'aveugle et comporte un grand nombre de faux négatifs. L'IRM nous aidera à déterminer le stade du cancer de l'endomètre.", "type": "GYNÉCOLOGIE ET OBSTÉTRIQUE", "options": {"1": "Conisation cervicale.", "2": "Cytologie de l'endomètre.", "3": "Hystéroscopie et biopsie de l'endomètre.", "4": "Imagerie par résonance magnétique du bassin.", "5": "Examen sous anesthésie de l'appareil génital et biopsie du col de l'utérus et de l'endomètre."}, "correct_option": 3, "explanations": {"1": {"exist": false, "char_ranges": [], "word_ranges": [], "text": ""}, "2": {"exist": true, "char_ranges": [[185, 283]], "word_ranges": [[27, 43]], "text": "La cytologie de l'endomètre est réalisée à l'aveugle et comporte un grand nombre de faux négatifs."}, "3": {"exist": true, "char_ranges": [[24, 184]], "word_ranges": [[5, 27]], "text": "L'étude qui nous fournit le plus d'informations sur la pathologie endométriale est l'hystéroscopie, qui nous permet de réaliser une biopsie endométriale ciblée."}, "4": {"exist": true, "char_ranges": [[284, 349]], "word_ranges": [[43, 54]], "text": "L'IRM nous aidera à déterminer le stade du cancer de l'endomètre."}, "5": {"exist": false, "char_ranges": [], "word_ranges": [], "text": ""}}} +{"id": 374, "year": 2016, "question_id_specific": 101, "full_question": "Un homme de 52 ans se présente aux urgences avec des céphalées et de la fièvre (37,8°C) depuis 2 jours. Au cours des dernières heures, il a également présenté des difficultés de nomination et de compréhension. L'examen ne révèle aucune rigidité du cou, l'élément le plus frappant étant la présence d'une aphasie mixte. Le scanner crânien montre une légère hypodensité dans le lobe temporal gauche sans effet de masse et sans prise de contraste. Laquelle des affirmations suivantes est correcte ?", "full_answer": "Le tableau décrit est typique d'une méningo-encéphalite virale, plus précisément du VHZ, avec un tropisme significatif pour le lobe temporal, provoquant une fièvre et, dans de nombreux cas, une aphasie. La bonne option serait donc 2, puisque le liquide céphalo-rachidien présente normalement une pléiocytose lymphocytaire modérée, avec une glycorrhachie normale. Les autres options ne sont pas correctes pour les raisons suivantes : La première parce qu'il ne s'agit pas d'une méningite bactérienne qui se présenterait avec plus de fièvre, la troisième parce que l'encéphalite limbique se présente normalement afébrile avec des changements de comportement et la dernière parce que l'image décrite n'est pas un abcès, bien délimité.", "type": "NEUROLOGIE", "options": {"1": "La méningite bactérienne est la première impression diagnostique et un traitement par céphalosporine de 3e génération doit être instauré dès que possible.", "2": "Le LCR de ce patient présentera très probablement une pléiocytose à prédominance lymphocytaire avec une glycorrhachie normale.", "3": "Nous soupçonnerions une encéphalite limbique.", "4": "Il s'agit d'un abcès cérébral à un stade précoce.", "5": null}, "correct_option": 2, "explanations": {"1": {"exist": true, "char_ranges": [[433, 539]], "word_ranges": [[62, 79]], "text": "La première parce qu'il ne s'agit pas d'une méningite bactérienne qui se présenterait avec plus de fièvre,"}, "2": {"exist": true, "char_ranges": [[0, 362]], "word_ranges": [[0, 50]], "text": "Le tableau décrit est typique d'une méningo-encéphalite virale, plus précisément du VHZ, avec un tropisme significatif pour le lobe temporal, provoquant une fièvre et, dans de nombreux cas, une aphasie. La bonne option serait donc 2, puisque le liquide céphalo-rachidien présente normalement une pléiocytose lymphocytaire modérée, avec une glycorrhachie normale."}, "3": {"exist": true, "char_ranges": [[540, 655]], "word_ranges": [[79, 94]], "text": "la troisième parce que l'encéphalite limbique se présente normalement afébrile avec des changements de comportement"}, "4": {"exist": true, "char_ranges": [[659, 731]], "word_ranges": [[95, 107]], "text": "la dernière parce que l'image décrite n'est pas un abcès, bien délimité."}, "5": {"exist": false, "char_ranges": [], "word_ranges": [], "text": ""}}} +{"id": 116, "year": 2012, "question_id_specific": 101, "full_question": "Une femme de 25 ans souhaite être enceinte et veut savoir quel traitement elle doit suivre pendant sa grossesse éventuelle, car elle est porteuse hétérozygote du facteur V Leiden. Elle n'a jamais eu de phénomènes thrombotiques. La détermination de ce facteur a été réalisée dans le cadre d'une étude familiale à la suite d'un épisode d'embolie pulmonaire chez un frère ou une sœur. Quel traitement doit-on lui recommander ?", "full_answer": "En lisant les réponses, la première chose qui est claire pour nous est que l'acénocoumarol ne doit PAS être administré à une femme enceinte en raison du risque de tératogénicité. Quiconque le fait pendant l'internat devrait être puni de 100 claques dans le dos. Nous écartons la réponse 5. Deuxièmement : si un traitement est administré, il devrait l'être tout au long de la grossesse, l'état prothrombotique n'est pas seulement pendant la puerpéralité, donc la réponse 2 est écartée. Conseiller à une femme de ne pas tomber enceinte alors qu'il existe des traitements qui garantissent un risque très faible de thrombose et/ou d'atteinte fœtale ? Réponse 1 écartée. Il reste la possibilité de ne pas donner de traitement parce qu'il est à faible risque ou d'administrer de l'aspirine à faible dose. Il est vrai que le facteur V Leiden en hétérozygotie est classé à faible risque, sans antécédent thrombotique ni combinaison avec une autre thrombophilie ; Si nous consultons le guide CHEST, VIIIe édition, et que nous revoyons ce qui a été dit lors du dernier congrès de la Société espagnole de gynécologie et d'obstétrique, nous constatons que l'aspirine est utilisée en cas de syndrome phospholipidique et que dans le cas de cette femme, sans antécédents de thrombose, sans antécédents de fausses couches (au moins deux sont nécessaires) et étant également hétérozygote, le traitement n'est pas nécessaire. Réponse correcte, 3.", "type": "HÉMATOLOGIE", "options": {"1": "La grossesse étant un état prothrombotique, le risque de thromboembolie veineuse est élevé et la grossesse doit être déconseillée.", "2": "Un traitement à l'héparine de faible poids moléculaire à des doses prophylactiques dans la période post-partum immédiate doit être entrepris, avec un suivi facultatif pendant la grossesse.", "3": "Le facteur V Leiden hétérozygote est une thrombophilie à faible risque et aucun traitement n'est nécessaire pendant la grossesse et la puerpéralité.", "4": "L'aspirine à faible dose doit être conseillée pendant la grossesse et la période postnatale.", "5": "Le traitement par antivitamine K (acénocoumarol) doit être entrepris pendant la grossesse."}, "correct_option": 3, "explanations": {"1": {"exist": true, "char_ranges": [[485, 646]], "word_ranges": [[79, 105]], "text": "Conseiller à une femme de ne pas tomber enceinte alors qu'il existe des traitements qui garantissent un risque très faible de thrombose et/ou d'atteinte fœtale ?"}, "2": {"exist": true, "char_ranges": [[305, 452]], "word_ranges": [[51, 73]], "text": "si un traitement est administré, il devrait l'être tout au long de la grossesse, l'état prothrombotique n'est pas seulement pendant la puerpéralité,"}, "3": {"exist": true, "char_ranges": [[1124, 1407]], "word_ranges": [[185, 227]], "text": "nous constatons que l'aspirine est utilisée en cas de syndrome phospholipidique et que dans le cas de cette femme, sans antécédents de thrombose, sans antécédents de fausses couches (au moins deux sont nécessaires) et étant également hétérozygote, le traitement n'est pas nécessaire."}, "4": {"exist": true, "char_ranges": [[1124, 1407]], "word_ranges": [[185, 227]], "text": "nous constatons que l'aspirine est utilisée en cas de syndrome phospholipidique et que dans le cas de cette femme, sans antécédents de thrombose, sans antécédents de fausses couches (au moins deux sont nécessaires) et étant également hétérozygote, le traitement n'est pas nécessaire."}, "5": {"exist": true, "char_ranges": [[75, 178]], "word_ranges": [[14, 30]], "text": "l'acénocoumarol ne doit PAS être administré à une femme enceinte en raison du risque de tératogénicité."}}} +{"id": 473, "year": 2020, "question_id_specific": 141, "full_question": "Diagnostic le plus probable chez un patient de 74 ans qui souffre depuis deux mois de douleurs lombaires irradiant vers les membres inférieurs, d'une claudication neurogène et d'une limitation de l'extension du tronc :", "full_answer": "On nous montre une image de sténose du canal lombaire (Réponse 4 correcte). Le patient présente le tableau clinique caractéristique : douleur lombaire soulagée par la flexion du tronc en avant, douleur radiculaire avec atteinte des membres inférieurs et claudication neurogène. Une douleur lombaire chez un patient de plus de 60 ans accompagnée d'une faiblesse de la jambe lors de la marche, qui doit être interrompue à une certaine distance, appelée claudication neurogène, est le tableau typique d'une sténose. Le patient est soulagé par la flexion de la colonne vertébrale vers l'avant car elle élargit le canal lombaire et s'aggrave avec l'extension...", "type": "CHIRURGIE ORTHOPÉDIQUE ET TRAUMATOLOGIE", "options": {"1": "Hernie discale à L4-L5.", "2": "Fracture vertébrale lombaire.", "3": "Instabilité vertébrale L5-S1.", "4": "Sténose du canal lombaire.", "5": null}, "correct_option": 4, "explanations": {"1": {"exist": false, "char_ranges": [], "word_ranges": [], "text": ""}, "2": {"exist": false, "char_ranges": [], "word_ranges": [], "text": ""}, "3": {"exist": false, "char_ranges": [], "word_ranges": [], "text": ""}, "4": {"exist": true, "char_ranges": [[0, 277]], "word_ranges": [[0, 41]], "text": "On nous montre une image de sténose du canal lombaire (Réponse 4 correcte). Le patient présente le tableau clinique caractéristique : douleur lombaire soulagée par la flexion du tronc en avant, douleur radiculaire avec atteinte des membres inférieurs et claudication neurogène."}, "5": {"exist": false, "char_ranges": [], "word_ranges": [], "text": ""}}} +{"id": 34, "year": 2011, "question_id_specific": 66, "full_question": "Une femme de 51 ans se présente au service des urgences avec une baisse soudaine de l'acuité visuelle, des céphalées sévères, des nausées et des vomissements. Elle est hypotendue et afébrile. Elle présente une ophtalmoparésie droite due à une atteinte du troisième nerf crânien. Un scanner crânien montre une masse dans la région hyperdense de Selar avec une érosion des apophyses clinoïdes antérieures. Quel est le meilleur plan d'action ?", "full_answer": "L'apoplexie hypophysaire est un syndrome caractérisé par l'apparition soudaine de céphalées accompagnées de troubles de la motilité oculaire et d'un degré variable d'insuffisance hypophysaire. La cause est généralement une masse hypophysaire qui grossit rapidement et qui est secondaire à un événement vasculaire (infarctus ou hémorragie). Les symptômes généraux vont des vomissements ou des nausées à l'irritation méningée. La compression du chiasma et du nerf optique peut entraîner divers déficits campimétriques (généralement une hémianopsie bitemporale plus importante dans les quadrants supérieurs) et une diminution de l'acuité visuelle, voire la cécité. Si la masse se développe latéralement par rapport à la selle turque, elle peut comprimer les nerfs oculomoteurs, le nerf oculaire moteur commun étant le plus fréquemment touché, en raison de son parcours plus médian dans le sinus caverneux. L'examen de choix pour le diagnostic de l'apoplexie hypophysaire est l'IRM, car la tomodensitométrie peut ne pas distinguer la région assez clairement pour distinguer les changements dégénératifs ou kystiques des hémorragies antérieures. La découverte d'une érosion des apophyses clinoïdes antérieures au scanner du crâne peut confirmer l'existence d'une lésion chronique qui, à la suite d'un phénomène vasculaire aigu, a provoqué la symptomatologie. La prise en charge de ces patients implique un contrôle strict des troubles hydriques et électrolytiques et la correction des déficits hormonaux. Dans les situations instables, comme dans le cas décrit ici, avec une déficience visuelle importante, la chirurgie est généralement le traitement de choix, avec une décompression transsphénoïdale.", "type": "NEUROLOGIE ET NEUROCHIRURGIE", "options": {"1": "Je suspecterais une méningite chimique résultant de la rupture d'une tumeur épidermoïde et commencerais immédiatement un traitement aux corticostéroïdes.", "2": "Il conviendrait d'effectuer une angiographie cérébrale pour écarter la possibilité d'un anévrisme, car il est très probable que nous ayons affaire à un cas d'hémorragie sous-arachnoïdienne et que la masse observée au scanner soit un anévrisme parasellaire thrombosé.", "3": "Admission aux soins intensifs et traitement du choc subi par le patient et, une fois stabilisé, IRM cérébrale en vue d'une intervention chirurgicale programmée.", "4": "Biochimie et hémogramme urgents, instauration d'une corticothérapie à forte dose et chirurgie transsphénoïdale urgente.", "5": "Ponction lombaire pour exclure une méningite bactérienne après avoir commencé une antibiothérapie empirique. Une fois le patient stabilisé, une étude de la masse sélaire est réalisée."}, "correct_option": 5, "explanations": {"1": {"exist": false, "char_ranges": [], "word_ranges": [], "text": ""}, "2": {"exist": false, "char_ranges": [], "word_ranges": [], "text": ""}, "3": {"exist": false, "char_ranges": [], "word_ranges": [], "text": ""}, "4": {"exist": false, "char_ranges": [], "word_ranges": [], "text": ""}, "5": {"exist": true, "char_ranges": [[1500, 1696]], "word_ranges": [[212, 239]], "text": "Dans les situations instables, comme dans le cas décrit ici, avec une déficience visuelle importante, la chirurgie est généralement le traitement de choix, avec une décompression transsphénoïdale."}}} +{"id": 343, "year": 2016, "question_id_specific": 158, "full_question": "Une femme de 24 ans, primigeste, fait une fausse couche spontanée à 7 semaines de grossesse. L'étude anatomopathologique des restes avortés indique une maladie molaire. Nous devons l'informer que :", "full_answer": "La réponse est 2. Selon la SEGO (Société espagnole de gynécologie et d'obstétrique) dans son protocole de 2005 \"Maladie trophoblastique gestationnelle\", le risque de malignisation, c'est-à-dire de développement d'un néoplasme trophoblastique gestationnel, varie de 5 à 20 % selon qu'il s'agit d'une môle partielle ou complète, respectivement. Selon ce protocole : \"Les patientes seront surveillées chaque semaine avec un dosage d'hcg jusqu'à ce qu'il devienne indétectable, pendant trois fois consécutives. Ensuite, le suivi sera mensuel pendant six mois, puis tous les deux mois pendant six mois supplémentaires. Sur la base des données disponibles, un suivi de trois à six mois est recommandé pour une mola partielle et de 12 mois pour une mola complète\". Le risque d'une nouvelle gestation molaire, bien qu'augmenté par rapport à la population normale, n'est pas de 50 %.", "type": "GYNÉCOLOGIE ET OBSTÉTRIQUE", "options": {"1": "Le risque d'une nouvelle gestation molaire lors d'une prochaine grossesse est de 50 %.", "2": "Vous ne devez pas tomber enceinte avant d'avoir subi des examens réguliers et d'avoir passé un an avec des taux de BHCG négatifs.", "3": "Aucune surveillance supplémentaire n'est nécessaire si l'évacuation du tissu trophoblastique a été complète.", "4": "Des contrôles réguliers sont nécessaires car dans 40% des cas, une néoplasie trophoblastique gestationnelle se développe.", "5": null}, "correct_option": 2, "explanations": {"1": {"exist": false, "char_ranges": [], "word_ranges": [], "text": ""}, "2": {"exist": true, "char_ranges": [[153, 613]], "word_ranges": [[21, 87]], "text": "le risque de malignisation, c'est-à-dire de développement d'un néoplasme trophoblastique gestationnel, varie de 5 à 20 % selon qu'il s'agit d'une môle partielle ou complète, respectivement. Selon ce protocole : \"Les patientes seront surveillées chaque semaine avec un dosage d'hcg jusqu'à ce qu'il devienne indétectable, pendant trois fois consécutives. Ensuite, le suivi sera mensuel pendant six mois, puis tous les deux mois pendant six mois supplémentaires."}, "3": {"exist": false, "char_ranges": [], "word_ranges": [], "text": ""}, "4": {"exist": false, "char_ranges": [], "word_ranges": [], "text": ""}, "5": {"exist": false, "char_ranges": [], "word_ranges": [], "text": ""}}} +{"id": 210, "year": 2014, "question_id_specific": 187, "full_question": "Une femme de 27 ans, enceinte de 10 semaines, souffrant d'asthme allergique persistant sévère. Elle est actuellement bien contrôlée par le budésonide inhalé quotidien et le salbutamol inhalé de secours à la demande. Elle se présente à votre cabinet, préoccupée par les effets tératogènes possibles de son traitement anti-asthmatique. Laquelle des mesures suivantes serait la bonne ?", "full_answer": "- Budésonide inhalé : évaluation des bénéfices/risques. Les résultats d'une vaste étude épidémiologique prospective et l'expérience étendue acquise après l'autorisation de mise sur le marché indiquent que le budésonide inhalé pendant la grossesse n'a pas d'effets indésirables sur la santé du fœtus et du nouveau-né. Catégorie B. - Salbutamol inh : N'utiliser que si les bénéfices pour la mère l'emportent sur les risques pour le fœtus. Catégorie C.", "type": "GYNÉCOLOGIE ET OBSTÉTRIQUE", "options": {"1": "Comme l'asthme s'améliore pendant la grossesse chez la plupart des patientes, il est préférable pour la patiente et le fœtus d'arrêter le traitement de l'asthme.", "2": "Arrêter le budésonide car il a été associé à un risque accru de malformations fœtales et le remplacer par un anti-leucotriène oral (montelukast).", "3": "Interrompre le traitement en cours et le remplacer par de la prednisone orale à la dose la plus faible possible.", "4": "Maintenir le traitement actuel et rassurer la patiente sur ses effets secondaires et sur la nécessité d'un contrôle adéquat de l'asthme pendant la grossesse.", "5": "Remplacer le budésonide par un anticorps monoclonal anti-IgE (omalizumab) car il est plus sûr pendant la grossesse puisqu'il ne s'agit pas d'un médicament."}, "correct_option": 4, "explanations": {"1": {"exist": false, "char_ranges": [], "word_ranges": [], "text": ""}, "2": {"exist": false, "char_ranges": [], "word_ranges": [], "text": ""}, "3": {"exist": false, "char_ranges": [], "word_ranges": [], "text": ""}, "4": {"exist": true, "char_ranges": [[56, 316]], "word_ranges": [[7, 45]], "text": "Les résultats d'une vaste étude épidémiologique prospective et l'expérience étendue acquise après l'autorisation de mise sur le marché indiquent que le budésonide inhalé pendant la grossesse n'a pas d'effets indésirables sur la santé du fœtus et du nouveau-né."}, "5": {"exist": false, "char_ranges": [], "word_ranges": [], "text": ""}}} +{"id": 187, "year": 2013, "question_id_specific": 68, "full_question": "Une femme de 52 ans, originaire d'un village de la Costa Brava, remarque lors de l'application d'une crème sur son cou une grosseur dans la région antérieure, par ailleurs asymptomatique ; elle consulte son médecin traitant qui confirme la présence d'une masse ferme et lisse de 2 cm de diamètre maximum, qui se soulève lors de la déglutition, sans adénopathie palpable. Pas d'adénopathie palpable. Quels examens demanderiez-vous d'emblée ?", "full_answer": "Il s'agit d'un nodule thyroïdien et le test indiqué est une FNA (ponction à l'aiguille fine). J'ai lu la réponse écrite par Emilio au sujet du défi ; la question se trouve dans le bloc endocrinien et il semble qu'il essaie de préciser qu'il s'agit d'un nodule. L'échographie facilite la FNA mais n'est pas obligatoire, un nodule thyroïdien de 2 cm peut être ponctionné sans écho. Il est important de procéder à une analyse de la fonction thyroïdienne avant la ponction ; mais comme il parle d'une patiente asymptomatique, il semble clair qu'elle n'est pas hyperthyroïdienne. Je ne pense pas que cela soit contesté et je pense que c'est une question à laquelle la plupart des gens auront bien répondu.", "type": "ENDOCRINOLOGIE", "options": {"1": "Un dosage de la thyroglobuline dans le sang.", "2": "Un scanner cervical.", "3": "Une détermination des anticorps anti-thyroïdiens circulants (anti-thyroglubuline et anti-peroxydase).", "4": "Une ponction à l'aiguille fine.", "5": "Une détermination de la T3 libre."}, "correct_option": 4, "explanations": {"1": {"exist": false, "char_ranges": [], "word_ranges": [], "text": ""}, "2": {"exist": false, "char_ranges": [], "word_ranges": [], "text": ""}, "3": {"exist": false, "char_ranges": [], "word_ranges": [], "text": ""}, "4": {"exist": true, "char_ranges": [[0, 93]], "word_ranges": [[0, 16]], "text": "Il s'agit d'un nodule thyroïdien et le test indiqué est une FNA (ponction à l'aiguille fine)."}, "5": {"exist": false, "char_ranges": [], "word_ranges": [], "text": ""}}} +{"id": 406, "year": 2016, "question_id_specific": 33, "full_question": "Une femme de 67 ans a été diagnostiquée avec un carcinome canalaire infiltrant du sein sans antécédents familiaux de néoplasie. Quelles sont les études complémentaires à réaliser sur la tumeur pour ses implications cliniques et thérapeutiques ?", "full_answer": "Actuellement, la classification moléculaire du cancer du sein est basée sur l'étude des récepteurs hormonaux, de HER2 et de l'indice de prolifération des cellules tumorales (Ki67). Nous avons quatre types fondamentaux : Luminal (A : HRH+, HER2- et Ki67 bas et B : HRH+, HER2+/ - et Ki67 élevé), HER2 + (HR -, HER2 + et Ki67 élevé) et Basal Like ou triple négatif (HR -, HER2 - et Ki67 élevé).", "type": "ONCOLOGIE (ECTOPIQUE)", "options": {"1": "Étude phénotypique complète par cytométrie de flux.", "2": "Étude des récepteurs hormonaux et de HER2.", "3": "Étude des récepteurs hormonaux, de l'ecadhérine et étude des parents au premier degré.", "4": "Étude BRCA l-2 et étude des parents au premier degré.", "5": null}, "correct_option": 2, "explanations": {"1": {"exist": false, "char_ranges": [], "word_ranges": [], "text": ""}, "2": {"exist": true, "char_ranges": [[0, 180]], "word_ranges": [[0, 26]], "text": "Actuellement, la classification moléculaire du cancer du sein est basée sur l'étude des récepteurs hormonaux, de HER2 et de l'indice de prolifération des cellules tumorales (Ki67)."}, "3": {"exist": false, "char_ranges": [], "word_ranges": [], "text": ""}, "4": {"exist": false, "char_ranges": [], "word_ranges": [], "text": ""}, "5": {"exist": false, "char_ranges": [], "word_ranges": [], "text": ""}}} +{"id": 526, "year": 2021, "question_id_specific": 54, "full_question": "Une femme de 86 ans souffre de fibrillation auriculaire non valvulaire. Son score CHADS2 est de 3 points. Dans la littérature, des patients similaires sous warfarine ont un risque d'AVC de 2,2 % contre 5,2 % chez les patients sans warfarine. Quel serait le nombre nécessaire à traiter (NNT) pour prévenir un AVC embolique avec un traitement anticoagulant ?", "full_answer": "Le NNT (nombre nécessaire à traiter) indique le nombre de patients qu'il faudrait traiter avec une intervention pour obtenir un bénéfice ; dans cette question, combien de patients devrions-nous traiter avec de la warfarine pour prévenir les accidents vasculaires cérébraux ? Pour calculer ce nombre, nous utilisons la formule suivante : NNT= 1/RRA (RRA : réduction du risque absolu). RRA= Io - Ie (Io : incidence chez les patients non exposés à la warfarine ; Ie : incidence chez les patients exposés à la warfarine). RRA : 5,2%- 2,2%= 3= 0,03. NNT : 1/0,03= 33,3.", "type": "MÉDECINE PRÉVENTIVE", "options": {"1": "3.", "2": "19,2.", "3": "33,3.", "4": "49,5.", "5": null}, "correct_option": 3, "explanations": {"1": {"exist": false, "char_ranges": [], "word_ranges": [], "text": ""}, "2": {"exist": false, "char_ranges": [], "word_ranges": [], "text": ""}, "3": {"exist": true, "char_ranges": [[0, 564]], "word_ranges": [[0, 95]], "text": "Le NNT (nombre nécessaire à traiter) indique le nombre de patients qu'il faudrait traiter avec une intervention pour obtenir un bénéfice ; dans cette question, combien de patients devrions-nous traiter avec de la warfarine pour prévenir les accidents vasculaires cérébraux ? Pour calculer ce nombre, nous utilisons la formule suivante : NNT= 1/RRA (RRA : réduction du risque absolu). RRA= Io - Ie (Io : incidence chez les patients non exposés à la warfarine ; Ie : incidence chez les patients exposés à la warfarine). RRA : 5,2%- 2,2%= 3= 0,03. NNT : 1/0,03= 33,3."}, "4": {"exist": false, "char_ranges": [], "word_ranges": [], "text": ""}, "5": {"exist": false, "char_ranges": [], "word_ranges": [], "text": ""}}} +{"id": 6, "year": 2011, "question_id_specific": 45, "full_question": "Un homme de 30 ans, dont le père est décédé à l'âge de 38 ans d'un cancer du côlon, subit une coloscopie qui révèle des centaines d'adénomes dans le côlon. Une coloscopie est effectuée et montre des centaines d'adénomes dans tout le côlon. Laquelle des affirmations suivantes est fausse ?", "full_answer": "Patient atteint de polypose adénomateuse familiale. Toutes les affirmations sont correctes, sauf 2. Le traitement doit être chirurgical en cas de polypose. L'âge de 40 ans est le point habituel d'incidence du cancer.", "type": "DIGESTIF", "options": {"1": "Le patient est atteint de polypose adénomateuse familiale.", "2": "La prise en charge la plus appropriée est une coloscopie annuelle de contrôle et une colectomie à l'âge de 40 ans.", "3": "Sans traitement chirurgical, le patient développera presque à coup sûr un cancer colorectal.", "4": "Les parents au premier degré doivent être étudiés.", "5": "Les enfants du patient ont un risque de 50% de développer la même maladie."}, "correct_option": 2, "explanations": {"1": {"exist": true, "char_ranges": [[0, 216]], "word_ranges": [[0, 33]], "text": "Patient atteint de polypose adénomateuse familiale. Toutes les affirmations sont correctes, sauf 2. Le traitement doit être chirurgical en cas de polypose. L'âge de 40 ans est le point habituel d'incidence du cancer."}, "2": {"exist": true, "char_ranges": [[0, 216]], "word_ranges": [[0, 33]], "text": "Patient atteint de polypose adénomateuse familiale. Toutes les affirmations sont correctes, sauf 2. Le traitement doit être chirurgical en cas de polypose. L'âge de 40 ans est le point habituel d'incidence du cancer."}, "3": {"exist": true, "char_ranges": [[0, 216]], "word_ranges": [[0, 33]], "text": "Patient atteint de polypose adénomateuse familiale. Toutes les affirmations sont correctes, sauf 2. Le traitement doit être chirurgical en cas de polypose. L'âge de 40 ans est le point habituel d'incidence du cancer."}, "4": {"exist": true, "char_ranges": [[0, 216]], "word_ranges": [[0, 33]], "text": "Patient atteint de polypose adénomateuse familiale. Toutes les affirmations sont correctes, sauf 2. Le traitement doit être chirurgical en cas de polypose. L'âge de 40 ans est le point habituel d'incidence du cancer."}, "5": {"exist": true, "char_ranges": [[0, 216]], "word_ranges": [[0, 33]], "text": "Patient atteint de polypose adénomateuse familiale. Toutes les affirmations sont correctes, sauf 2. Le traitement doit être chirurgical en cas de polypose. L'âge de 40 ans est le point habituel d'incidence du cancer."}}} +{"id": 573, "year": 2022, "question_id_specific": 92, "full_question": "Une femme de 75 ans est amenée en consultation par sa famille qui la trouve déprimée et avec des trous de mémoire depuis des mois. Ils craignent qu'elle ne soit atteinte de la maladie d'Alzheimer. La patiente dit qu'elle ne pense pas que quelque chose ne va pas et qu'elle est comme d'habitude. Dans quel ordre les symptômes suivants apparaissent-ils généralement dans l'évolution de la maladie d'Alzheimer ?", "full_answer": "Je réponds à cette question en pensant davantage à ce qu'ils veulent demander qu'à ce qu'ils demandent réellement. Le développement des symptômes de la maladie d'Alzheimer est hétérogène. Il existe des variantes comportementales, mais dans le MIR, on veut généralement que vous pensiez à d'autres types de démence lorsque vous êtes confrontés à des symptômes comportementaux. En fait, l'apparition de ces symptômes à un stade précoce devrait nous alerter et ouvrir l'éventail des possibilités de diagnostic. Entre 1 et 3, je choisis 3 pour la même raison et parce que l'accent est mis sur le déclin de l'indépendance fonctionnelle qui se produit chez ces patients.", "type": "NEUROLOGIE", "options": {"1": "Changements d'humeur, symptômes comportementaux, déficits cognitifs.", "2": "Symptômes comportementaux, symptômes moteurs, déclin de l'indépendance fonctionnelle.", "3": "Changements d'humeur, déficits cognitifs, déclin de l'indépendance fonctionnelle.", "4": "Symptômes comportementaux, sautes d'humeur, symptômes moteurs.", "5": null}, "correct_option": 3, "explanations": {"1": {"exist": true, "char_ranges": [[508, 664]], "word_ranges": [[76, 105]], "text": "Entre 1 et 3, je choisis 3 pour la même raison et parce que l'accent est mis sur le déclin de l'indépendance fonctionnelle qui se produit chez ces patients."}, "2": {"exist": true, "char_ranges": [[188, 507]], "word_ranges": [[28, 76]], "text": "Il existe des variantes comportementales, mais dans le MIR, on veut généralement que vous pensiez à d'autres types de démence lorsque vous êtes confrontés à des symptômes comportementaux. En fait, l'apparition de ces symptômes à un stade précoce devrait nous alerter et ouvrir l'éventail des possibilités de diagnostic."}, "3": {"exist": true, "char_ranges": [[508, 664]], "word_ranges": [[76, 105]], "text": "Entre 1 et 3, je choisis 3 pour la même raison et parce que l'accent est mis sur le déclin de l'indépendance fonctionnelle qui se produit chez ces patients."}, "4": {"exist": true, "char_ranges": [[188, 507]], "word_ranges": [[28, 76]], "text": "Il existe des variantes comportementales, mais dans le MIR, on veut généralement que vous pensiez à d'autres types de démence lorsque vous êtes confrontés à des symptômes comportementaux. En fait, l'apparition de ces symptômes à un stade précoce devrait nous alerter et ouvrir l'éventail des possibilités de diagnostic."}, "5": {"exist": false, "char_ranges": [], "word_ranges": [], "text": ""}}} +{"id": 164, "year": 2013, "question_id_specific": 123, "full_question": "Un patient de 80 ans, ayant des antécédents d'hypertension et traité à l'énalapril et à la spironolactone, arrive à l'hôpital avec une asthénie et une faiblesse musculaire sévère. La tension artérielle est de 110/70 mmHg. L'ECG montre des ondes T élevées et nettes, des extrasystoles ventriculaires et un QT court. Quel est le diagnostic le plus probable ?", "full_answer": "La bonne réponse est : 2. Hyperkaliémie. La spironolactone, en tant que diurétique d'épargne potassique, inhibe l'action de l'aldostérone au niveau rénal et est responsable d'une diminution de l'excrétion de potassium. L'hyperkaliémie entraîne des troubles de la repolarisation qui se traduisent par l'apparition d'un T large, symétrique, à base étroite, visible dans les dérivations précordiales. L'espace QT est raccourci. Ces anomalies apparaissent pour une kaliémie de l'ordre de 5,5 à 6 mmol/l. Au-delà de 6,5 mmol/l, les modifications électrocardiographiques sont constantes et dominées par les troubles de la conduction (rendant possibles les extrasystoles ventriculaires).", "type": "NEPHROLOGIE", "options": {"1": "Hypercalcémie.", "2": "Hyperkaliémie.", "3": "Hypomagnésémie.", "4": "Hypocalcémie.", "5": "Hypernatrémie."}, "correct_option": 2, "explanations": {"1": {"exist": false, "char_ranges": [], "word_ranges": [], "text": ""}, "2": {"exist": true, "char_ranges": [[219, 397]], "word_ranges": [[31, 55]], "text": "L'hyperkaliémie entraîne des troubles de la repolarisation qui se traduisent par l'apparition d'un T large, symétrique, à base étroite, visible dans les dérivations précordiales."}, "3": {"exist": false, "char_ranges": [], "word_ranges": [], "text": ""}, "4": {"exist": false, "char_ranges": [], "word_ranges": [], "text": ""}, "5": {"exist": false, "char_ranges": [], "word_ranges": [], "text": ""}}} +{"id": 208, "year": 2014, "question_id_specific": 183, "full_question": "Une femme enceinte de 40 semaines est en train d'accoucher avec une dilatation de 6 cm. Elle présentait un schéma fœtal en décélération dans l'enregistrement cardiotocographique et il a été décidé de procéder à un micro-prélèvement de sang fœtal pour évaluer l'état du fœtus. Résultat 7.22. La conduite à tenir est la suivante :", "full_answer": "- pH inférieur à 7,20 Valeur pathologique. Indication d'extraction du fœtus par la voie la plus rapide, en l'occurrence une césarienne. - pH compris entre 7,20 et 7,25 Valeur pré-pathologique. Répéter le microtome dans 15-20 min. - pH supérieur à 7,25. Valeur dans les limites de la normalité. Observation.", "type": "GYNÉCOLOGIE ET OBSTÉTRIQUE", "options": {"1": "Acidose sévère. Césarienne urgente.", "2": "Valeur pré-pathologique répéter le microtome 15-20 minutes.", "3": "Acidose modérée. Répéter le microtome dans 1 à 2 heures.", "4": "Valeur dans les limites normales, laisser l'évolution naturelle de la naissance.", "5": "Répétez l'opération dès qu'il y a une erreur possible dans l'obtention du cliché."}, "correct_option": 2, "explanations": {"1": {"exist": false, "char_ranges": [], "word_ranges": [], "text": ""}, "2": {"exist": true, "char_ranges": [[138, 229]], "word_ranges": [[22, 36]], "text": "pH compris entre 7,20 et 7,25 Valeur pré-pathologique. Répéter le microtome dans 15-20 min."}, "3": {"exist": false, "char_ranges": [], "word_ranges": [], "text": ""}, "4": {"exist": false, "char_ranges": [], "word_ranges": [], "text": ""}, "5": {"exist": false, "char_ranges": [], "word_ranges": [], "text": ""}}} +{"id": 232, "year": 2014, "question_id_specific": 103, "full_question": "Une jeune fille de 19 ans, sans antécédents médicaux particuliers, à l'exception d'une grippe autolimitée survenue trois semaines auparavant, s'est présentée aux urgences avec des pétéchies et des ecchymoses d'apparition spontanée. À l'examen physique, la patiente était en bon état général, afébrile, normotendue et orientée dans le temps et l'espace. Il présentait des pétéchies disséminées dans les membres inférieurs et l'abdomen, ainsi que de petites ecchymoses dans les zones de décubitus. Il n'y avait pas de ganglions lymphatiques palpables ni de splénomégalie. Les examens de laboratoire ont donné les résultats suivants : Hb 12,6 g/dL, leucocytes 5 500/mm3, plaquettes 7 000/mm3. Le frottis du sang périphérique a montré une morphologie normale des érythrocytes, une numération différentielle normale des leucocytes et une numération plaquettaire conforme à l'autoanalysateur sans agrégats plaquettaires. La biochimie et le protéinogramme, la bêta 2 microglobuline et la LDH sont normaux. Quel est, selon vous, le traitement initial le plus approprié ?", "full_answer": "Purpura thrombocytopénique idiopathique, mais avec un taux de plaquettes très bas, ce qui nécessite une reconstitution. La dose de stéroïdes proposée est trop faible.", "type": "HÉMATOLOGIE", "options": {"1": "Transfusion de plaquettes.", "2": "Rituximab hebdomadaire.", "3": "Cyclophosphamide en impulsions de 4 jours tous les 21 jours.", "4": "Plasmaphérèse quotidienne.", "5": "Prednisone à 1 mg/jour pendant 2 à 3 semaines."}, "correct_option": 1, "explanations": {"1": {"exist": true, "char_ranges": [[0, 119]], "word_ranges": [[0, 16]], "text": "Purpura thrombocytopénique idiopathique, mais avec un taux de plaquettes très bas, ce qui nécessite une reconstitution."}, "2": {"exist": false, "char_ranges": [], "word_ranges": [], "text": ""}, "3": {"exist": false, "char_ranges": [], "word_ranges": [], "text": ""}, "4": {"exist": false, "char_ranges": [], "word_ranges": [], "text": ""}, "5": {"exist": true, "char_ranges": [[120, 166]], "word_ranges": [[16, 24]], "text": "La dose de stéroïdes proposée est trop faible."}}} +{"id": 64, "year": 2011, "question_id_specific": 210, "full_question": "Une femme de 42 ans atteinte d'un cancer du sein présente des métastases au niveau des foramina intervertébraux entre les 4e et 5e vertèbres cervicales et entre les 4e et 5e vertèbres thoraciques. Quels sont les nerfs rachidiens concernés ?", "full_answer": "L'astuce de cette question est qu'il y a 8 racines cervicales pour 7 vertèbres, donc la racine C1 sort AU-DESSUS de l'atlas, et ainsi de suite jusqu'à C8 qui sort ENTRE C7 et T1. Les racines thoraciques sortent respectivement sous la vertèbre avec la même numérotation (nerf T1 sous T1, T2 sous T2, etc.). Ainsi, entre C4 et C5 sort le 5e nerf cervical et entre T4 et T5 sort le 4e nerf thoracique. La bonne réponse est 4.", "type": "ANATOMIE", "options": {"1": "Quatrième nerf cervical et quatrième nerf thoracique.", "2": "Cinquième nerf cervical et cinquième nerf thoracique.", "3": "Quatrième nerf cervical et cinquième nerf thoracique.", "4": "Cinquième nerf cervical et quatrième nerf thoracique.", "5": "Troisième nerf cervical et quatrième nerf thoracique."}, "correct_option": 4, "explanations": {"1": {"exist": false, "char_ranges": [], "word_ranges": [], "text": ""}, "2": {"exist": false, "char_ranges": [], "word_ranges": [], "text": ""}, "3": {"exist": false, "char_ranges": [], "word_ranges": [], "text": ""}, "4": {"exist": false, "char_ranges": [], "word_ranges": [], "text": ""}, "5": {"exist": false, "char_ranges": [], "word_ranges": [], "text": ""}}} +{"id": 280, "year": 2016, "question_id_specific": 32, "full_question": "Une femme de 59 ans s'est présentée avec une diarrhée aqueuse chronique de 4 mois. À l'endoscopie, la muqueuse ne présentait aucun aspect pertinent. En particulier, aucun ulcère ou zone friable n'a été observé. Une biopsie du côlon transverse a été réalisée. L'histopathologie a révélé une zone épaissie sous l'épithélium de revêtement superficiel, qui était plus évidente par la technique du trichrome de Masson et impliquait une atrophie et une dénudation de l'épithélium. Le diagnostic de la lésion intestinale est ?", "full_answer": "Tout ce qui nous est dit dans ce cas est typique de la COLITE COLLAGENIQUE. En principe, les résultats cliniques et endoscopiques peuvent nous laisser un peu comme au début, mais l'histologie n'est guère plus que la définition de cette pathologie. Le critère spécifique du diagnostic de colite collagénique est la présence supplémentaire d'une bande irrégulière de collagène juste sous l'épithélium de surface de la muqueuse colique en continuité avec la membrane basale, visible à la coloration trichrome (coloration des fibres de collagène de type I), qui emprisonne les capillaires superficiels en produisant des lésions dans l'épithélium de surface. En outre, elle s'accompagne d'un infiltrat inflammatoire chronique dans la lamina propria, composé principalement de lymphocytes, de plasmocytes et d'éosinophiles.", "type": "ANATOMIE PATHOLOGIQUE", "options": {"1": "Colite ulcéreuse chronique.", "2": "Colite pseudomembraneuse.", "3": "Colite collagénique.", "4": "Maladie de Crohn fibrosante.", "5": null}, "correct_option": 3, "explanations": {"1": {"exist": false, "char_ranges": [], "word_ranges": [], "text": ""}, "2": {"exist": false, "char_ranges": [], "word_ranges": [], "text": ""}, "3": {"exist": true, "char_ranges": [[248, 653]], "word_ranges": [[41, 99]], "text": "Le critère spécifique du diagnostic de colite collagénique est la présence supplémentaire d'une bande irrégulière de collagène juste sous l'épithélium de surface de la muqueuse colique en continuité avec la membrane basale, visible à la coloration trichrome (coloration des fibres de collagène de type I), qui emprisonne les capillaires superficiels en produisant des lésions dans l'épithélium de surface."}, "4": {"exist": false, "char_ranges": [], "word_ranges": [], "text": ""}, "5": {"exist": false, "char_ranges": [], "word_ranges": [], "text": ""}}} +{"id": 545, "year": 2022, "question_id_specific": 122, "full_question": "Un patient de 78 ans, sans antécédents, consulte pour une asthénie et une dyspnée qui durent depuis trois mois et qui ont évolué vers le repos au cours des derniers jours. Les jours précédents, il avait également signalé une douleur thoracique de type angineux lors d'un léger effort. L'examen révèle une tension artérielle à 110/80 mmHg, une fréquence cardiaque à 85 bpm et à l'auscultation un souffle systolique rugueux dans le deuxième espace intercostal droit et des crépitants dans les deux bases pulmonaires. Quel est le diagnostic le plus probable parmi les suivants ?", "full_answer": "Sténose aortique.", "type": "CARDIOLOGIE", "options": {"1": "Insuffisance mitrale.", "2": "Sténose aortique.", "3": "Insuffisance aortique.", "4": "Cardiomyopathie dilatée d'origine ischémique.", "5": null}, "correct_option": 2, "explanations": {"1": {"exist": false, "char_ranges": [], "word_ranges": [], "text": ""}, "2": {"exist": true, "char_ranges": [[0, 17]], "word_ranges": [[0, 2]], "text": "Sténose aortique."}, "3": {"exist": false, "char_ranges": [], "word_ranges": [], "text": ""}, "4": {"exist": false, "char_ranges": [], "word_ranges": [], "text": ""}, "5": {"exist": false, "char_ranges": [], "word_ranges": [], "text": ""}}} +{"id": 257, "year": 2014, "question_id_specific": 95, "full_question": "Une patiente de 30 ans présente un taux de calcium de 11 mg/dl (normale inférieure à 10,5 mg/dl) lors d'un examen de routine de l'entreprise. Le taux de PTH était de 45 pg/ml (VN 10-55 pg/ml). Les antécédents sont sans particularité, à l'exception du fait que la mère et le grand-père paternel ont été diagnostiqués avec une hyperparathyroïdie et ont subi une intervention chirurgicale, bien qu'ils soient restés hypercalcémiques. Quel est le test le plus utile pour confirmer le diagnostic ?", "full_answer": "Rapport calcium/créatinine dans les urines. Il s'agit d'une hypercalcémie hypocalciurique familiale ou d'une hypercalcémie familiale bénigne. Il s'agit d'une maladie autosomique dominante. Asymptomatique. Non curable après traitement chirurgical ; due à une mutation du récepteur sensible au calcium. Question difficile car il est nécessaire de connaître des données sur une maladie qui n'est pas habituellement posée dans le MIR.", "type": "ENDOCRINOLOGIE", "options": {"1": "25-OH D.", "2": "1,25-OH 2D.", "3": "Rapport calcium/créatinine dans les urines.", "4": "Réabsorption tubulaire du phosphate.", "5": "PTHrP."}, "correct_option": 3, "explanations": {"1": {"exist": false, "char_ranges": [], "word_ranges": [], "text": ""}, "2": {"exist": false, "char_ranges": [], "word_ranges": [], "text": ""}, "3": {"exist": true, "char_ranges": [[44, 300]], "word_ranges": [[5, 38]], "text": "Il s'agit d'une hypercalcémie hypocalciurique familiale ou d'une hypercalcémie familiale bénigne. Il s'agit d'une maladie autosomique dominante. Asymptomatique. Non curable après traitement chirurgical ; due à une mutation du récepteur sensible au calcium."}, "4": {"exist": false, "char_ranges": [], "word_ranges": [], "text": ""}, "5": {"exist": false, "char_ranges": [], "word_ranges": [], "text": ""}}} +{"id": 548, "year": 2022, "question_id_specific": 126, "full_question": "Un homme de 58 ans, souffrant d'hypertension depuis 6 ans, consulte pour un mauvais contrôle de sa pression artérielle malgré un traitement par inhibiteur de l'enzyme de conversion de l'angiotensine, diurétique et antagoniste du calcium. Lors de la consultation, il présente une tension artérielle de 149/100 mmHg. Examens de laboratoire : créatinine 1,2 mg/dl, potassium 2,2 mEq/l et alcalose métabolique compensée ; le reste de l'étude biochimique, l'hémogramme, la coagulation et le sédiment urinaire sont normaux. Cochez la bonne réponse :", "full_answer": "La tomodensitométrie fait partie du bilan diagnostique en cas de confirmation biochimique.", "type": "CARDIOLOGIE", "options": {"1": "L'origine de l'hypertension dans ce cas est une sécrétion excessive d'aldostérone causée par un hyperfonctionnement autonome de la médullosurrénale.", "2": "Dans la plupart des cas, le substrat anatomique est une hyperplasie bilatérale du cortex surrénalien.", "3": "La tomodensitométrie fait partie du bilan diagnostique en cas de confirmation biochimique.", "4": "La spironolactone est contre-indiquée dans la prise en charge de cette pathologie.", "5": null}, "correct_option": 3, "explanations": {"1": {"exist": false, "char_ranges": [], "word_ranges": [], "text": ""}, "2": {"exist": false, "char_ranges": [], "word_ranges": [], "text": ""}, "3": {"exist": true, "char_ranges": [[0, 90]], "word_ranges": [[0, 12]], "text": "La tomodensitométrie fait partie du bilan diagnostique en cas de confirmation biochimique."}, "4": {"exist": false, "char_ranges": [], "word_ranges": [], "text": ""}, "5": {"exist": false, "char_ranges": [], "word_ranges": [], "text": ""}}} +{"id": 318, "year": 2016, "question_id_specific": 140, "full_question": "Vous évaluez un patient de 66 ans qui présente des douleurs à l'aine accentuées par une station debout prolongée quelques jours par mois. La radiographie simple des hanches montre un rétrécissement de l'espace articulaire fémoro-acétabulaire, une sclérose et des ostéphytes.", "full_answer": "Il s'agit d'une coxarthrose. Les critères chirurgicaux dépendent des symptômes du patient et de l'échec du traitement médical. Aucun médicament ne ralentit ou ne modifie l'évolution de la maladie. L'IRM n'est pas nécessaire car elle n'apporte rien. Le traitement est médical et séquentiel, commençant par la première étape antalgique de l'OMS.", "type": "RHEUMATOLOGIE", "options": {"1": "Je pose un diagnostic de coxarthrose et envoie le traumatologue poser une prothèse de hanche.", "2": "Initiation d'un traitement avec des opioïdes faibles qui ont démontré leur capacité à stopper la progression de la maladie.", "3": "J'ai commencé le traitement avec du paracétamol, j'ai expliqué que l'évolution est très variable et que l'indication d'une intervention chirurgicale dépend de la fonctionnalité et du contrôle de la douleur.", "4": "En raison des caractéristiques radiologiques décrites, j'ai besoin d'une IRM de la hanche avant de prendre une décision thérapeutique.", "5": null}, "correct_option": 3, "explanations": {"1": {"exist": false, "char_ranges": [], "word_ranges": [], "text": ""}, "2": {"exist": true, "char_ranges": [[127, 196]], "word_ranges": [[18, 29]], "text": "Aucun médicament ne ralentit ou ne modifie l'évolution de la maladie."}, "3": {"exist": true, "char_ranges": [[249, 343]], "word_ranges": [[37, 51]], "text": "Le traitement est médical et séquentiel, commençant par la première étape antalgique de l'OMS."}, "4": {"exist": true, "char_ranges": [[197, 248]], "word_ranges": [[29, 37]], "text": "L'IRM n'est pas nécessaire car elle n'apporte rien."}, "5": {"exist": false, "char_ranges": [], "word_ranges": [], "text": ""}}} +{"id": 147, "year": 2012, "question_id_specific": 157, "full_question": "Une étudiante universitaire de 19 ans se présente à la clinique accompagnée de ses parents et déclare s'être sentie progressivement plus asthénique au cours des deux derniers mois, avec une perte d'appétit et de poids et de plus grandes difficultés à se concentrer sur ses études. L'anamnèse souligne également qu'elle a perdu l'envie de sortir avec ses amis, qu'elle a des idées de mort sans idées suicidaires et des cognitions pessimistes quant à l'avenir. Son poids correspond à 90 % de ce qui est considéré comme idéal pour son âge et son sexe. Elle ne présente pas de phobie du poids ni de distorsion de l'image corporelle. Le diagnostic le plus approprié est le suivant :", "full_answer": "Dès que nous entendons parler de poids, nous pensons à l'anorexie mentale, mais l'anorexie mentale présente trois caractéristiques : une perte de poids importante auto-induite, une distorsion de l'image corporelle et l'aménorrhée. Supprimons donc le numéro 1. 3 n'a pas de sens dans le MIR à cause de la clinique -dans la réalité l'histoire est différente et en psychiatrie tout est évolution-. Le 4 doit répondre à des critères temporels qui ne sont pas remplis ici. Nous sommes donc entre 2 et 5. Dépression ou anxiété ? Il n'y a pas d'anxiété clinique et elle répond également aux critères de dépression. La pauvre fille a été diagnostiquée comme souffrant de dépression. La réponse est 5 (une autre option est qu'elle étudiait le MIR).", "type": "PSYCHIATRIE", "options": {"1": "Anorexie mentale.", "2": "Troubles anxieux.", "3": "Le trouble de la personnalité limite.", "4": "Dysthymie.", "5": "Trouble dépressif majeur."}, "correct_option": 5, "explanations": {"1": {"exist": true, "char_ranges": [[133, 259]], "word_ranges": [[19, 37]], "text": "une perte de poids importante auto-induite, une distorsion de l'image corporelle et l'aménorrhée. Supprimons donc le numéro 1."}, "2": {"exist": true, "char_ranges": [[523, 607]], "word_ranges": [[87, 101]], "text": "Il n'y a pas d'anxiété clinique et elle répond également aux critères de dépression."}, "3": {"exist": true, "char_ranges": [[260, 394]], "word_ranges": [[37, 62]], "text": "3 n'a pas de sens dans le MIR à cause de la clinique -dans la réalité l'histoire est différente et en psychiatrie tout est évolution-."}, "4": {"exist": true, "char_ranges": [[395, 467]], "word_ranges": [[62, 76]], "text": "Le 4 doit répondre à des critères temporels qui ne sont pas remplis ici."}, "5": {"exist": true, "char_ranges": [[523, 607]], "word_ranges": [[87, 101]], "text": "Il n'y a pas d'anxiété clinique et elle répond également aux critères de dépression."}}} +{"id": 530, "year": 2021, "question_id_specific": 167, "full_question": "Un patient atteint d'une maladie rénale chronique avancée connue (CKD G4, eGFR 20 ml/min), qui se présente au service des urgences pour une faiblesse générale et qui présente une hyperkaliémie sévère (K 7 mEq/l) avec des altérations électrocardiographiques. Quelle serait la première mesure à prendre ?", "full_answer": "On nous décrit le cas d'un patient présentant une urgence hyperkaliémique (potassium >6,5 mmol/l avec altérations de l'ECG). La première étape consiste à administrer du gluconate de calcium par voie intraveineuse pour contrer la toxicité cardiaque de l'hyperkaliémie (option 3 correcte), ce qui nous donne plus de temps pour commencer les traitements hypokaliémiques (initialement la glycémie avec de l'insuline, du salbutamol, du furosémide, et évaluer la dialyse s'il n'y a pas de bonne réponse).", "type": "NEPHROLOGIE", "options": {"1": "Administration de résines échangeuses de cations.", "2": "Mise en place d'un cathéter pour initier la dialyse.", "3": "Administration de gluconate de calcium par voie intraveineuse.", "4": "Administration de furosémide par voie orale.", "5": null}, "correct_option": 3, "explanations": {"1": {"exist": false, "char_ranges": [], "word_ranges": [], "text": ""}, "2": {"exist": false, "char_ranges": [], "word_ranges": [], "text": ""}, "3": {"exist": true, "char_ranges": [[0, 287]], "word_ranges": [[0, 41]], "text": "On nous décrit le cas d'un patient présentant une urgence hyperkaliémique (potassium >6,5 mmol/l avec altérations de l'ECG). La première étape consiste à administrer du gluconate de calcium par voie intraveineuse pour contrer la toxicité cardiaque de l'hyperkaliémie (option 3 correcte),"}, "4": {"exist": false, "char_ranges": [], "word_ranges": [], "text": ""}, "5": {"exist": false, "char_ranges": [], "word_ranges": [], "text": ""}}} +{"id": 494, "year": 2020, "question_id_specific": 157, "full_question": "Une femme de 26 ans a consulté pour une sensation de faiblesse généralisée qui s'est progressivement développée au cours des trois dernières semaines, devenant particulièrement intense au cours des deux derniers jours. Au cours des deux dernières années, elle a signalé des épisodes de douleurs articulaires dans les mains qui ont nécessité la prise d'anti-inflammatoires, ainsi que l'apparition de quelques lésions érythémateuses de cause incertaine dans la région du décolleté, principalement en été. L'examen physique n'a révélé qu'une pâleur de la peau et des muqueuses et une fréquence cardiaque de 100 bpm. L'hémogramme montre : Hb 6 gr/dL, Hto 27 %, MCV 105 fL, 3 420 leucocytes/mm3 (2300 neutrophiles/mm3, 800 lymphocytes/mm3, 250 monocytes/mm3, 50 éosinophiles/mm3, 20 basophiles/mm3), plaquettes 170 000/mm3. Biochimie : AST 30 UI/L, ALT 35 UI/L, GGT 59 UI/L, phosphatase alcaline 105 UI/L, LDH 490 UI/L, urée 20 mg/dL, créatinine 0,8 mg/dL. Compte tenu des informations disponibles, veuillez indiquer, parmi les paramètres analytiques supplémentaires suivants, ceux qu'il vous faudrait connaître pour prendre la décision immédiate la plus appropriée :", "full_answer": "Ils discutent du cas d'une jeune femme présentant des symptômes d'une maladie systémique d'origine auto-immune probable (lupus) qui présente une anémie macrocytaire, les options 2 et 4 sont des options pour identifier la maladie auto-immune mais ce qu'ils nous demandent est ce qu'il faut faire pour traiter l'anémie, donc nous devrions suspecter que l'anémie est secondaire à la maladie auto-immune et donc demander un test de Coombs direct, car s'il est positif, le patient ne devrait pas être transfusé, sauf si c'est extrêmement grave, et la cause primaire devrait être traitée. Il est également question d'une apparition rapide, alors que dans le cas de la vitamine B12, la maladie est plus progressive.", "type": "HÉMATOLOGIE", "options": {"1": "Vitamine B12.", "2": "Anticorps antinucléaires.", "3": "Test de Coombs direct.", "4": "Anticorps anti-ADN.", "5": null}, "correct_option": 3, "explanations": {"1": {"exist": false, "char_ranges": [], "word_ranges": [], "text": ""}, "2": {"exist": true, "char_ranges": [[0, 316]], "word_ranges": [[0, 48]], "text": "Ils discutent du cas d'une jeune femme présentant des symptômes d'une maladie systémique d'origine auto-immune probable (lupus) qui présente une anémie macrocytaire, les options 2 et 4 sont des options pour identifier la maladie auto-immune mais ce qu'ils nous demandent est ce qu'il faut faire pour traiter l'anémie,"}, "3": {"exist": true, "char_ranges": [[323, 582]], "word_ranges": [[49, 91]], "text": "nous devrions suspecter que l'anémie est secondaire à la maladie auto-immune et donc demander un test de Coombs direct, car s'il est positif, le patient ne devrait pas être transfusé, sauf si c'est extrêmement grave, et la cause primaire devrait être traitée."}, "4": {"exist": true, "char_ranges": [[0, 316]], "word_ranges": [[0, 48]], "text": "Ils discutent du cas d'une jeune femme présentant des symptômes d'une maladie systémique d'origine auto-immune probable (lupus) qui présente une anémie macrocytaire, les options 2 et 4 sont des options pour identifier la maladie auto-immune mais ce qu'ils nous demandent est ce qu'il faut faire pour traiter l'anémie,"}, "5": {"exist": false, "char_ranges": [], "word_ranges": [], "text": ""}}} +{"id": 117, "year": 2012, "question_id_specific": 112, "full_question": "Un homme de 64 ans consulte pour fièvre, toux, dyspnée et douleur pleurétique droite depuis une semaine. La radiographie du thorax montre un épanchement pleural droit localisé occupant les deux tiers de l'hémithorax. Lors de la thoracentèse, un liquide jaunâtre est obtenu et l'analyse montre : leucocytes 15 000/uL, 92% de neutrophiles, glucose 30 mg/dl, pH 7, lactate déshydrogénase 3500 U/L, adénosine désaminase 45 U/L et absence de germes à la coloration GRAM. Quelle est la conduite à tenir la plus appropriée chez ce patient ?", "full_answer": "Dans cette question, il ne devrait pas y avoir beaucoup de doute non plus (d'ailleurs, je pense que c'est plus de la Pneumologie que la mienne, mais bon, avec cette fièvre mon ami Emilienko s'est égaré), puisque le tableau clinique suggère un épanchement pleural parapneumonique, les caractéristiques macroscopiques du liquide le semblent et son analyse montre que l'épanchement est aussi compliqué, presque à la limite d'être un empyème (LDH très élevée, cellularité avec prédominance de PMN, consommation de glucose). Avec ce pH, en outre, (et sachant que l'épanchement est déjà localisé) le tube endothoracique est plus qu'indiqué.", "type": "INFECTOLOGIE", "options": {"1": "Antibiothérapie intraveineuse.", "2": "Antibiothérapie intraveineuse et nouvelle thoracentèse diagnostique dans les 24 heures.", "3": "Antibiothérapie intraveineuse et nouvelle thoracentèse diagnostique en l'absence d'amélioration dans les 48 heures.", "4": "Antibiothérapie intraveineuse et thoracentèse évacuatrice (thérapeutique) si des germes sont isolés dans la culture du liquide pleural.", "5": "Antibiothérapie intraveineuse et mise en place d'un drain thoracique ou d'un cathéter pour drainer tout le liquide pleural."}, "correct_option": 5, "explanations": {"1": {"exist": false, "char_ranges": [], "word_ranges": [], "text": ""}, "2": {"exist": false, "char_ranges": [], "word_ranges": [], "text": ""}, "3": {"exist": false, "char_ranges": [], "word_ranges": [], "text": ""}, "4": {"exist": false, "char_ranges": [], "word_ranges": [], "text": ""}, "5": {"exist": true, "char_ranges": [[223, 634]], "word_ranges": [[39, 97]], "text": "clinique suggère un épanchement pleural parapneumonique, les caractéristiques macroscopiques du liquide le semblent et son analyse montre que l'épanchement est aussi compliqué, presque à la limite d'être un empyème (LDH très élevée, cellularité avec prédominance de PMN, consommation de glucose). Avec ce pH, en outre, (et sachant que l'épanchement est déjà localisé) le tube endothoracique est plus qu'indiqué."}}} +{"id": 27, "year": 2011, "question_id_specific": 203, "full_question": "Un patient de 52 ans a été admis à l'hôpital pour une pneumonie sévère. Grâce à un traitement antibiotique approprié, les symptômes respiratoires du patient se sont améliorés. Après 4 jours dans le service, son évolution est compliquée par l'apparition d'un processus diarrhéique sévère. Quel est le micro-organisme le plus fréquemment responsable de cette affection ?", "full_answer": "Dans ce cas, compte tenu du traitement antibiotique administré et de la gravité de la diarrhée, il semble que nous soyons en présence d'une infection à Clostridium. L'option 2 est peu crédible, en effet, bien que les patients atteints de PAC puissent avoir une diarrhée associée, il est peu probable qu'une bactérie soit sensible à un antibiotique au début des symptômes et cesse de l'être au bout de 3 jours.", "type": "INFECTIEUX", "options": {"1": "Salmonella enterica.", "2": "La bactérie responsable de la pneumonie elle-même, qui est devenue résistante à l'antibiotique.", "3": "Campylobacter jejuni.", "4": "Clostridium difficile.", "5": "Yersinia enterocolitica."}, "correct_option": 4, "explanations": {"1": {"exist": false, "char_ranges": [], "word_ranges": [], "text": ""}, "2": {"exist": true, "char_ranges": [[165, 409]], "word_ranges": [[27, 70]], "text": "L'option 2 est peu crédible, en effet, bien que les patients atteints de PAC puissent avoir une diarrhée associée, il est peu probable qu'une bactérie soit sensible à un antibiotique au début des symptômes et cesse de l'être au bout de 3 jours."}, "3": {"exist": false, "char_ranges": [], "word_ranges": [], "text": ""}, "4": {"exist": true, "char_ranges": [[13, 164]], "word_ranges": [[3, 27]], "text": "compte tenu du traitement antibiotique administré et de la gravité de la diarrhée, il semble que nous soyons en présence d'une infection à Clostridium."}, "5": {"exist": false, "char_ranges": [], "word_ranges": [], "text": ""}}} +{"id": 471, "year": 2020, "question_id_specific": 139, "full_question": "Un garçon de 7 ans est amené aux urgences après être tombé d'une balançoire sur sa main droite. Il n'a pas d'antécédents particuliers. Il présente une déformation dorsale en fourche du poignet et une impotence fonctionnelle, avec une situation neurovasculaire distale normale. Quelle lésion vous attendez-vous à trouver sur la radiographie urgente que vous demandez ?", "full_answer": "Compte tenu de la déformation et de la douleur au niveau du radius distal, nous nous attendons à une fracture à ce niveau, et nous marquons donc 3. Cependant, je considère qu'il n'y a pas de doute, étant donné qu'il pourrait également y avoir une fracture concomitante au niveau de la tige verte de l'ulna, même si la lésion principale est au niveau du radius.", "type": "CHIRURGIE ORTHOPÉDIQUE ET TRAUMATOLOGIE", "options": {"1": "Fracture de Monteggia - luxation.", "2": "Fracture de la tête radiale.", "3": "Epiphysiolyse du radius distal.", "4": "Fracture de la tige verte du cubitus.", "5": null}, "correct_option": 3, "explanations": {"1": {"exist": true, "char_ranges": [[0, 122]], "word_ranges": [[0, 23]], "text": "Compte tenu de la déformation et de la douleur au niveau du radius distal, nous nous attendons à une fracture à ce niveau,"}, "2": {"exist": true, "char_ranges": [[0, 122]], "word_ranges": [[0, 23]], "text": "Compte tenu de la déformation et de la douleur au niveau du radius distal, nous nous attendons à une fracture à ce niveau,"}, "3": {"exist": true, "char_ranges": [[74, 147]], "word_ranges": [[13, 28]], "text": "distal, nous nous attendons à une fracture à ce niveau, et nous marquons donc 3."}, "4": {"exist": false, "char_ranges": [], "word_ranges": [], "text": ""}, "5": {"exist": false, "char_ranges": [], "word_ranges": [], "text": ""}}} +{"id": 327, "year": 2016, "question_id_specific": 81, "full_question": "Une femme de 30 ans se présente à la clinique en faisant état d'anxiété, d'une perte de poids d'environ 6 kg et d'un sentiment de \"nervosité\" au cours des trois derniers mois. L'examen physique révèle une tachycardie, une hyperréflexie et l'absence de goitre. Une scintigraphie révèle une absence de captation dans la région thyroïdienne. Quel est, selon vous, le diagnostic le plus probable ?", "full_answer": "Réponse 2 incorrecte : La maladie de Graves se caractérise par une hyperthyroïdie et une ou plusieurs des caractéristiques suivantes : goitre, exophtalmie et myxœdème prétibial. Réponse 4 incorrecte : Le symptôme le plus évident de la thyroïdite subaiguë est la douleur au niveau du cou. Parfois, la douleur peut s'étendre à la mâchoire ou aux oreilles. L'hypertrophie douloureuse de la glande thyroïde peut durer des semaines ou des mois. Le taux sérique de thyréostimuline (TSH) est généralement bas. Un taux élevé de T4 libre dans le sérum. Un taux élevé de thyroglobuline sérique. Mauvaise réponse 4 : tumeur très rare.", "type": "ENDOCRINOLOGIE", "options": {"1": "Thyrotoxicose factice.", "2": "Hyperthyroïdie due à la maladie de Graves.", "3": "Tératome de l'ovaire (stroma ovarien).", "4": "Thyroïdite subaiguë.", "5": null}, "correct_option": 1, "explanations": {"1": {"exist": false, "char_ranges": [], "word_ranges": [], "text": ""}, "2": {"exist": true, "char_ranges": [[0, 177]], "word_ranges": [[0, 26]], "text": "Réponse 2 incorrecte : La maladie de Graves se caractérise par une hyperthyroïdie et une ou plusieurs des caractéristiques suivantes : goitre, exophtalmie et myxœdème prétibial."}, "3": {"exist": false, "char_ranges": [], "word_ranges": [], "text": ""}, "4": {"exist": true, "char_ranges": [[178, 353]], "word_ranges": [[26, 57]], "text": "Réponse 4 incorrecte : Le symptôme le plus évident de la thyroïdite subaiguë est la douleur au niveau du cou. Parfois, la douleur peut s'étendre à la mâchoire ou aux oreilles."}, "5": {"exist": false, "char_ranges": [], "word_ranges": [], "text": ""}}} +{"id": 450, "year": 2018, "question_id_specific": 154, "full_question": "Un homme présente à l'examen neurologique un déficit sensitif thermoalgésique de la jambe gauche associé à une perte de sensibilité vibratoire et positionnelle de la jambe droite. Parallèlement, il présente une maladresse et une perte de force distale de la jambe droite ainsi qu'un réflexe cutané plantaire droit en extension. Laquelle des affirmations suivantes est correcte ?", "full_answer": "Ils décrivent une hémisection typique ou un tableau de Brown-Séquard. Atteinte de la motricité et des cordes postérieures ipsilatérales avec douleur et température contralatérales.", "type": "NEUROSURGÉRIE", "options": {"1": "Il s'agit d'un syndrome de type syringomyélie centromédullaire.", "2": "Il s'agit d'un syndrome hémimédullaire.", "3": "Il s'agit d'une lésion transversale de la moelle épinière.", "4": "Il s'agit d'une lésion bulbaire latérale.", "5": null}, "correct_option": 2, "explanations": {"1": {"exist": false, "char_ranges": [], "word_ranges": [], "text": ""}, "2": {"exist": true, "char_ranges": [[0, 180]], "word_ranges": [[0, 24]], "text": "Ils décrivent une hémisection typique ou un tableau de Brown-Séquard. Atteinte de la motricité et des cordes postérieures ipsilatérales avec douleur et température contralatérales."}, "3": {"exist": false, "char_ranges": [], "word_ranges": [], "text": ""}, "4": {"exist": false, "char_ranges": [], "word_ranges": [], "text": ""}, "5": {"exist": false, "char_ranges": [], "word_ranges": [], "text": ""}}} +{"id": 265, "year": 2014, "question_id_specific": 137, "full_question": "Un homme de 65 ans, ayant des antécédents de néoplasie pancréatique, est sous chimiothérapie. Il a consulté le service des urgences pour des douleurs et un œdème de tout le membre inférieur à partir de l'aine. Quel est le test diagnostique le plus rentable pour confirmer le diagnostic suspecté ?", "full_answer": "L'échographie Doppler est le test le plus précis et permet un suivi ultérieur non invasif de l'évolution de la maladie.", "type": "PNEUMOLOGIE", "options": {"1": "D-dimère.", "2": "Imagerie par résonance magnétique.", "3": "Phlébographie.", "4": "Échographie Doppler veineuse.", "5": "CT hélicoïdal."}, "correct_option": 4, "explanations": {"1": {"exist": false, "char_ranges": [], "word_ranges": [], "text": ""}, "2": {"exist": false, "char_ranges": [], "word_ranges": [], "text": ""}, "3": {"exist": false, "char_ranges": [], "word_ranges": [], "text": ""}, "4": {"exist": true, "char_ranges": [[0, 119]], "word_ranges": [[0, 20]], "text": "L'échographie Doppler est le test le plus précis et permet un suivi ultérieur non invasif de l'évolution de la maladie."}, "5": {"exist": false, "char_ranges": [], "word_ranges": [], "text": ""}}} +{"id": 189, "year": 2013, "question_id_specific": 71, "full_question": "Garçon de 10 ans présentant de brefs épisodes de distraction (< 1 minute) au cours desquels il ne répond pas aux appels et cligne des yeux. Un EEG montre des décharges de pointes-ondes à 3 cycles par seconde. Le traitement électif de première intention serait l'utilisation de :", "full_answer": "Les crises décrites et l'EEG correspondent à des crises d'absence typiques et le traitement initial de choix est l'acide valproïque.", "type": "PÉDIATRIE", "options": {"1": "Valproate.", "2": "Carbamazépine.", "3": "Phénytoïne.", "4": "Gabapentin.", "5": "Clonazepam."}, "correct_option": 1, "explanations": {"1": {"exist": true, "char_ranges": [[0, 132]], "word_ranges": [[0, 20]], "text": "Les crises décrites et l'EEG correspondent à des crises d'absence typiques et le traitement initial de choix est l'acide valproïque."}, "2": {"exist": false, "char_ranges": [], "word_ranges": [], "text": ""}, "3": {"exist": false, "char_ranges": [], "word_ranges": [], "text": ""}, "4": {"exist": false, "char_ranges": [], "word_ranges": [], "text": ""}, "5": {"exist": false, "char_ranges": [], "word_ranges": [], "text": ""}}} +{"id": 508, "year": 2021, "question_id_specific": 148, "full_question": "Une femme de 72 ans sans antécédents intéressants. Après une colique néphrétique compliquée, un scanner abdominal a révélé une lésion kystique de 2 cm dans la queue du pancréas, ainsi que de multiples lésions kystiques rénales bilatérales. L'échographie endoscopique montre une lésion polykystique constituée de multiples vésicules avec une calcification centrale dans la queue du pancréas sans connexion avec le canal de Wirsung. L'analyse du liquide est compatible avec un cystadénome séreux. Laquelle des approches suivantes est la plus correcte pour la prise en charge de ce patient ?", "full_answer": "Le cystadénome séreux du pancréas est une entité bénigne, souvent polykystique (également appelée adénome microkystique), constituée de cellules productrices de collagène (non mucineuses). Il est généralement asymptomatique et découvert fortuitement. Compte tenu de sa nature bénigne, la chirurgie n'est pas recommandée et n'est réservée qu'aux personnes symptomatiques ou en cas de doute sur la nature de la lésion après une étude complète par scanner, IRM et échoendoscopie avec biopsie. Elle est souvent sporadique, mais peut être associée au syndrome de Von-Hippel-Lindau.", "type": "DIGESTIF", "options": {"1": "Résection chirurgicale (pancréatectomie corporocaudale).", "2": "Ponction endoscopique guidée par ultrasons et éthanolisation de la ponction.", "3": "Suivi de la lésion par IRM.", "4": "Transplantation rein-pancréas.", "5": null}, "correct_option": 3, "explanations": {"1": {"exist": true, "char_ranges": [[251, 319]], "word_ranges": [[30, 41]], "text": "Compte tenu de sa nature bénigne, la chirurgie n'est pas recommandée"}, "2": {"exist": true, "char_ranges": [[251, 319]], "word_ranges": [[30, 41]], "text": "Compte tenu de sa nature bénigne, la chirurgie n'est pas recommandée"}, "3": {"exist": true, "char_ranges": [[251, 319]], "word_ranges": [[30, 41]], "text": "Compte tenu de sa nature bénigne, la chirurgie n'est pas recommandée"}, "4": {"exist": true, "char_ranges": [[251, 319]], "word_ranges": [[30, 41]], "text": "Compte tenu de sa nature bénigne, la chirurgie n'est pas recommandée"}, "5": {"exist": false, "char_ranges": [], "word_ranges": [], "text": ""}}} +{"id": 151, "year": 2012, "question_id_specific": 70, "full_question": "Une femme de 25 ans, ayant des antécédents d'éruption cutanée après exposition au soleil et de polyarthrite occasionnelle dans les articulations des mains, contrôlée par des anti-inflammatoires non stéroïdiens, s'est présentée il y a 15 jours avec un malaise général, une sensation progressive de faiblesse généralisée et une pâleur. Les analyses sanguines montrent une hémoglobine à 7 g/dL, un MCV à 108 mm/h, 150 000 plaquettes/mm3, 3000 leucocytes/mm3, une LDH élevée, une haptoglobine indétectable. Si vous ne choisissez qu'un seul test diagnostique, indiquez lequel des tests suivants doit être effectué en premier :", "full_answer": "Le tableau clinique et biologique de cette patiente fait suspecter une anémie hémolytique dans le cadre d'un lupus érythémateux disséminé. Si nous devons choisir un seul test diagnostique, et ce devrait être le premier, nous sommes intéressés à confirmer la présence d'une anémie hémolytique auto-immune par le test de Coombs direct afin de commencer le traitement. Les autres tests peuvent attendre...", "type": "RHEUMATOLOGIE", "options": {"1": "Test de Coombs direct.", "2": "Anticorps antinucléaires (ANA).", "3": "Vitamine B12.", "4": "Ferritine.", "5": "Acide folique."}, "correct_option": 1, "explanations": {"1": {"exist": true, "char_ranges": [[0, 365]], "word_ranges": [[0, 56]], "text": "Le tableau clinique et biologique de cette patiente fait suspecter une anémie hémolytique dans le cadre d'un lupus érythémateux disséminé. Si nous devons choisir un seul test diagnostique, et ce devrait être le premier, nous sommes intéressés à confirmer la présence d'une anémie hémolytique auto-immune par le test de Coombs direct afin de commencer le traitement."}, "2": {"exist": true, "char_ranges": [[366, 402]], "word_ranges": [[56, 61]], "text": "Les autres tests peuvent attendre..."}, "3": {"exist": true, "char_ranges": [[366, 402]], "word_ranges": [[56, 61]], "text": "Les autres tests peuvent attendre..."}, "4": {"exist": true, "char_ranges": [[366, 402]], "word_ranges": [[56, 61]], "text": "Les autres tests peuvent attendre..."}, "5": {"exist": true, "char_ranges": [[366, 402]], "word_ranges": [[56, 61]], "text": "Les autres tests peuvent attendre..."}}} +{"id": 404, "year": 2016, "question_id_specific": 139, "full_question": "Une femme de 70 ans ayant des antécédents d'anorexie, de perte de poids, de gêne au niveau de la musculature et des articulations proximales ainsi que de douleurs dans la région temporo-mandibulaire se présente aux urgences pour une perte de vision unilatérale (mouvement de la main), d'apparition soudaine et indolore (anomalie pupillaire afférente). Quel examen demanderiez-vous en premier lieu à des fins diagnostiques ?", "full_answer": "Il s'agit d'une femme qui a subi une perte soudaine, indolore et sévère de la vision d'un œil (elle ne voit que le mouvement de ses mains). L'examen ophtalmologique ne nous donne pas beaucoup d'informations, si ce n'est qu'elle présente un défaut de pupillarité afférente. Nous devons nous fier aux symptômes systémiques pour diagnostiquer le problème. Il s'agit d'une femme âgée qui présente les symptômes typiques de la polymyalgie rhumatismale, et la douleur dans la région temporo-mandibulaire est très suggestive. Il s'agit d'une artérite à cellules géantes ou d'une artérite temporale. La manifestation oculaire la plus fréquente est la neuropathie optique ischémique antérieure. Au fond d'œil, on observera probablement un œdème papillaire de l'œil, de couleur blanc gypseux. Un défaut de pupille afférente est très caractéristique de ces neuropathies optiques ischémiques graves. En cas de suspicion d'artérite à cellules géantes, trois marqueurs analytiques sont typiquement élevés : la CRP, l'ESR et la numération plaquettaire. Nous demanderons d'abord l'un de ces paramètres et le traiterons souvent sans attendre la confirmation de la biopsie. L'option 2 est donc correcte.", "type": "OPHTALMOLOGIE (ECTOPIQUE)", "options": {"1": "Ponction lombaire.", "2": "Protéine C-réactive.", "3": "Angiographie par résonance magnétique.", "4": "Échographie de la carotide.", "5": null}, "correct_option": 2, "explanations": {"1": {"exist": false, "char_ranges": [], "word_ranges": [], "text": ""}, "2": {"exist": true, "char_ranges": [[888, 1037]], "word_ranges": [[132, 154]], "text": "En cas de suspicion d'artérite à cellules géantes, trois marqueurs analytiques sont typiquement élevés : la CRP, l'ESR et la numération plaquettaire."}, "3": {"exist": false, "char_ranges": [], "word_ranges": [], "text": ""}, "4": {"exist": false, "char_ranges": [], "word_ranges": [], "text": ""}, "5": {"exist": false, "char_ranges": [], "word_ranges": [], "text": ""}}} +{"id": 215, "year": 2014, "question_id_specific": 106, "full_question": "Un patient de 68 ans consulte pour des œdèmes et une asthénie. Un bilan sanguin montre une créatinine à 5 mg/dl, une hémoglobine à 10 g/dl et une hypogammaglobulinémie marquée dans le sérum aux dépens des IgG, IgA et IgM. Une analyse d'urine révèle la présence de chaînes légères kappa. Quelle est votre suspicion diagnostique ?", "full_answer": "Il s'agit d'une question mixte hémato-néphro. Elle est compliquée et nous allons donc écarter les options une par une. L'option 2, le syndrome néphrotique, en plus d'être large et non spécifique, ne serait pas évaluable, car on ne nous donne pas la quantification des protéines dans l'urine (on nous dit seulement que l'analyse de l'urine révèle la présence de chaînes légères kappa, mais pas la quantité). L'option 3 est peu probable ; dans l'amylose, les chaînes légères qui ont tendance à se déposer sont de type lambda. L'option 4 semble également peu probable, car dans le myélome IgA, on s'attendrait à un pic monoclonal de cette Ig, qui n'apparaît pas dans ce cas. Le doute qui se pose à moi est entre 1 et 5 ; j'opterais pour 1, car cette entité remplit les caractéristiques décrites (absence de pic monoclonal dans la plupart des cas, insuffisance rénale aiguë et présence de chaînes kappa dans les urines), le myélome à chaînes légères étant une entité un peu plus large, mais l'option 5 pourrait également être considérée comme valable.", "type": "NEPHROLOGIE", "options": {"1": "Maladie des dépôts de chaînes légères kappa.", "2": "Syndrome néphrotique.", "3": "Amyloïdose.", "4": "Myélome IgA avec protéinurie de Bence-Jones.", "5": "Myélome à chaîne légère."}, "correct_option": 1, "explanations": {"1": {"exist": true, "char_ranges": [[718, 916]], "word_ranges": [[126, 156]], "text": "j'opterais pour 1, car cette entité remplit les caractéristiques décrites (absence de pic monoclonal dans la plupart des cas, insuffisance rénale aiguë et présence de chaînes kappa dans les urines),"}, "2": {"exist": true, "char_ranges": [[119, 406]], "word_ranges": [[19, 66]], "text": "L'option 2, le syndrome néphrotique, en plus d'être large et non spécifique, ne serait pas évaluable, car on ne nous donne pas la quantification des protéines dans l'urine (on nous dit seulement que l'analyse de l'urine révèle la présence de chaînes légères kappa, mais pas la quantité)."}, "3": {"exist": true, "char_ranges": [[407, 523]], "word_ranges": [[66, 87]], "text": "L'option 3 est peu probable ; dans l'amylose, les chaînes légères qui ont tendance à se déposer sont de type lambda."}, "4": {"exist": true, "char_ranges": [[524, 671]], "word_ranges": [[87, 113]], "text": "L'option 4 semble également peu probable, car dans le myélome IgA, on s'attendrait à un pic monoclonal de cette Ig, qui n'apparaît pas dans ce cas."}, "5": {"exist": true, "char_ranges": [[917, 980]], "word_ranges": [[156, 168]], "text": "le myélome à chaînes légères étant une entité un peu plus large,"}}} +{"id": 264, "year": 2014, "question_id_specific": 135, "full_question": "Une femme de 24 ans a été trouvée allongée dans la rue par des passants. À l'arrivée de l'équipe d'urgence, elle présentait une saturation en oxygène de 88 %, respirait l'air ambiant et présentait des pupilles en pointe à l'examen. Elle a été transportée au service des urgences de l'hôpital le plus proche, où les gaz du sang artériel de référence ont montré : pH 7,25, PaC02 60 mmHg, Pa02 58 mmHg, bicarbonate 26 mEq/1 et excès de base de -1. Dans le sang, le sodium était de 137 mEq/1 et le chlorure de 100 mEq/1 :", "full_answer": "Une acidose avec une PCO2 élevée, une hypoxémie et un taux de bicarbonate normal suggère une acidose respiratoire aiguë d'apparition rapide, probablement due à une intoxication médicamenteuse.", "type": "PNEUMOLOGIE", "options": {"1": "Insuffisance respiratoire partielle.", "2": "Acidose métabolique.", "3": "Acidose respiratoire pure.", "4": "Alcalose respiratoire due au manque de chlore.", "5": "Les gaz du sang ne peuvent être obtenus qu'à partir du sang veineux."}, "correct_option": 3, "explanations": {"1": {"exist": false, "char_ranges": [], "word_ranges": [], "text": ""}, "2": {"exist": false, "char_ranges": [], "word_ranges": [], "text": ""}, "3": {"exist": true, "char_ranges": [[0, 192]], "word_ranges": [[0, 27]], "text": "Une acidose avec une PCO2 élevée, une hypoxémie et un taux de bicarbonate normal suggère une acidose respiratoire aiguë d'apparition rapide, probablement due à une intoxication médicamenteuse."}, "4": {"exist": false, "char_ranges": [], "word_ranges": [], "text": ""}, "5": {"exist": false, "char_ranges": [], "word_ranges": [], "text": ""}}} +{"id": 594, "year": 2022, "question_id_specific": 80, "full_question": "Un nourrisson de 2 mois souffrant d'un rhume des voies respiratoires supérieures depuis 3 jours, qui commence par présenter une détresse respiratoire modérée et une auscultation pulmonaire avec une respiration sifflante expiratoire. La saturation en oxygène est de 89 %. Le virus respiratoire syncytial est isolé dans l'exsudat nasopharyngé. Parmi les traitements suivants, lequel vous semble le plus approprié pour cette pathologie ?", "full_answer": "Nous présentons un cas de bronchiolite à VRS positif. Selon les recommandations actuelles, une oxygénothérapie complémentaire doit être mise en place.", "type": "PÉDIATRIE", "options": {"1": "Rivabirine orale.", "2": "Salbutamol en nébulisation.", "3": "Oxygène supplémentaire.", "4": "Corticostéroïdes intraveineux.", "5": null}, "correct_option": 3, "explanations": {"1": {"exist": false, "char_ranges": [], "word_ranges": [], "text": ""}, "2": {"exist": false, "char_ranges": [], "word_ranges": [], "text": ""}, "3": {"exist": true, "char_ranges": [[0, 150]], "word_ranges": [[0, 21]], "text": "Nous présentons un cas de bronchiolite à VRS positif. Selon les recommandations actuelles, une oxygénothérapie complémentaire doit être mise en place."}, "4": {"exist": false, "char_ranges": [], "word_ranges": [], "text": ""}, "5": {"exist": false, "char_ranges": [], "word_ranges": [], "text": ""}}} +{"id": 236, "year": 2014, "question_id_specific": 108, "full_question": "Une femme de 67 ans, sous traitement à la ticlopidine, s'est présentée au service des urgences avec des céphalées, une asthénie et des pétéchies aux extrémités inférieures. Les analyses sanguines ont révélé une hémoglobine de 8,2 g/dl, un MCV de 100 fl, des plaquettes de 25 000/ul et des leucocytes de 7 500/ul avec une formule normale. Le nombre de réticulocytes était élevé et le frottis sanguin montrait de nombreux schistocytes. Les études de coagulation (TCA, TP et fibrinogène) sont normales. La biochimie montre une LDH de 2700 UI/l et une bilirubine de 2,6 mg/dl. Quel est le diagnostic le plus probable ?", "full_answer": "Typique de la ticlopidine. Non seulement thrombocytopénie mais aussi céphalées, schistocytes, LDH et bilirubine élevées.", "type": "HÉMATOLOGIE", "options": {"1": "Purpura thrombocytopénique auto-immun.", "2": "Purpura thrombocytopénique thrombotique.", "3": "Aplasie de la moelle épinière.", "4": "Thrombopénie médicamenteuse.", "5": "Coagulation intravasculaire disséminée."}, "correct_option": 2, "explanations": {"1": {"exist": false, "char_ranges": [], "word_ranges": [], "text": ""}, "2": {"exist": true, "char_ranges": [[0, 120]], "word_ranges": [[0, 15]], "text": "Typique de la ticlopidine. Non seulement thrombocytopénie mais aussi céphalées, schistocytes, LDH et bilirubine élevées."}, "3": {"exist": false, "char_ranges": [], "word_ranges": [], "text": ""}, "4": {"exist": false, "char_ranges": [], "word_ranges": [], "text": ""}, "5": {"exist": false, "char_ranges": [], "word_ranges": [], "text": ""}}}